You are on page 1of 290

TEORIA

CALCOLO
INTEGRALE
Indice

I Teoria dell’integrazione di una funzione reale di vari-


abile reale 5
1 Integrale definito 6
1.1 Funzioni primitive . . . . . . . . . . . . . . . . . . . . . . . . . . . 6
1.2 Misura degli insiemi piani . . . . . . . . . . . . . . . . . . . . . . 8
1.3 Area del rettangoloide . . . . . . . . . . . . . . . . . . . . . . . . 14
1.4 Definizione di integrale definito . . . . . . . . . . . . . . . . . . . 19
1.5 Proprietà dell’integrale definito . . . . . . . . . . . . . . . . . . . 22
1.5.1 Proprietà additiva . . . . . . . . . . . . . . . . . . . . . . 23
1.5.2 Proprietà distributiva . . . . . . . . . . . . . . . . . . . . . 23
1.6 Teorema della media . . . . . . . . . . . . . . . . . . . . . . . . . 23
1.7 Teorema fondamentale del calcolo integrale . . . . . . . . . . . . . 25

2 Integrale indefinito 28
2.1 Definizione di integrale indefinito . . . . . . . . . . . . . . . . . . 28
2.2 Integrali indefiniti fondamentali . . . . . . . . . . . . . . . . . . . 29

II Esercizi sull’integrazione di una funzione reale di vari-


abile reale 31
3 Integrali indefiniti 32
3.1 Integrali indefiniti fondamentali . . . . . . . . . . . . . . . . . . . 32
3.2 Soluzioni . . . . . . . . . . . . . . . . . . . . . . . . . . . . . . . . 32
3.2.1 Soluzioni . . . . . . . . . . . . . . . . . . . . . . . . . . . . 34
3.3 Integrali di somme di funzioni . . . . . . . . . . . . . . . . . . . . 34
3.3.1 Soluzioni . . . . . . . . . . . . . . . . . . . . . . . . . . . . 34
3.4 Integrali di una f (ξ (x)), con ξ (x) funzione lineare . . . . . . . . 36
3.4.1 Soluzioni . . . . . . . . . . . . . . . . . . . . . . . . . . . . 36
3.5 Integrazione per introduzione sotto il segno di integrale . . . . . . 39
3.5.1 Calcolare gli integrali: . . . . . . . . . . . . . . . . . . . . 40
3.5.2 Soluzioni . . . . . . . . . . . . . . . . . . . . . . . . . . . . 40
3.5.3 Soluzioni . . . . . . . . . . . . . . . . . . . . . . . . . . . . 49
INDICE 2

3.5.4 Calcolare gli integrali: . . . . . . . . . . . . . . . . . . . . 57


3.6 Integrazione per sostituzione . . . . . . . . . . . . . . . . . . . . . 61
3.6.1 Calcolare gli integrali: . . . . . . . . . . . . . . . . . . . . 64
3.6.2 Soluzioni . . . . . . . . . . . . . . . . . . . . . . . . . . . . 64
3.6.3 Calcolare i seguenti integrali utilizzando le sostituzioni trigono-
Z metriche
Z . . . . . . . . . . . . . . . . . . . . . . . . . . . . 69
3.7 sin xdx, cos2 xdx . . . . . . . . . . . . . . . . . . . . . . . . 72
2

3.8 Integrazione per parti . . . . . . . . . . . . . . . . . . . . . . . . . 73


3.8.1 Calcolare gli integrali: . . . . . . . . . . . . . . . . . . . . 74
3.8.2 Soluzioni . . . . . . . . . . . . . . . . . . . . . . . . . . . . 74
3.8.3 Calcolare gli integrali: . . . . . . . . . . . . . . . . . . . . 77
3.8.4 Soluzioni . . . . . . . . . . . . . . . . . . . . . . . . . . . . 78
3.8.5 Formule ricorrenti . . . . . . . . . . . . . . . . . . . . . . . 81
3.8.6 Calcolare i seguenti integrali applicando il metodo più op-
portuno . . . . . . . . . . . . . . . . . . . . . . . . . . . . 85
3.8.7 Soluzioni . . . . . . . . . . . . . . . . . . . . . . . . . . . . 85
3.9 Integrali contenenti un trinomio di secondo grado . . . . . . . . . 87
3.9.1 Calcolare gli integrali: . . . . . . . . . . . . . . . . . . . . 90
3.9.2 Soluzioni . . . . . . . . . . . . . . . . . . . . . . . . . . . . 91
3.9.3 Calcolare gli integrali . . . . . . . . . . . . . . . . . . . . . 95
3.9.4 Soluzioni . . . . . . . . . . . . . . . . . . . . . . . . . . . . 95
3.9.5 Calcolare gli integrali . . . . . . . . . . . . . . . . . . . . . 98
3.9.6 Soluzioni . . . . . . . . . . . . . . . . . . . . . . . . . . . . 98
3.10 Coppia di integrali notevoli . . . . . . . . . . . . . . . . . . . . . 103
3.11 Esercizi riepilogativi sugli integrali contenenti un trinomio di sec-
ondo grado . . . . . . . . . . . . . . . . . . . . . . . . . . . . . . 104
3.11.1 Calcolare i seguenti integrali . . . . . . . . . . . . . . . . . 104
3.11.2 Calcolare i seguenti integrali . . . . . . . . . . . . . . . . . 106
3.11.3 Soluzioni . . . . . . . . . . . . . . . . . . . . . . . . . . . . 107
3.12 Integrazione delle funzioni razionali . . . . . . . . . . . . . . . . . 108
3.12.1 Funzioni
Z razionali proprie . . . . . . . . . . . . . . . . . . 109
3.12.2 dx
(x2 +1)2
. . . . . . . . . . . . . . . . . . . . . . . . . . . . 117
3.12.3 Calcolare i seguenti integrali . . . . . . . . . . . . . . . . . 117
3.12.4 Soluzioni . . . . . . . . . . . . . . . . . . . . . . . . . . . . 117
3.12.5 Calcolare i seguenti integrali . . . . . . . . . . . . . . . . . 126
3.12.6 Soluzioni . . . . . . . . . . . . . . . . . . . . . . . . . . . . 127
3.12.7 Calcolare i seguenti integrali . . . . . . . . . . . . . . . . . 134
3.12.8 Soluzioni . . . . . . . . . . . . . . . . . . . . . . . . . . . . 134
3.12.9 Calcolare i seguenti integrali . . . . . . . . . . . . . . . . . 143
3.12.10 Soluzioni . . . . . . . . . . . . . . . . . . . . . . . . . . . . 143
3.12.11 Funzioni razionali improprie . . . . . . . . . . . . . . . . . 150
INDICE 3

3.12.12 Calcolare i seguenti integrali . . . . . . . . . . . . . . . . . 151


3.12.13 Soluzioni
Z . . . . . . Z
. . . . . . . . . . . . . . . . . . . . . . 151
3.13 In (x) = (x2 −1)n , Jn (x) = (x2dx
dx
+1)n
. . . . . . . . . . . . . . . . 157
3.14 Integrali di funzioni irrazionali . . . . . . . . . . . . . . . . . . . . 161
3.14.1 Integrali del “tipo 1” . . . . . . . . . . . . . . . . . . . . . 161
3.14.2 Calcolare i seguenti integrali . . . . . . . . . . . . . . . . . 162
3.14.3 Soluzioni . . . . . . . . . . . . . . . . . . . . . . . . . . . . 162
3.14.4 Integrali del “tipo 2” . . . . . . . . . . . . . . . . . . . . . 170
3.14.5 Calcolare i seguenti integrali . . . . . . . . . . . . . . . . . 172
3.14.6 Integrali del “tipo 3” . . . . . . . . . . . . . . . . . . . . . 175
3.14.7 Calcolare i seguenti integrali . . . . . . . . . . . . . . . . . 175
3.14.8 Soluzioni . . . . . . . . . . . . . . . . . . . . . . . . . . . . 176
3.14.9 Integrali del “tipo 4” . . . . . . . . . . . . . . . . . . . . . 178
3.14.10 Calcolare i seguenti integrali . . . . . . . . . . . . . . . . . 179
3.14.11 Soluzioni . . . . . . . . . . . . . . . . . . . . . . . . . . . . 180
3.15 Esercizi riepilogativi sugli integrali di funzioni irrazionali . . . . . 185
3.15.1 Calcolare i seguenti integrali . . . . . . . . . . . . . . . . . 185
3.15.2 Soluzioni . . . . . . . . . . . . . . . . . . . . . . . . . . . . 185
3.16 Integrali di funzioni trigonometriche . . . . . . . . . . . . . . . . . 198
3.16.1 Integrali del “tipo 1” . . . . . . . . . . . . . . . . . . . . . 198
3.16.2 Integrali del “tipo 2” . . . . . . . . . . . . . . . . . . . . . 204
3.16.3 Esercizi . . . . . . . . . . . . . . . . . . . . . . . . . . . . 205
3.16.4 Soluzioni . . . . . . . . . . . . . . . . . . . . . . . . . . . . 205
3.16.5 Integrali del “tipo 3” . . . . . . . . . . . . . . . . . . . . . 215
3.16.6 Esercizi . . . . . . . . . . . . . . . . . . . . . . . . . . . . 215
3.16.7 Soluzioni . . . . . . . . . . . . . . . . . . . . . . . . . . . . 215
3.16.8 Integrali del “tipo 4” . . . . . . . . . . . . . . . . . . . . . 217
3.16.9 Esercizi . . . . . . . . . . . . . . . . . . . . . . . . . . . . 218
3.16.10 Soluzioni . . . . . . . . . . . . . . . . . . . . . . . . . . . . 219
3.16.11 Esercizi riepilogativi sugli integrali trigonometrici . . . . . 228
3.17 Integrazione delle funzioni iperboliche . . . . . . . . . . . . . . . . 234
3.17.1 Esercizi . . . . . . . . . . . . . . . . . . . . . . . . . . . . 235
3.18 Sostituzioni trigonometriche ed iperboliche . . . . . . . . . . . . . 238

4 Integrali definiti 239


4.1 Somme integrali . . . . . . . . . . . . . . . . . . . . . . . . . . . . 239
4.1.1 Soluzioni . . . . . . . . . . . . . . . . . . . . . . . . . . . . 239
4.2 Teorema fondamentale del calcolo integrale. . . . . . . . . . . . . 243

5 Estensione del concetto di integrale 257


5.1 Introduzione . . . . . . . . . . . . . . . . . . . . . . . . . . . . . . 257
5.2 Rettangoloide generalizzato . . . . . . . . . . . . . . . . . . . . . 260
INDICE 4

A Integrali
Z notevoliZ 285
A.1 sin xdx, cos2 xdx . . .
2
. . . . . . . . . . . . . . . . . . . . . 285
Z
A.2 dx
(x2 −1)n
. . . . . . . . . . . . . . . . . . . . . . . . . . . . . . . . 285
Z
A.3 dx
(x2 +1)n
. . . . . . . . . . . . . . . . . . . . . . . . . . . . . . . . 286
Z Z
A.4 dx
(sin x)n
; (cosdxx)n . . . . . . . . . . . . . . . . . . . . . . . . . . . 287
Z Z
A.5 (tan x) dx; (cot x)n dx
n
. . . . . . . . . . . . . . . . . . . . . 287
Parte I

Teoria dell’integrazione di una


funzione reale di variabile reale
Capitolo 1

Integrale definito

1.1 Funzioni primitive


Sia f (x) una funzione reale definita in (a, b) ⊂ R.
Definizione. Dicesi funzione primitiva o semplicemente primitiva di f (x),
ogni funzione reale F (x) definita in (a, b) e tale che:
d
F (x) = f (x) (1.1)
dx
Il problema fondamentale del calcolo integrale consiste nel determinare eventuali
primitive di una assegnata funzione f (x).
Osserviamo innanzitutto che se F (x) è una primitiva di f (x), per ogni c ∈ R,
F (x) + c è ancora una primitiva di f (x):
d
[F (x) + c] = f (x) (1.2)
dx
In altri termini se una funzione è dotata di una primitiva, tale funzione ammette
infinite primitive. Inoltre:

(F1 (x) , F2 (x) primitive di f (x)) =⇒ (F1′ (x) = f (x) , F2′ (x) = f (x))
da cui:

F1′ (x) − F2′ (x) = 0 =⇒ F1 (x) − F2 (x) = C, essendo C una costante reale
Cioè due primitive di un’assegnata funzione f (x) differiscono per una costante
additiva.
Si conclude che se f (x) ammette una primitiva F (x) in (a, b), la totalità delle
primitive di f (x) è:

G (x) = F (x) + c, (1.3)


con c variabile da −∞ a +∞.
1.1 Funzioni primitive 7

***

Supponiamo ora che f (x) sia continua in [a, b] e non negativa. Consideriamo il
grafico di f (x), cioè la curva di equazione Γ)y = f (x)1 . Sussite la seguente

Definizione. Dicesi rettangoloide di base [a, b] relativo alla funzione f (x),


il sottoinsieme di R2 :

R = (x, y) ∈ R2 | a ≤ x ≤ b, 0 ≤ y ≤ f (x) (1.4)

Ciò premesso, consideriamo il sottoinsieme di R:

R (ξ) = {(x, y) ∈ R | a ≤ x ≤ ξ, 0 ≤ y ≤ f (x)} , (1.5)


essendo ξ ∈ [a, b]. Quindi la funzione reale di variabile reale:

F : ξ → misR (ξ) , (1.6)


essendo misR (ξ) l’area di R (ξ).
Incrementiamo la variabile ξ:

ξ → ξ + ∆ξ | ( ξ + ∆ξ) ∈ [a, b]
Da cui l’incremento di F (ξ):

∆F = F (ξ + ∆ξ) − F (ξ)
Siano:

def
m (ξ) = min f (x)
[ξ,ξ+∆ξ]
def
M (ξ) = max f (x)
[ξ,ξ+∆ξ]

Senza perdita di generalità supponiamo che sia ∆ξ > 0:


∆F
m (ξ) ∆ξ ≤ ∆F ≤ M (ξ) ∆ξ =⇒ m (ξ) ≤ ≤ M (ξ)
∆ξ
Per una nota proprietà delle funzioni continue in un intervallo chiuso:

   
∆F ∆F
m (ξ) ≤ ≤ M (ξ) =⇒ ∃ξ∗ ∈ [ξ, ξ + ∆ξ] | f (ξ∗ ) =
∆ξ f (x) è continua ∆ξ
in [ξ, ξ + ∆ξ]
1
Tale curva giace nel semipiano y ≥ 0, giacché f (x) è non negativa per ipotesi.
1.2 Misura degli insiemi piani 8

o, ciò che è lo stesso:

F (ξ + ∆ξ) − F (ξ)
∃θ ∈ [0, 1] | = f (ξ + θ∆ξ)
∆ξ
Da ciò segue (tenendo conto della continuità di f (x)):

F (ξ + ∆ξ) − F (ξ)
lim = f (ξ) (1.7)
∆ξ→0 ∆ξ
Poiché ξ è una variabile muta, la (1.7) può scriversi:

F ′ (x) = f (x) (1.8)


Cioè la funzione (1.6) che associa ad ogni ξ ∈ [a, b] l’area di R (ξ) è una primitiva
di f (x). Da tale conclusione si evince che il problema della ricerca delle funzioni
primitive di un’assegnata funzione f (x) è direttamente connesso al problema
della misura di un insieme piano.

1.2 Misura degli insiemi piani


Fissiamo un riferimento cartesiano ortogonale (Oxy). Siano A (a′ , a′′ ), B (b′ , b′′ )
punti del piano, essendo a′ ≤ a′′ , b′ ≤ b′′ .

Definizione. Dicesi rettangolo chiuso di estremi A, B, l’insieme di punti:



(x, y) ∈ R2 | a′ ≤ x ≤ b′ , a′′ ≤ y ≤ b′′ (1.9)

Definizione. Dicesi rettangolo aperto di estremi A, B, l’insieme di punti:



(x, y) ∈ R2 | a′ < x < b′ , a′′ < y < b′′ (1.10)

Le suddette definizioni si specializzano nel caso in cui vale la disuguaglianza


in senso stretto a sinistra o a destra (reattangolo semiaperto a sinistra e
rettangolo semiaperto a destra, rispettivamente).

Definizione. Il centro del rettangolo (1.9 o 1.10) è il punto C (x0 , y0 ) tale che:

a′ + b ′ a′′ + b′′
x0 = , y0 =
2 2
Le dimensioni del rettangolo (1.9 o 1.10) sono i numeri reali non negativi:

2α = b′ − a′ , 2β = b′′ − a′′

Le semidimensioni del rettangolo sono i numeri reali non negativi α, β.


1.2 Misura degli insiemi piani 9

Assegnato il rettangolo chiuso (1.9):

2x0 = a′ + b′
2α = b′ − a′ ,

da cui:

x0 − α ≤ x ≤ x0 + α,
o, ciò che è lo stesso:

|x − x0 | ≤ α
Similmente:

|y − y0 | ≤ β
Si conclude che il rettangolo chiuso (1.9) può essere scritto come:

(x, y) ∈ R2 | |x − x0 | ≤ α, |y − y0 | ≤ β
mentre il rettangolo chiuso (1.10):

(x, y) ∈ R2 | |x − x0 | < α, |y − y0 | < β
Definizione. Dicesi intorno rettangolare di centro P0 , ogni rettangolo aperto
di centro P0 .

***

Assegnato un punto P0 (x0 , y0 ) e un r ∈ (0, +∞):


Definizione. Dicesi cerchio chiuso di centro P0 e raggio r, l’insieme:

(x, y) ∈ R2 | (x − x0 )2 + (y − y0 )2 ≤ r2

Definizione. Dicesi cerchio aperto di centro P0 e raggio r, l’insieme:



(x, y) ∈ R2 | (x − x0 )2 + (y − y0 )2 < r2

Definizione. Dicesi intorno circolare di centro P0 , ogni cerchio aperto di


centro P0 .
Nel seguito ci riferiremo indifferentemente ad un intorno rettangolare o circolare
di un assegnato punto P del piano. Indicheremo tale intorno con I (P ).

***
1.2 Misura degli insiemi piani 10

Sia A ⊂ R2 .
def
Definizione. A è limitato ⇐⇒ ∃ cerchio γ ⊃ A di centro O (0, 0)
def
A è non limitato ⇐⇒ ∀ cerchio γ di centro O (0, 0), (R − γ) ∩ A 6= ∅.
def
Definizione. P0 è interno ad A ⇐⇒ ∃I (P0 ) ⊂ A
def
P0 è esterno ad A ⇐⇒ ∃I (P0 ) : I (P0 ) ∩ A = ∅
def
P0 è punto di frontiera per A ⇐⇒ ∀I (P0 ) , I (P0 ) * A, I (P0 ) ∩ A 6= ∅

Definizione. Dicesi frontiera di A l’insieme di punti:

∂A = {P | P è di frontiera per A}

Indichiamo con il simbolo Å l’insieme dei punti interni di A:


def
Å = {P ∈ A | P è punto interno}
Ciò premesso, sia u l’unità di misura dei segmenti. Conseguentemente l’unità di
misura delle aree è u2 . Indichiamo con P la classe dei poligoni. Ad ogni poligono
π ∈ P possiamo associare la sua misura µ (π) ∈ [0, +∞) rispetto all’unità di
misura u. Resta perciò definita la seguente funzione non negativa:

µ : P −→ R+
0 (1.11)
La (1.11) verifica le seguenti proprietà:

1. Proprietà additiva:
◦ ◦
∀π1 , π2 ∈ P : π 1 ∩ π 2 = ∅ =⇒ µ (π1 ∪ π2 ) = µ (π1 ) + µ (π2 ) (1.12)

2. Invarianza per conguenza:

π1 , π2 ∈ P : π1 , π2 congruenti =⇒ µ (π1 ) = µ (π2 )

Consideriamo ora un insieme A ⊂ R2 limitato. Se A ∈ / P, si pone il problema


della definizione di µ (A). A tale scopo costruiamo gli insiemi:

Σ1 = {π ∈ P | π ⊂ A} (1.13)
Σ2 = {Π ∈ P | Π ⊃ A}

Osservazione. Se A è privo di punti interni, allora Σ1 = ∅.


1.2 Misura degli insiemi piani 11

Poniamo:

def
α (A) = {µ (π) | π ∈ Σ1 }
def
β (A) = {µ (Π) | Π ∈ Σ2 }

Evidentemente:

(∀π ∈ Σ1 , ∀Π ∈ Σ2 , π ⊆ Π) =⇒ µ (π) ≤ µ (Π) ,


cioè α (A) e β (A) sono insiemi separati.

Definizione. Si definisce misura interna di A il numero reale non negativo

µi (A) = sup α (A)

Definizione. Si definisce misura esterna di A il numero reale non negativo

µe (A) = inf β (A)

Risulta:

µi (A) ≤ µe (A)

Osservazione. Se A è privo di punti interni:


def
Σ1 = ∅ =⇒ α (A) = ∅ =⇒ µi (A) = 0

Definizione. L’insieme A è misurabile (secondo Peano - Jordan) se risulta


µi (A) = µe (A). In tal caso si pone:
def
µ (A) = µi (A) = µe (A) ,

essendo µ (A) la misura di A.

Osservazione. Se A è misurabile ed è privo di punti interni, segue necessaria-


mente che µ (A) = 0, poiché è µi (A) = 0.
Dalla definizione di misurabilità segue che A è misurabile se e solo se gli insiemi
α (A) e β (A) sono contigui:

∀ε > 0, ∃πε ∈ Σ1 , ∃Πε ∈ Σ2 | µ (Πε ) − µ (πε ) < ε (1.14)

Nel caso speciale in cui A è misurabile e privo di punti interni, la (1.14) si scrive:

∀ε > 0, ∃Πε ∈ Σ2 | µ (Πε ) < ε


1.2 Misura degli insiemi piani 12

Resta così definita la misura µ degli insiemi limitati A ⊂ R2 non appartenenti a


P:

µ : M → R+
0 (1.15)
essendo:

M = A ⊂ R2 | A è limitato e misurabile (1.16)

Teorema 1. P ⊂ M

Dimostrazione. Osserviamo che i poligoni sono particolari insiemi limitati e mis-


urabili, donde:
∀π ∈ P, π ∈ M (1.17)
Consideriamo ora un qualunque cerchio di centro (x0 , y0 ) e raggio r:

γ = (x, y) ∈ R2 : (x − x0 )2 + (y − y0 )2 ≤ r2

Abbiamo: µ (γ) = πr2 ; cioè γ è limitato e misurabile =⇒ γ ∈ M. Ma γ ∈


/ P,
donde (tenendo conto della (1.15)) l’asserto.

***

La misura (1.15) conserva le proprietà (additività e congruenza) della misura


(1.11). Rispetto a quest’ultima possiede la proprietà di monotonia:

∀A, B ∈ M | A ⊆ B =⇒ µ (A) ≤ µ (B)


Altre proprietà.
Se A, B ∈ M

A ∪ B ∈ M, A − B ∈ M, A ∩ B ∈ M (1.18)

A ∩ B = ∅ =⇒ µ (A + B) = µ (A) + µ (B)
A ⊆ B =⇒ µ (A − B) = µ (A) − µ (B)
∀A, B ∈ M, µ (A ∪ B) = µ (A) + µ (B) − µ (A ∩ B)

Dall’ultima delle (1.18) segue:

A, B ∈ M, µ (A ∪ B) ≤ µ (A) + µ (B) (1.19)


Ciò si esprime dicendo che la misura (1.15) è subadditiva.

***
1.2 Misura degli insiemi piani 13

La nozione di misura di un insieme si estende facilmente al caso di insiemi non


limitati. A tale scopo sia A ⊂ R2 un insieme non limitato.
Definizione.
def
A è misurabile ⇐⇒ ∀X ∈ M, A ∩ X ∈ M
In tal caso, la misura di A è:

µ (A) = sup µ (A ∩ X) ≤ +∞
X∈M

Precisamente:

µ (A) < +∞ =⇒ A è di misura finita


µ (A) = +∞ =⇒ A è di misura infinita

Indichiamo con R la classe dei rettangoli Θ (α, β) di centro l’origine e dimensioni


α, β:

Θ (α, β) = (x, y) ∈ R2 | |x| ≤ α, |y| ≤ β
Ciò premesso, sussiste il seguente
Teorema 2.
(A è misurabile) ⇐⇒ (∀Θ ∈ R, A ∩ Θ ∈ M)
Se A è misurabile:
µ (A) = sup µ (A ∩ Θ)
Θ∈R

Dimostrazione. Implicazione diretta Abbiamo:

(A è misurabile) =⇒ (∀X ∈ M, A ∩ X ∈ M) =⇒ (∀Θ ∈ R, Θ ∩ A ∈ M)


R⊂M

Implicazione inversa L’ipotesi è:

∀Θ ∈ R, A ∩ Θ ∈ M

Preso ad arbitrio X ∈ M scegliamo Θ ∈ R tale che Θ ⊃ X, donde:

A ∩ X = (A ∩ Θ) ∩ X

Quindi
A ∩ Θ ∈ M =⇒ A ∩ X ∈ M
cioè la tesi. Inoltre, nelle medesime ipotesi:

sup µ (A ∩ Θ) ≤ µ (A) ≤ µ (A ∩ Θ) =⇒ µ (A) = sup µ (A ∩ Θ)


Θ∈R Θ∈R
1.3 Area del rettangoloide 14

1.3 Area del rettangoloide


Premettiamo la seguente
Definizione. Sia A 6= ∅. Gli insiemi non vuoti A1 , A2 , ..., An costituiscono una
partizione di A se:
n
[
Ak = A
k=1
◦\◦
Ak Ak′ = ∅, per k, k ′ ∈ {1, 2, ..., n} con k 6= k ′

Sia f (x) continua in [a, b] ⊂ R e ivi non negativa. Eseguiamo una partizione
dell’intervallo [a, b] attraverso n + 1 punti:

x0 , x1 , ...., xn ∈ [a, b]
Precisamente:

a = x0 < x1 < x2 < ... < xn−1 < xn = b


Si tratta di una partizione, poiché:

n−1
[
[xk , xk+1 ] = [a, x1 ] ∪ [x1 , x2 ] ∪ ... ∪ [xn−1 , b] = [a, b]
k=0
∀k, k ′ ∈ {0, 1, ..., n − 1} con k 6= k ′ , (xk , xk+1 ) ∩ (xk′ , xk′ +1 ) = ∅
Indichiamo tale partizione con il simbolo convenzionale D (x0 , x1 , ..., xn ). Poni-
amo per definizione:

δ = max (xk+1 − xk ) , N = {0, 1, 2, ..., n − 1}


k∈N

Il numero reale δ > 0 si chiama ampiezza della partizione. Inoltre, se f (x) non
è identicamente nulla consideriamo il minimo e il massimo di f (x) in [xk , xk+1 ]:

mk = min f (x)
[xk ,xk+1 ]

Mk = max f (x)
[xk ,xk+1 ]

Quindi:


def
rk = (x, y) ∈ R2 | xk ≤ x ≤ xk+1 , 0 ≤ y ≤ mk (1.20)
def 
Rk = (x, y) ∈ R2 | xk ≤ x ≤ xk+1 , 0 ≤ y ≤ Mk
1.3 Area del rettangoloide 15

Cioè rk è il rettangolo di base [xk , xk+1 ] e altezza mk , mentre Rk è il rettangolo


di base [xk , xk+1 ] e altezza Mk . Restano così definiti i seguenti poligoni:

n−1
[
π (D) = rk (1.21)
k=0
n−1
[
Π (D) = Rk
k=0

Definizione. π (D) è il plurirettangolo inscritto a R associato alla partizione


D.
Π (D) è il plurirettangolo circoscritto a R associato alla partizione D.
Evidentemente:

n−1
X n−1
X
def
sD = µ [π (D)] = µ (rk ) = mk (xk+1 − xk ) (1.22)
k=0 k=0
n−1
X n−1
X
def
SD = µ [Π (D)] = µ (Rk ) = Mk (xk+1 − xk )
k=0 k=0

Inoltre:

∀D, D′ , π (D) ⊆ Π (D′ ) =⇒ ∀D, D′ , sD ≤ SD′


Teorema 3. Nelle suddette ipotesi il rettangoloide

R = (x, y) ∈ R2 : a ≤ x ≤ b, 0 ≤ y ≤ f (x)

è misurabile, risultando:

µ (R) = supsD = inf SD


D D

Dimostrazione. Se f (x) è identicamente nulla, l’asserto è banale:



R = (x, y) ∈ R2 : a ≤ x ≤ b, y = 0 =⇒ µ (R) = 0
∀D, sD = SD = 0 =⇒ supsD = inf SD = 0
D D

Consideriamo quindi il caso non banale, cioè f (x) non identicamente nulla in
[a, b]. Per il teorema di Heine-Cantor, la funzione è ivi uniformemente continua,
per cui:
ε
∀ε > 0, ∃δε > 0 : x′ , x′′ ∈ [a, b] , |x′ − x′′ | < δε =⇒ |f (x′ ) − f (x′′ )| <
b−a
1.3 Area del rettangoloide 16

Eseguiamo una partizione D̄ (x0 , x1 , ..., xn ) dell’intervallo [a, b] di ampiezza δ̄ < δε .


Siano
x̄k , x̄′k ∈ [xk , xk+1 ] : f (x̄k ) = mk , f (x̄′k ) = Mk
Quindi:
ε
|x̄′k − x̄k | ≤ xk+1 − xk ≤ δ̄ < δε =⇒ Mk − mk <
b−a
Inoltre:
n−1 n−1
X ε X
SD̄ − sD̄ = (Mk − mk ) (xk+1 − xk ) < (xk+1 − xk ) = ε
k=0
b − a k=0

Ma SD̄ e sD̄ sono le aree di due poligoni:



π D̄ = πε

Π D̄ = Πε

Perciò:
∀ε > 0, ∃Πε , πε : µ (Πε ) − µ (πε ) < ε
ciò implica la misurabilità di R. Infine:

sD̄ ≤ µ (R) , sD̄ > SD − ε,

cioè:
SD̄ − µ (R) < ε (1.23)
Similmente:
µ (R) ≤ SD̄ , SD̄ < sD̄ + ε
per cui:
µ (R) − sD̄ < ε (1.24)
Dalle (1.23)-(1.24) segue:

µ (R) = supsD̄ = inf SD̄


D D

***

Dal teorema appena dimostrato segue che ∀n ∈ N, ∀D (x0 , x1 , ..., xn ):

sD = valore approssimato per difetto di µ (R)


SD = valore approssimato per eccesso di µ (R)
1.3 Area del rettangoloide 17

Scegliere come valore approssimato di µ (R) la somma sD , equivale ad approssi-


mare ∀k ∈ N , il rettangoloide:

(x, y) ∈ R2 | xk ≤ x ≤ xk+1 , 0 ≤ y ≤ f (x) (1.25)
con il rettangolo rk in esso inscritto. Viceversa, scegliere come valore approssi-
mato di µ (R) la somma SD , equivale ad approssimare ∀k ∈ N , il rettangoloide
(1.25) con il rettangolo Rk ad esso circoscritto.
Ora poniamo:
def 
∀k ∈ N , τk = (x, y) ∈ R2 | xk ≤ x ≤ xk+1 , 0 ≤ y ≤ ηk ,
essendo ηk ∈ [mk , Mk ].
Il poligono:
n−1
[
τ (D) = τk
k=0

è un plurirettangolo che non è inscritto ad R e al tempo stesso non è circoscritto


ad R.
def
σD = µ (τ (D)) =⇒ sD ≤ σD ≤ SD
Il numero reale σD è comunque un valore approssimato di µ (R). Inoltre:

(mk ≤ ηk ≤ Mk ) =⇒ (∃ξk ∈ [xk , xk+1 ] | f (ξk ) = ηk )


f (x) è continua
in [xk , xk+1 ]
Quindi:

n−1
X n−1
X
µ (τk ) = f (ξk ) (xk+1 − xk ) =⇒ σD = µ (τk ) = f (ξk ) (xk+1 − xk ) (1.26)
k=0 k=0

Dalla (1.26) segue che σD dipende da ξk per ogni k ∈ N .

∀ξk ∈ [xk , xk+1 ] (con k ∈ N ), sD ≤ σD ≤ SD =⇒ |σD − µ (R)| ≤ SD − sD

Dalla dimostrazione dell’ultimo teorema segue

Teorema 4.

∀ε > 0, ∃δε > 0 | ∀D (δ < δε ) , |σD − µ (R)| < ε (1.27)


1.3 Area del rettangoloide 18

La (1.27) può essere scritta nela forma simbolica:

limσD = µ (R) (1.28)


δ→0

Si osservi che la (1.28) non è l’usuale operazione di passaggio al limite, giacché


σD non una funzione ad un sol valore di δ. Infatti, assegnato un numero reale
positivo δ < b − a, esistono infinite partizioni di ampiezza δ, e per ciascuna
partizione esistono infiniti valori di σD , giacché questi ultimi dipendono dai punti
ξk (che possono essere scelti in infiniti modi). Da ciò si conclude che σD è una
funzione ad infiniti valori di δ. Pertanto, la (1.28) andrebbe riscritta nella forma:

σD −→ µ (R) ,
δ→0

e cioè le somme σD tendono all’area del rettangoloide, quando la loro ampiezza


tende a zero.

***

Sia f (x) continua in [a, b] e ivi non positiva. In tal caso il rettangoloide di base
[a, b], relativo a f (x), si ridefinisce:
def 
R = (x, y) ∈ R2 | a ≤ x ≤ b, f (x) ≤ y ≤ 0
Indichiamo con R′ il rettangoloide di base [a, b], relativo a −f (x). È facile
convincersi che R′ è simmetrico a R rispetto all’asse x.
Per i teoremi precedenti si ha che R′ è misurabile:

µ (R′ ) = limσD

, (1.29)
δ→0

essendo:

n−1
X

σD = [−f (ξk )] (xk+1 − xk ) (1.30)
k=0
n−1
X
=− f (ξk ) (xk+1 − xk )
k=0
= −σD ,

per una generica partizione D di ampiezza δ, e per ogni ξk ∈ [xk , xk+1 ]. Dalle
(1.29)-(1.30) segue

µ (R′ ) = −limσD
δ→0

Dalla misurabilità di R e dalla simmetria tra R′ e R, segue che R è misurabile:



1.4 Definizione di integrale definito 19

µ (R) = µ (R′ ) ,
donde:

limσD = −µ (R) (1.31)


δ→0

1.4 Definizione di integrale definito


Sia f (x) continua in [a, b]. Eseguiamo ad arbitrio una partizione D (x0 , x1 , ..., xn )
di ampiezza δ dell’intervallo [a, b]. Per ogni ξk ∈ [xk , xk+1 ], consideriamo le
somme:
n−1
X
σD = f (ξk ) (xk+1 − xk )
k=0

Posto

λ = limσD , (1.32)
δ→0

risulta:

µ (R) , se f (x) ≥ 0
λ= , (1.33)
−µ (R) , se f (x) ≤ 0
essendo R il rettangoloide di base [a, b] relativo a f (x).

Teorema 5.
∃!λ ∈ R | limσD = λ (1.34)
δ→0

Dimostrazione. Poniamo:
f (x) + |f (x)| − f (x) − |f (x)|
f + (x) = , f (x) = (1.35)
2 2
Le funzioni continue f ± (x) sono rispettivamente la parte non negativa (+) e
la parte non positiva (−) di f (x). Evidentemente:

f (x) = f + (x) + f − (x) (1.36)

Se Γ+ )y = f + (x):

x ∈ [a, b] : f (x) > 0 =⇒ f + (x) = f (x)


x ∈ [a, b] : f (x) < 0 =⇒ f + (x) = 0
1.4 Definizione di integrale definito 20

Cioè Γ+ è composto dalle parti del grafico Γ)y = f (x) di ordinata positiva, e dai
punti dell’asse x le cui ordinate sono negative. Similmente Γ− )y = f − (x)

x ∈ [a, b] : f (x) < 0 =⇒ f − (x) = f (x)


x ∈ [a, b] : f (x) > 0 =⇒ f + (x) = 0

∀D, ∀ξk
n−1
X
σD = f (ξk ) (xk+1 − xk )
k=0
n−1
X
±
σD = f ± (ξk ) (xk+1 − xk )
k=0

Dalle (1.36):
+ −
σD = σD + σD
Inoltre:
+ −
limσD = µ (R+ ) , limσD = −µ (R− ) ,
δ→0 δ→0

essendo R± il rettangoloide di base [a, b], relativo a f ± (x). Per una nota proprietà
del valore assoluto:
+ −
|σD − [µ (R+ ) − µ (R− )]| ≤ σD − µ (R+ ) + σD − µ (R− ) = ε
| {z } | {z }
<ε/2 <ε/2

Posto
λ = µ (R+ ) − µ (R− ) , (1.37)
segue:
|σD − λ| ≤ ε =⇒ limσD = λ
δ→0

La (1.37) implica esistenza e unicità di λ.

***

Per il teorema appena dimostrato, assegnata una funzione f (x) continua in [a, b],
esiste ed è unico il limite λ dato dall’equazione (1.32). Tale numero reale si chiama
integrale della funzione f (x) esteso all’intervallo [a, b]:

Zb
λ= f (x) dx (1.38)
a

Cioè:
1.4 Definizione di integrale definito 21

n−1 Z b
X
lim f (ξk ) (xk+1 − xk ) = f (x) dx (1.39)
δ→0
k=0 a

L’interpretazione geometrica dell’integrale (1.39) è:

Zb
f (x) dx = µ (R) , se f (x) ≥ 0 (1.40)
a
Zb
f (x) dx = −µ (R) , se f (x) ≤ 0
a
Zb
f (x) dx = µ (R+ ) − µ (R− ) , altrimenti
a

Se f (x) è una funzione definita nell’intervallo X ed è ivi continua, per ogni


a, b ∈ X con a < b, risulta definito l’oggetto:

Zb
f (x) dx (1.41)
a

Poniamo quindi per definizione:

Za Zb
def
f (x) dx = − f (x) dx (1.42)
b a

Da ciò segue che (1.41) ha senso per ogni coppia di punti x′ , x′′ ∈ X:

Zx′′
f (x) dx (1.43)
x′

Il numero reale (1.43) si chiama integrale definito della funzione f (x) esteso
all’intervallo orientato di estremi x′ e x′′ .
Seguono le denominazioni:
1.5 Proprietà dell’integrale definito 22

Z
segno di integrale

x′ , x′′ limiti di integrazione


x′ limite inferiore
x′′ limite superiore
f (x) funzione integranda

Osservazione. Il numero reale (1.41) dipende da a, b e da f (x), ma non dalla


variabile x. Ciò si esprime dicendo che x è una variabile muta, donde possiamo
scrivere:
Zx′′ Zx′′ Zx′′ Zx′′ Zx′′
f (x) dx = f (y) dy = f (t) dt = f (ξ) dξ = f (η) dη = ... (1.44)
x′ x′ x′ x′ x′

1.5 Proprietà dell’integrale definito


Sia f (x) funzione continua nell’intervallo X e ivi non negativa. Sussistono le
seguenti proprietà di cui omettiamo le dimostrazioni:

∀a, b ∈ X,
Zb
f (x) dx = 0 ⇐⇒ f (x) ≡ 0
a
Zb
a < b =⇒ f (x) dx ≥ 0
a
Zb
a > b =⇒ f (x) dx ≤ 0
a
Zb′ Zb
′ ′
a < b, ∀ [a , b ] ⊂ [a, b] , f (x) dx ≤ f (x) dx
a′ a

Qualunque sia il segno di f (x):


b b
Z Z

∀a, b ∈ X, f (x) dx ≤ |f (x)| dx

a a
1.6 Teorema della media 23

1.5.1 Proprietà additiva

Zb Zc Zb
∀a, b, c ∈ X, f (x) dx = f (x) dx + f (x) dx (1.45)
a a c

1.5.2 Proprietà distributiva


f1 (x), f2 (x) , ..., fn (x) funzioni continue in X:

∀c1 , c2 , ...cn ∈ R, (1.46)


Zb
[c1 f1 (x) + c2 f2 (x) + ... + cn fn (x)] dx
a
Zb Zb Zb
= c1 f1 (x) dx + c2 f2 (x) dx + ... + cn fn (x) dx
a a a

Nel caso particolare: fk (x) ≡ 0, per ogni k ∈ {2, 3, ...n}, la (1.46) si scrive:

Zb Zb
c1 f1 (x) dx = c1 f1 (x) dx,
a a

che in generale si riscrive:

Zb Zb
cf (x) dx = c f (x) dx
a a

Ciò si esprime dicendo che ogni costante moltiplicativa può essere portata fuori
dal segno di integrale.

1.6 Teorema della media


Teorema della media.
Zb
f (x) dx
a
f (x) è continua in [a, b] =⇒ ∃ξ ∈ [a, b] | f (ξ) = (1.47)
b−a
1.6 Teorema della media 24

Dimostrazione. Poniamo:
Zb
f (x) dx
a
µ=
b−a
Eseguiamo quindi una partizione D (x0 , x1 , ..., xn ) di [a, b] con ampiezza δ. Presi
ad arbitrio i punti ξk ∈ [xk , xk+1 ], costruiamo la somma σD :
n−1
X
σD = f (ξk ) (xk+1 − xk ) (1.48)
k=0

La (1.48) verifica la doppia disuguaglianza:

m (b − a) ≤ σD ≤ M (b − a) , (1.49)

essendo m = minf (x), M = maxf (x). Eseguendo nella (1.49) l’operazione di


[a,b] [a,b]
passaggio al limite per δ → 0:

Zb
m (b − a) ≤ f (x) dx ≤ M (b − a) ⇐⇒ m ≤ µ ≤ M =⇒
a
f (x) è continua
in [a, b]
=⇒ ∃ξ ∈ [a, b] : f (ξ) = µ,

da cui l’asserto.

Definizione. Il numero reale µ si chiama media integrale.

Interpretazione geometrica.
Eseguiamo una equipartizione di [a, b], cioè una partizione Dn attraverso n
punti equidistanti. Ad esempio:

k (b − a)
xk = a + , k ∈ N = {0, 1, ..., n − 1}
n
Abbiamo:
b−a b−a
xk+1 − xk = =⇒ δn = max (xk+1 − xk ) =
n k∈N n
Determiniamo una somma σDn assumendo ξk = xk+1 :
n−1 n
X b − aX
σDn = f (xk+1 ) (xk+1 − xk ) = f (xk )
k=0
n k=1
Passiamo dalle somme all’integrale:
1.7 Teorema fondamentale del calcolo integrale 25

Zb
f (x) dx = lim σDn
δn →0
a
= lim σDn
n→+∞
n
X
f (xk )
k=1
= (b − a) lim
n→+∞ n
Da cui:
n
X
f (xk )
k=1 f (x1 ) + f (x2 ) + ... + f (xn )
µ = lim = lim
n→+∞ n n→+∞ n
[f (x1 ) + f (x2 ) + ... + f (xn )] /n è la media aritmetica dei valori assunti da f (x)
in n punti equidistanti di [a, b], e µ si presenta come il limite di tale media per
n → +∞. Da qui la denominazione di media integrale.

1.7 Teorema fondamentale del calcolo integrale


Siamo ora in grado di risolvere il problema della ricerca della primitiva di una
funzione continua f (x) in un intervallo X.
Assegnato un punto x0 ∈ X, consideriamo:
Zx
∀x ∈ X, F (x) = f (ξ) dξ (1.50)
x0

La funzione F (x) dicesi funzione integrale della funzione f (x) di punto


iniziale x0 .

Proposizione (Teorema fondamentale del calcolo integrale) 6. La fun-


zione integrale (1.50) è derivabile in X, risultando:

∀x ∈ X, F ′ (x) = f (x) ,

cioè F (x) è una primitiva di f (x).


1.7 Teorema fondamentale del calcolo integrale 26

Dimostrazione.
x+∆x
Z Zx
f (ξ) dξ − f (ξ) dξ
F (x + ∆x) − F (x) x0 x0
=
∆x ∆x
x+∆x
Z Zx0
f (ξ) dξ + f (ξ) dξ
x0 x
=
∆x
x+∆x
Z
f (ξ) dξ
x
=
∆x
Per il teorema della media:
x+∆x
Z
f (ξ) dξ
x F (x + ∆x) − F (x)
∃θ ∈ [0, 1] : f (x + θ∆x) = =
∆x ∆x
La continuità di f (x) implica:

F (x + ∆x) − F (x)
lim = f (x) ,
∆x→0 ∆x
donde l’asserto.

***

Per l’equazione (1.3) la famiglia delle primitive di una funzione f (x) continua
nell’intervallo X, è:
Zx
G (x) = f (ξ) dξ + c, con c ∈ (−∞, +∞) (1.51)
x0

In particolare, nella (1.51) possiamo porre c = G (x0 ), onde:


Zx
G (x) = G (x0 ) + f (ξ) dξ (1.52)
x0

La differenza tra la (1.51) e la (1.52) è evidente: mentre nella (1.51) la primitiva


è indeterminata, in quanto definita a meno di una costante additiva, nella (1.52)
1.7 Teorema fondamentale del calcolo integrale 27

essa è univocamente determinata. Inoltre, ponendo nella (1.52) x0 = a, x = b, si


ottiene:

Zb
f (x) dx = G (b) − G (a) (1.53)
a

La (1.52) è la formula fondamentale del calcolo integrale, poiché fornisce


l’integrale definito della funzione f (x) tra a e b, attraverso la differenza dei valori
assunti in a e in b da una qualunque primitiva.
La (1.52) viene spesso scritta con la notazione simbolica:

Zb
f (x) dx = G (x)|ba (1.54)
a
Capitolo 2

Integrale indefinito

2.1 Definizione di integrale indefinito


Definizione. Dicesi integrale indefinito, la totalità dele primitive di una as-
segnata funzione f (x) continua in un intervallo X, e si indica con il simbolo:
Z
f (x) dx (2.1)

Una qualunque primitiva di f (x) è una determinazione dell’integrale indefinito


(2.1).
Se F (x) è una primitiva, in forza della (1.3) si ha:
Z
f (x) dx = F (x) + c, con c ∈ (−∞, +∞) (2.2)

Il numero reale c si chiama costante di integrazione.


Osservazione. L’integrale definito
Zb
f (x) dx,
a

è un numero reale. Viceversa, l’integrale indefinito:


Z
f (x) dx,

è un insieme di funzioni.
L’integrale indefinito rappresenta l’operazione inversa della derivazione. Più
precisamente l’operazione di derivazione è definita da:

Df (x) = f ′ (x) , (2.3)


2.2 Integrali indefiniti fondamentali 29

essendo D l’operatore di derivazione e f (x) una qualunque funzione derivabile.


L’integrazione indefinita esegue l’operazione inversa della (2.3), giacché:
Z
D f (x) dx = f (x)

Si osservi che il risultato dell’applicazione dell’operatore di derivazione su una


qualunque funzione dotata di espressione elementare, è a sua volta una funzione
dotata di espressione elementare. Ci si può chiedere se tale circostanza si verifica
per l’operatore di integrazione indefinita. La risposta è negativa, nel senso che
esistono funzioni le cui primitive non sono dotate di espressioni elementari.

2.2 Integrali indefiniti fondamentali


Introdotta la nozione di integrale indefinito, si pone il problema della ricerca delle
funzioni primitive di una assegnata funzione dotata di espressione elementare
(quando ciò è possibile, secondo quanto esposto nella sezione precedente). Il
punto di partenza per la soluzione di tale problema è fornito dalla tabella degli
integrali indefiniti fondamentali:

Z Z
1) n
x dx = 1
n+1
xn+1 + C, per n 6= −1 10) sin xdx = − cos x + C
Z Z
2) dx
x
= ln |x| + C 11) cos xdx = sin x + C
Z Z

3) dx
x2 +a2
= 1
a
arctan xa + C, a 6= 0 12) dx
cos2 x
= tan x + C
Z Z

4) dx
x2 −a2
= 2a1
ln x−a
x+a
+ C, a 6= 0 13) dx
sin2 x
= − cot x + C
Z Z
a+x 
5) 2
dx
a −x 2 = 1
2a
ln
a−x
+ C, a 6= 0 14) dx
sin x
= ln tan x2 + C
Z Z
√ 
6) √ dx
2
x ±a 2 = ln x + x2 ± a2 + C, (a 6= 0) 15) dx
cos x
= ln tan x + π
2 4
Z Z

7) √ dx
a2 −x2
= arcsin xa + C, (a > 0) 16) sinh dx = cosh x + C
Z Z
ax
8) ax dx = ln a
+ C, (a > 0) 17) cosh dx = sinh x + C
 Z
dx
Z 
 cosh2 x
= tanh x + C
9) ex dx = ex + C 18) Z
 dx

sinh2 x
= coth x + C

Una coppia di integrali notevoli è la 14)-15) che può essere riscritta nella forma:
2.2 Integrali indefiniti fondamentali 30

Z
dx 1
= ln
− cot x + C (2.4)
sin x sin x
Z
dx 1
= ln + tan x + C
cos x cos x
Parte II

Esercizi sull’integrazione di una


funzione reale di variabile reale
Capitolo 3

Integrali indefiniti

3.1 Integrali indefiniti fondamentali


Calcolare i seguenti integrali, utilizzando la tabella degli integrali indefiniti fon-
damentali:
Z Z Z
1) x dx
3
7) 2 dxx
13) √5xdx2 −5
Z Z Z

2) xdx 8) 3 sin xdx 14) 5+5x dx
2
Z Z Z
3) √dxx 9) 2 sinh xdx 15) x3dx 2 −1
Z Z Z √

4) x xdx 10) x3 dx 16) √ax2a+a dx (a > 0)
Z Z Z
5) 5
4 3 dx
√ 11) √ 3
2 dx 17) 5a2 x6 dx
Z x Z 4−4x Z

6) e dx 12) √x2 −1
x/2 −3dx
18) 2pxdx

3.2 Soluzioni
Z
1. x3 dx = 41 x4 + C
Z
√ 1
x 2 +1
2. xdx = 1
+1
+ C = 32 x3/2 + C
2

Z 1
x− 2 +1 √
3. dx

x
= − 21 +1
+C =2 x+C
Z Z

4. x xdx = x3/2 dx = 25 x5/2 + C
3.2 Soluzioni 33

Z

5. √5
4 3 dx
x
= 20 √x
4 3 + C = 20
x
4
x+C
Z
1 1
6. ex/2 dx = 2e 2 x + C = 2e 2 x + C
Z
7. 2x dx = 1 x
ln 2
2 +C
Z Z
8. 3 sin xdx = 3 sin xdx = −3 cos x + C
Z Z
9. 2 sinh xdx = 2 sinh xdx = 2 cosh x + C
Z Z
10. 3
x
dx =3 dx
x
= 3 ln x + C
Z Z
11. √ 3
4−4x2
dx = 3
2
√dx
1−x
= 23 arcsin x + C
Z
p
12. √−3dx

x2 −1
= −3 ln x + (−1 + x2 ) + C
Z Z

13. √ dx
5x2 −5
= √1
5
√ dx
x2 −1
= √1
5
ln x + x2 − 1 + C
Z
14. dx
5+5x2
= 15 arctan x + C
Z

15. 3dx
x2 −1
= 32 ln x−1
x+1
+C

Z Z

a

16. √
ax2 +a
dx = √ 1
x2 +1
dx = ln x2 + x2 + 1 + C
Z Z
17. 2 6
5a x dx == 5a 2
x6 dx = 57 a2 x7 + C
Z Z
√ √ √
18. 2pxdx = 2px x1/2 dx = 2
3
2px3/2 + C = 32 x 2px + C

Altri esercizi:
Z Z Z
1) dx

nx 3) dx
x2 +7
5) √ dx
4+x2
Z Z Z
1−n
2) (nx) n dx 4) dx
x2 −10
6) √ dx
8−x2
3.3 Integrali di somme di funzioni 34

3.2.1 Soluzioni
Z √
1
1. dx n n n

nx = n−1
x1− n +C = n−1
xn−1 + C
Z Z
1−n 1−n 1−n 1−n
1−n
x n +1 √
2. (nx) n dx = n n x n dx = n n 1−n
+1
+ C = n1/n x1/n = n
nx
n

Z
3. dx
x2 +7
= √1
7
arctan √x7 + C
Z √
4. √ dx √1
x−√10
8−x2
= 10
ln x+ 10
+C
Z
√ 
5. √ dx
4+x2
= ln x + x2 + 4 + C
Z
6. √ dx
8−x2
= arcsin √x8 + C

3.3 Integrali di somme di funzioni


Calcolare i seguenti integrali, utilizzando la proprietà additiva:

Z Z   Z
1) (sin x + cos x) dx 7) √ 3
1−x2
− 2
1+x2
dx 13) cos 2x
cos x+sin x
dx
Z √  Z Z
2) 1
8) 2x4 −3x2 +5x
14) (ax + b)3 dx
3
x2 − √ 3 2
x
dx x2
dx
Z Z Z
3) 3x5 −2x3 +1
x
dx 9) cos 2x
sin x−cos x
dx 15) (2x + 1)4 dx
Z Z Z √ √
3x5 −2x3 +1
 2 2
4) x3
dx 10) 2 sin x + dx 16) √ − 2−x dx
sin 2x
2+x
sin x4−x4
Z Z Z
 
5) 3
cos2 x
+ sin22 x dx 11) e3x + x5 dx 17) (6x2 + 8x + 3) dx
Z Z   Z
6) sin 2x+2 cos x
cos x
dx 12) − √1−x2 + cos2 x dx 18) x (x + a) (x + b) dx
2 1

3.3.1 Soluzioni
Z Z Z
1. (sin x + cos x) dx = sin xdx+ cos xdx = (− cos x + C1 )+(sin x + C2 )

= sin x − cos x + C
Z √  Z √ Z
2. 1 1
3 2 3 2
x − √
3 2
x
dx = x dx − √ 3 2 dx
x
2 +1  − 2 +1
 √ √
= x23+1 + C1 − x− 23+1 + C2 = 35 x x2 − 3 3 x + C
3

3 3
3.3 Integrali di somme di funzioni 35

Z Z Z Z
3x5 −2x3 +1
3. x
dx 4 2
= 3 x dx − 2 x dx + dx
x
= 53 x5 − 23 x3 + ln |x| + C
Z Z Z Z
3x5 −2x3 +1
4. x3
dx = 3 x dx − 2 dx + x−3 dx = x3 − 2x −
2 1
2x2
+C
Z Z Z

5. 3
cos2 x
+ 2
sin2 x
dx = 3 dx
cos2 x
+2 dx
sin2 x
= 3 (tan x + C1 )+2 (− cot x + C2 )

= 3 tan x − 2 cot x + C
Z Z Z
6. sin 2x+2 cos x
cos x
dx = sin 2x
cos x
dx + 2 dx = 2 (− cos x + C1 ) + 2 (x + C2 ) =
−2 (cos x − x) + C
Z   Z Z
7. √ 3
1−x2
2
− 1+x2 dx = 3 √ dx
1−x2
−2 dx
1+x2
= 3 arcsin x − 2 arctan x + C
Z Z Z Z
2x4 −3x2 +5x
8. x2
dx 2
= 2 x dx − 3 dx + 5 dx
x

1 3

=2 3
x+ C1 − 3 (x + C2 ) + 5 (ln |x| + C3 ) = 32 x3 − x + 5 ln |x| + C
Z Z
2 x−sin2 x
9. cos 2x
sin x−cos x
dx = cos
sin x−cos x
dx
Z Z
= − sin xdx − cos xdx = cos x − sin x + C
Z Z Z

10. 2 sin x + sin 2x
sin x
dx = 2 sin xdx + 2 cos xdx = −2 cos x + 2 sin x

= 2 (sin x − cos x) + C
Z Z Z

11. 3x 5 3x
e + x dx = e dx + 5 dx
x
= 13 e3x + C1 + 5 ln |x| + C2

= 31 e3x + 5 ln x + C
Z   Z Z
12. 2 1
− √1−x2 + cos2 x dx = −2 √ dx
1−x2
+ dx
cos2 x
= 2 arccos x + tan x + C
Z Z Z Z
cos2 x−sin2 x
13. cos 2x
cos x+sin x
dx = cos x+sin x
dx = cos xdx− sin xdx = cos x+sin x+C
Z Z
3
14. (ax + b) dx = (a3 x3 + 3a2 bx2 + 3b2 ax + b3 ) dx

= 41 a3 x4 + a2 bx3 + 32 b2 ax2 + b3 x + C
Z Z
4
15. (2x + 1) dx = (16x4 + 32x3 + 24x2 + 8x + 1) dx
16 5
= 5
x + 8x4 + 8x3 + 4x2 + x + C
3.4 Integrali di una f (ξ (x)), con ξ (x) funzione lineare 36

Z √ √
Z √
Z √
2 2 2+x2 2−x2
16. √ − 2−x dx
2+x
4−x4
= √ dx − √ dx
(2−x2 )(2+x2 ) (2−x2 )(2+x2 )
Z Z
√ 
= √ dx − √ dx = arcsin √x2 − ln x + x2 + 2 + C
2−x2 2+x2
Z
17. (6x2 + 8x + 3) dx = 2x3 + 4x2 + 3x + C
Z Z Z Z
18. x (x + a) (x + b) dx = 3
x dx + (a + b) 2
x dx + ab xdx

= 41 x4 + 13 (a + b) x3 + 21 abx2

3.4 Integrali di una f (ξ (x)), con ξ (x) funzione lin-


eare
Calcolare gli integrali:
Z Z Z
1) sin (ax + b) dx 5) dx
(ax+b)n
9) 5
3
sinh (5x) dx
Z Z Z
2) (ax + b)n dx 6) (3 − 5x)3 dx 10) √ dx
1−4x2
Z Z Z
3) eax+b dx 7) cos (5x − 2) dx 11) adx
a−x
Z Z
4) cos (ax + b) dx 8) 3e−2x+5 dx

3.4.1 Soluzioni
Z
1. I (x) = sin (ax + b) dx; poniamo

ξ = ax + b =⇒ dξ = adx, (3.1)

donde: Z
1 1
I (ξ) = sin ξdξ = − cos ξ + C,
a a
Ripristinando la variabile x:
1
I (x) = − cos (ax + b) + C
a
Z
2. I (x) = (ax + b)n dx; eseguendo il cambio di varibile (3.1):
Z
1 1
I (ξ) = ξ n dξ = ξ n+1 + C,
a a (n + 1)
3.4 Integrali di una f (ξ (x)), con ξ (x) funzione lineare 37

Ripristinando la variabile x:
(ax + b)n+1
I (x) = + C, per n 6= −1
a (n + 1)
Z
3. I (x) = eax+b dx; eseguendo il cambio di varibile (3.1):
Z
1 1
I (ξ) = eξ dξ = eξ ,
a a

Ripristinando la variabile x:
1
I (x) = eax+b + C
a
Z
4. I (x) = cos (ax + b) dx; eseguendo il cambio di varibile (3.1):
Z
1 1
I (ξ) = cos ξdξ = sin ξ,
a a

Ripristinando la variabile x:
1
I (x) = +C
a sin (ax + b)
Z
5. I (x) = dx
(ax+b)n
; eseguendo il cambio di varibile (3.1):
Z
1 dξ 1 1 1−n
I (ξ) = n
= ξ + C,
a ξ a1−n

Ripristinando la variabile x:
(ax + b)1−n
I (x) = + C, per n 6= 1
a (1 − n)
Z
6. I (x) = (3 − 5x)3 dx; eseguendo il cambio di varibile (3.1):
Z
1 1 ξ4
I (ξ) = − ξ 3 dξ = − + C,
5 54

Ripristinando la variabile x:
(3 − 5x)4
I (x) = − +C
20
3.4 Integrali di una f (ξ (x)), con ξ (x) funzione lineare 38

Z
7. I (x) = cos (5x − 2) dx; eseguendo il cambio di variabile del tipo (3.1):
Z
1 1
I (ξ) = cos ξdξ = sin ξ + C,
5 5

Ripristinando la variabile x:

sin (5x − 2)
I (x) = +C
5
Z
8. I (x) = 3e−2x+5 dx; eseguendo il cambio di varibile del tipo (3.1):
Z
3 3
I (ξ) = − eξ dξ = − eξ + C,
2 2

Ripristinando la variabile x:
3
I (x) = − e−2x+5 + C
2
Z
9. I (x) = 5
3
sinh (5x) dx; eseguendo il cambio di varibile del tipo (3.1):
Z
1 1
I (ξ) = sinh ξdξ = cosh ξ + C,
3 3

Ripristinando la variabile x:
1
I (x) = cosh (2x) + C
3
Z
10. I (x) = √ dx
1−4x2
; eseguendo il cambio di varibile:

ξ = 2x,

segue: Z
1 dξ 1
I (ξ) = p = arcsin ξ + C,
2 1 − ξ2 2

Ripristinando la variabile x:
1
I (x) = arcsin (2x) + C
2
3.5 Integrazione per introduzione sotto il segno di integrale 39

Z
11. I (x) = adx
a−x
; eseguendo il cambio di varibile:

ξ = a − x,

segue: Z

I (ξ) = −a = −a ln |ξ| + C1 ,
ξ
Ripristinando la variabile x:

I (x) = −a ln |a − x| + C1

Poniamo:
C1 = a ln |C| ,

da cui:
C
I (x) = a ln

a − x

3.5 Integrazione per introduzione sotto il segno di


integrale
Esempio 1:
Z Z
xdx xdx
√ = q
1 + x4 1 + (x2 )2
Poniamo

ξ = x2 ,
donde:

Z Z
1 xdx 1 dξ 1  p 
xdx = dξ =⇒ q = p = ln ξ + 1 + ξ 2 + C
2 2 1 + ξ2 2
1 + (x2 )2

Quindi:
Z
xdx 1  2 √ 
√ 2
= ln x + 1 + x + C
1 + x4 2
Esempio 2
3.5 Integrazione per introduzione sotto il segno di integrale 40

Z
n
In (x) = xn−1 ex dx
Poniamo:

ξ = xn =⇒ xn−1 dx = ,
n
donde:
Z
1 1 ξn 1 n
In (ξ) = eξ dξ = e + C =⇒ In (x) = ex + C
n n n
***

3.5.1 Calcolare gli integrali:


Z Z √
Z √
x+ln x
1) xdx
cos2 x2
11) x
dx 21) x− arctan 2x
1+4x2
dx
Z Z Z
e−(x ) dx
2 +1
2) tan xdx 12) 2x−5
3x2 −2
dx 22)
Z Z Z
2
3) cot xdx 13) 3−2x
5x2 +7
dx 23) x7x dx
Z Z Z
√ e1/x
4) tan x √dxx 14) xdx
x2 −5
24) x2
dx
Z Z Z

5) x cot (x + 1) dx 15)
2 xdx
2x2 +3
25) 5 dx
x√
x
Z Z Z
(ax −bx )2
6) 3
sin 6x cos 6xdx 16) ax+b
a2 x2 +b2
dx 26) a x bx
dx
Z Z Z
2x −1
7) cos ax
sin5 ax
dx 17) √ xdx
a4 −x4
27) a√
ax
dx
Z Z Z
x2 dx ex
8) sin 3x
3+cos 3x
dx 18) 1+x6
28) ex −1
dx
Z Z q Z

9) √ sin x cos x dx 19) arcsin x
1−x2
dx 29) ex a − bex dx
Z cos2 x−sin2 x Z Z
arctan x2 1/3 x/a
10) √ x
x2 +1
dx 20) 4+x2
dx 30) ex/a + 1 e dx

3.5.2 Soluzioni
Z
1. I (x) = xdx
cos2 x2
; eseguiamo il cambio di variabile

1
ξ = x2 =⇒ xdx = dξ,
2
quindi:
Z
1 dξ 1 1
I (ξ) = 2
= tan ξ + C =⇒ I (x) = tan x2 + C
2 cos ξ 2 2
3.5 Integrazione per introduzione sotto il segno di integrale 41

Z Z
2. I (x) = tan xdx = sin x
cos x
dx; eseguiamo il cambio di variabile:

ξ = cos x =⇒ dξ = − sin xdx =⇒


Z

=⇒ I (ξ) = − = − ln |ξ| + C =⇒ I (x) = − ln |cos x| + C
ξ
Z Z
3. I (x) = cot xdx = cos x
sin x
dx; eseguiamo il cambio di variabile:

ξ = sin x =⇒ dξ = sin xdx =⇒


Z

=⇒ I (ξ) = = − ln |ξ| + C =⇒ I (x) = ln |sin x| + C
ξ
Z Z
√ √
sin x dx
4. I (x) = tan x √dxx = √ √ ;
cos x x
eseguiamo il cambio di variabile:

√ 1 √
ξ = cos x =⇒ dξ = − √ sin xdx,
2 x

quindi:
Z

I (ξ) = −2 = −2 ln |ξ| + C
ξ

I (x) = −2 ln cos x + C
Z Z
cos(x2 +1)
5. 2
x cot (x + 1) dx = x sin(x2 +1) dx; eseguiamo il cambio di variabile:
 
ξ = sin x2 + 1 =⇒ dξ = 2x cos x2 + 1 dx,

quindi:
Z
1 dξ 1
I (ξ) = = ln |ξ| + C
2 ξ 2
2

I (x) = −2 ln sin x + 1 + C

Z Z
6. I (x) = 3
sin 6x cos 6xdx = 1
6
sin3 6xd (sin 6x) = 1
24
sin4 6x
Z Z


7. I (x) = cos ax
sin5 ax
dx = 1
a ξ
= 1
a
· − 41 ξ −4 + C =⇒ I (x) = − 4a sin1 4 ax + C
3.5 Integrazione per introduzione sotto il segno di integrale 42

Z
8. I (x) = sin 3x
3+cos 3x
dx; eseguiamo il cambio di variabile:

ξ = 3 + cos 3x =⇒ dξ = −3 sin 3xdx

Quindi:
Z
1 dξ 1 1
I (ξ) = − = − ln |ξ| + C =⇒ I (x) = − ln |3 + cos 3x| + C
3 ξ 3 3
Z Z
9. I (x) = √ 2 sin x cos x
2
1
dx = 2 √sin 2x
cos 2x
dx; eseguiamo il cambio di variabile:
cos x−sin x

ξ = cos 2x =⇒ dξ = −2 sin 2xdx

Quindi:
Z
1 dξ 1p 1√
I (ξ) = − √ =− ξ + C =⇒ I (x) = − cos 2x + C
4 ξ 2 2
Z
10. I (x) = √ x
x2 +1
dx; eseguiamo il cambio di variabile:

ξ = x2 + 1 =⇒ dξ = 2xdx

Quindi:
Z
1 dξ 1p 1√ 2
I (ξ) = √ =− ξ + C =⇒ I (x) = x +1+C
2 ξ 2 2
Z √ Z  Z Z
x+ln x
 √
11. I (x) = x
dx = √1
x
ln x
+ x = dx

x
+ lnxx dx = (2 x + C1 ) +
Z
√ 
ln xd (ln x) = 12 4 x + ln2 x + C
Z Z Z Z

12. I (x) = 2x−5
3x2 −2
dx = 2x
3x2 −2
− 5
3x2 −22
dx = 2 xdx
3x2 −2
− 5 dx
3x2 −2
=
2I1 (x) − 5I2 (x)
Calcolo di I1 (x)
Eseguiamo il cambio di variabile:

ξ = 3x2 − 2 =⇒ dξ = 6xdx

Quindi:
Z
1 dξ 1 1
I1 (ξ) = = ln |ξ| + C1 =⇒ I1 (x) = ln 3x2 − 2 + C1
6 ξ 6 6
3.5 Integrazione per introduzione sotto il segno di integrale 43

Calcolo di I2 (x)
Eseguiamo il cambio di variabile:
√ √
ξ = 3x =⇒ dξ = 3dx

Quindi:
Z
1 dξ
I2 (ξ) = √ √ 2
3 ξ2 − 2


1 1 ξ − 2
= √ · √ · ln √ + C2
3 2 2 ξ + 2
ξ − √2

1
= √ ln √ + C2
2 6 ξ + 2

3x − √2

1
=⇒ I1 (x) = √ ln √ √ + C2
2 6 3x + 2

donde: √
3x − √2

1 2 5
I (x) = ln 3x − 2 − √ ln √ √ +C
3 2 6 3x + 2
Z Z Z
13. I (x) = 5x2 +7 dx = 3 5x2 +7 dx − 2 5x2x+7 dx = 3I1 (x) − 2I2 (x)
3−2x 1

Z Z
Calcolo di I1 (x) = 5x2 +7 dx =
1
√ 2
1
√ 2 dx
( ) +( 7)
5x

Eseguiamo il cambio di variabile:


√ √
ξ = 5x =⇒ dξ = 5dx

Quindi:
Z
1 dξ 1 1 ξ
I1 (ξ) = √ √ 2 = √ · √ arctan √ + C1
5 ξ2 + 7 5 7 7
r !
1 5
=⇒ I1 (x) = √ arctan x + C1
35 7
Z
d(5x2 +7)
Calcolo di I2 (x) = 1
5 5x2 +7
= 1
10
ln |5x2 + 7| + C2 , donde:
r !
3 5 1 2
I (x) = √ arctan x − ln 5x + 7 + C
35 7 5
3.5 Integrazione per introduzione sotto il segno di integrale 44

Z Z
d(x2 −5)
14. I (x) = xdx
x2 −5
= 1
2 x2 −5
= 21 ln |x2 − 5| + C
Z Z
d(2x2 +3)
15. I (x) = xdx
2x2 +3
= 1
4 2x2 +3
= 41 ln (2x2 + 3) + C
Z Z Z
16. I (x) = ax+b
a2 x2 +b2
dx =a xdx
a2 x2 +b2
+b dx
a2 x2 +b2
= aI1 (x) + bI2 (x)
Z
Calcolo di I1 (x) = xdx
a2 x2 +b2

Eseguiamo il cambio di variabile:


ξ = a2 x2 + b2 =⇒ dξ = 2a2 xdx

Quindi:
Z
1dξ 1
I1 (ξ) = 2 = 2 ln |ξ| + C1
2aξ 2a
1
=⇒ I1 (x) = 2 ln a2 x2 + b2 + C1
2a
Z
Calcolo di I2 (x) = dx
a2 x2 +b2

Eseguiamo il cambio di variabile:


ξ = ax =⇒ dξ = adx

Quindi:
Z  
1 dξ 1 1 ξ
I2 (ξ) = 2 2 2
= · arctan + C2
a ξ +b a b b
1 a 
=⇒ I2 (x) = arctan x + C2
ab b
donde: a 
1 2 2 1
2
I (x) = 2 ln a x + b + arctan x + C

2a a b
Z
17. I (x) = √ xdx .
a4 −x4
Eseguiamo il cambio di variabile:

ξ = x2 =⇒ dξ = 2xdx

Quindi:
Z  
1 dξ ξ
I (ξ) = q = arcsin +C
2 a2
(a2 )2 − ξ 2
 
1 x2
=⇒ I (x) = arcsin +C
2 a2
3.5 Integrazione per introduzione sotto il segno di integrale 45

Z Z
x2 dx x2 dx
18. I (x) = 1+x6
= 1+(x3 )2
. Eseguiamo il cambio di variabile:

ξ = x3 =⇒ dξ = 3x2 dx

Quindi:
Z
1 dξ 1
I (ξ) = 2
= arctan ξ + C
3 1+ξ 3
1 
=⇒ I (x) = arctan x3 + C
3
Z q
19. I (x) = arcsin x
1−x2
dx. Eseguiamo il cambio di variabile:

dx
ξ = arcsin x =⇒ dξ = √
1 − x2

Quindi:
Z p
2
I (ξ) = ξdξ = ξ 3/2 + C
3
2
=⇒ I (x) = (arcsin x)3/2 + C
3
Z
arctan x2
20. I (x) = 4+x2
dx. Eseguiamo il cambio di variabile:

x 1 dx dx 1
ξ = arctan =⇒ dξ = 2 =⇒ 2
= dξ
2 21+ x 1 + 4x 2
2

Quindi:
Z
1 1
I (ξ) = ξdξ = ξ 2 + C
2 4
1 x 2
=⇒ I (x) = arctan +C
4 2
Z √ Z Z √
21. I (x) = x− arctan 2x
1+4x2
x
dx = 1+4x2 dx − arctan 2x
1+4x2
dx = I1 (x) − I2 (x)
Z Z
d(1+4x2 )
Calcolo di I1 (x) = 1+4x2 dx = 8
x 1
1+4x2
dx = 81 ln (1 + 4x2 ) + C1
Z √
Calcolo di I2 (x) = arctan 2x
1+4x2
dx
3.5 Integrazione per introduzione sotto il segno di integrale 46

Eseguiamo il cambio di variabile:


2 dx 1
ξ = arctan 2x =⇒ dξ = dx =⇒ = dξ
1 + 4x2 1 + 4x2 2

Quindi:
Z p
1 1
ξdξ = ξ 3/2 + C1
I2 (ξ) =
2 3
1
=⇒ I2 (x) = (arctan 2x)3/2 + C2 ,
3

donde:
1  1
I (x) = ln 1 + 4x2 − (arctan 2x)3/2 + C
8 3
Z
e−(x ) dx. Eseguiamo il cambio di variabile:
2 +1
22. I (x) =

 1
ξ = − x2 + 1 =⇒ dξ = −2xdx =⇒ xdx = − dξ
2

Quindi:
Z
1 1
I (ξ) = eξ dξ = − ξ 2 + C
2 2
1 −(x2 +1)
=⇒ I (x) = − e +C
2
Z Z
2
x2 2 7x
23. I (x) = x7 dx = 1
2
7x d (x2 ) = 2 ln 7
+C
Z
e1/x
24. I (x) = x2
dx. Eseguiamo il cambio di variabile:

1 dx dx
ξ= =⇒ dξ = − 2 =⇒ 2 = −dξ
x x x

Quindi:
Z
I (ξ) = − eξ dξ = −eξ + C
1
=⇒ I (x) = − e1/x + C
2
3.5 Integrazione per introduzione sotto il segno di integrale 47

Z

25. I (x) = 5 dx
x√
x
. Eseguiamo il cambio di variabile:

√ dx dx
ξ= x =⇒ dξ = √ =⇒ √ = 2dξ
2 x x

Quindi:
Z

I (ξ) = − 5ξ dξ = 2 +C
ln 5

5 x
=⇒ I (x) = 2 +C
ln 5
Z Z
(ax −bx )2 ax bx

26. I (x) = x
a b x dx = b x − 2 + a x dx = I1 (x) − 2 (x − C2 ), essendo:
Z Z
ax bx def
I1 (x) = dx + dx = J1 (x) + J2 (x)
bx a x

Qui è: Z Z
ax bx
J1 (x) = dx, J2 (x) = dx
bx ax
Risulta: 
Z  x a x
a b
J1 (x) = dx = + K1
b ln a − ln b
Scambiando a con b:

b x
a
J2 (x) = + K2
ln b − ln a
Quindi:
ax b−x − a−x bx ax b−x − a−x bx
I1 (x) = + C1 =⇒ I (x) = − 2x + C
ln a − ln b ln a − ln b
Z Z   Z
2x −1 a2x 2x− x2

27. I (x) = a√
a x dx = a x/2 − a −x/2
dx = a − a −x/2
dx
Z Z
3
= a dx − a−x/2 dx = I1 (x) − I2 (x) ,
2
x

essendo:
Z   3
2 3
x 3 2 a2x
I1 (x) = a2 d x = + C1
3 2 3 ln a
Z x
− x2
x a− 2
I2 (x) = −2 a d = −2 + C2
2 ln a
3.5 Integrazione per introduzione sotto il segno di integrale 48

Quindi:
2  3x x

I (x) = a 2 + 3a− 2 + C
3 ln a
Z
ex
28. I (x) = ex −1
dx. Eseguiamo il cambio di variabile:

ξ = ex =⇒ ex dx = dξ

Quindi:
Z Z
dξ d (ξ − 1)
I (ξ) = = = ln |ξ − 1| + C
ξ−1 ξ−1
=⇒ I (x) = ln |ex − 1| + C
Z

29. I (x) = ex a − bex dx. Eseguiamo il cambio di variabile:

1
ξ = a − bex =⇒ ex dx = − dξ
b

Quindi:
Z p
1 1 1 3/2
I (ξ) = − ξdξ = − · ξ +C
b b 1 + 12
q
2
=⇒ I (x) = − (a − bex )3 + C
3b
Z
1/3 x/a
30. I (x) = ex/a + 1 e dx. Eseguiamo il cambio di variabile:

1
ξ = ex/a =⇒ dξ = ex/a dx
a

Quindi:
Z
3a
I (ξ) = a (ξ + 1)1/3 dξ = (ξ + 1)4/3 + C
4
3a x/a 4/3
=⇒ I (x) = e +1 +C
4

***
3.5 Integrazione per introduzione sotto il segno di integrale 49

3.5.2.1 Calcolare gli integrali


Z Z Z
ax dx
1) 1+a2x
11) (2 sinh 5x − 3 cosh 5x) dx 21) asin x cos xdx
Z Z Z
e−bx x√n−1 dx
2) 1−e−2bx
dx tanh dx 12) 22) n n
x +1
dx
Z Z Z
ex

3) √1−e 2x dx 13) x n n − x2 dx 23) tan2 axdx
Z Z Z √
2 2 3
4) (cos ax + sin ax) dx 14) xe−x dx 24) 1+ln x
x
dx
Z Z √ Z
2+3x2

5) sin (ln x) x dx
15) 3−2+3x 2 dx 25) tan x − 1 √dx
x−1
Z √ Z Z
earctan x +x ln(1+x2 )+1
6) tan x
cos2 x
dx 16) √dxex 26) 1+x2
dx
Z Z Z
7) x sin (1 − x ) dx 2
17) x+cos
1−sin x
x
dx 27) sin x−cos x
sin x+cos x
dx
Z Z Z
x)2/3
8) (cotsin2 x
dx 18) x ln
dx
2
x
28) x2
x2 −2
dx
Z Z Z
2
9) 1+sin 3x
cos3 3x
dx 19) cos2 x√dx 2−tan2 x
29) esin x
sin 2xdx
Z Z Z

10) b−adx cot 3x
20) 2 + 2x2x+1 2xdx 2 +1 30) √5−3x dx
4−3x2

3.5.3 Soluzioni
Z
ax dx
1. I (x) = 1+a2x
. Eseguiamo il cambio di variabile:

ξ = ax =⇒ 1 + a2x = 1 + ξ 2 ; dξ = axdx

Quindi:
Z

I (ξ) = = arctan ξ + C
1 + ξ2
=⇒ I (x) = arctan (ax ) + C
Z
e−bx
2. I (x) = 1−e−2bx
dx; Eseguiamo il cambio di variabile:

ξ = e−bx =⇒ 1 − e−2bx = 1 − ξ 2 ; dξ = −be−bxdx

Quindi:
Z
1 dξ 1 1 + ξ
I (ξ) = − = − ln +C
b 1 − ξ2 2b 1 − ξ

1 1 + e−bx
=⇒ I (x) = − ln +C
2b 1 − e−bx
3.5 Integrazione per introduzione sotto il segno di integrale 50

Z
x
3. I (x) = √ e
1−e2x
dx; Eseguiamo il cambio di variabile:

ξ = ex =⇒ ex dx = dξ

Quindi:
Z

I (ξ) = p = arcsin ξ + C
1 − ξ2
=⇒ I (x) = arcsin (ex ) + C
Z Z Z
2 
4. I (x) = (cos ax + sin ax) dx = 2 2
cos ax + sin ax + sin 2ax dx = dx+
Z
1 1 1
+ 2a sin 2axd (2ax) = (x + C1 ) + 2a (− cos 2ax + C2 ) = x − 2a cos 2ax + C
Z
5. I (x) = sin (ln x) dx
x
. Eseguiamo il cambio di variabile:

dx
ξ = ln x =⇒ dξ =
x

Quindi:
Z
I (ξ) = sin ξdξ = − cos ξ + C

=⇒ I (x) = − cos (ln x) + C


Z √
6. I (x) = tan x
cos2 x
dx. Eseguiamo il cambio di variabile:

dx
ξ = tan x =⇒ dξ =
cos2 x

Quindi:
Z p
2
I (ξ) = ξdξ = ξ 3/2 + C
3
2 √
=⇒ I (x) = tan3 x + C
3
Z
7. I (x) = x sin (1 − x2 ) dx. Eseguiamo il cambio di variabile:

1
ξ = 1 − x2 =⇒ xdx = − dξ
2
3.5 Integrazione per introduzione sotto il segno di integrale 51

Quindi:
Z
1 1
I (ξ) = − cos ξ + C
sin ξdξ =
2 2
1 
=⇒ I (x) = cos 1 − x2 + C
2
Z
(cot x)2/3
8. I (x) = sin2 x
dx. Eseguiamo il cambio di variabile:

dx
ξ = cot x =⇒ dξ = −
sin2 x

Quindi:
Z
3
I (ξ) = − ξ 2/3 dξ = − ξ 5/3 + C
5
3
=⇒ I (x) = − (cot x)5/3 + C
5
Z Z Z
9. I (x) = 1+sin 3x
cos3 3x
dx = dx
cos3 3x
+ sin 3x
cos3 3x
dx = I1 (x) + I2 (x)
Z
dx 1
I1 (x) = 3
= tan 3x + C1
cos 3x 3
Z
sin 3x
I2 (x) = dx
cos3 3x

Per calcolare I2 (x) eseguiamo il cambio di variabile:

ξ = cos 3x =⇒ dξ = −3 sin 3xdx

Quindi:
Z
1 dξ 11
I2 (ξ) = − = + C2
3 ξ2 3ξ
 
1 1 1
=⇒ I2 (x) = + C2 =⇒ I (x) = tan 3x + +C
3 cos 3x 3 cos 3x
Z
10. I (x) = dx
b−a cot 3x
. Eseguiamo il cambio di variabile:

dx 1
ξ = cot 3x =⇒ 2 = − dξ
sin 3x 3
3.5 Integrazione per introduzione sotto il segno di integrale 52

Quindi:
Z Z
1 d (b − aξ)
dξ 1 1
I (ξ) = − = = ln |b − aξ| + C
3 b − aξ
b − aξ
3a 3a
1
=⇒ I (x) = ln |b − a cot 3x| + C
3a
Z Z Z
11. I (x) = (2 sinh 5x − 3 cosh 5x) dx = 2· 5 sinh 5xd (5x)− 5 cosh 5xd (5x)
1 3

= 25 cosh 5x − 35 sinh 5x + C = 15 (2 cosh 5x − 3 sinh 5x) + C


Z Z Z
12. I (x) = tanh dx = cosh x dx = d(cosh
sinh x x)
cosh x
dx = ln (cosh x) + C
Z

13. In (x) = x n n − x2 dx. Eseguiamo il cambio di variabile:

1
ξ = n − x2 =⇒ dξ = −2xdx =⇒ xdx = − dξ
2

Quindi:
n 1+n
In (ξ) = −
ξ n +C
2 (1 + n)
q
3n
=⇒ In (x) = − (n − x2 )1+n
5
Z Z
−x2 2 2
14. I (x) = xe dx = − 2 e−x d (−x2 ) = − 12 e−x + C
1

Z √
Z Z
3− 2+3x2
15. I (x) = 2+3x2
dx =3 dx
2+3x2
− √ dx
2+3x2
= 3I1 (x) − I2 (x)
Z Z Z √
dx dx d( 3x)
I1 (x) = = √ 2 √ 2 = √1 √ 2 √ 2
2+3x2 ( 2) +( 3x) 3 ( 2) +( 3x)
q 
= √1arctan 3
x
+ C1
6 2
Z Z
dx q √ dx √
I2 (x) = √2+3x 2 = 2 2 =
( 2) +( 3x)
Z √
d( 3x) √ √
= 3 1 √1 ln 3x + 2 + C ,
2 = 2 + 3x
√ q √ 2 √ 2
3
( 2) +( 3x)
donde:
r r ! √
3 3 1 √

I (x) = arctan x − √ ln 3x + 2 + 3x2 + C
2 2 3
3.5 Integrazione per introduzione sotto il segno di integrale 53

Z Z Z
− 21 x 1 
16. I (x) = √dx
ex
= e dx = −2 e− 2 x d − 21 x = 2e−x/2 + C
Z Z
d(x+cos x)
17. I (x) = 1−sin x
x+cos x
dx = x+cos x
= ln |x + cos x| + C
Z
18. I (x) = dx
x ln2 x
. Eseguiamo il cambio di variabile:

dx
ξ = ln x =⇒ dξ =
x
Quindi:
Z
dξ 1
I (ξ) = =− +C
ξ ξ
1
=⇒ I (x) = − +C
ln x
Z
19. I (x) = √dx
cos2 x 2−tan2 x
. Eseguiamo il cambio di variabile:

dx
ξ = tan x =⇒ dξ =
cos2 x
Quindi:
Z  
dξ ξ
I (ξ) = p = arcsin √ +C
2 − ξ2 2
 
tan x
=⇒ I (x) = arcsin √ +C
2
Z Z Z
 dx
20. I (x) = x
2 + 2x2 +1 2x2 +1 = 2 2x2 +1 + (2xxdx
dx
2 +1)2 = 2I1 (x) + I2 (x)

Qui è:
Z
dx
I1 (x) =
2x2 + 1
Z
dx
= √ 2
1+ 2x
Z √ 2
1 d 2x
=√
2 2x2 + 1
1 √ 
= √ arctan 2x + C1 ;
2
Z
d (2x2 + 1) 1 1
I2 (x) = 2 =− 2
+ C2
(2x2 + 1) 4 2x + 1
3.5 Integrazione per introduzione sotto il segno di integrale 54

Quindi:
√ √  1
I (x) = 2 arctan 2x − +C
4 (2x2 + 1)
Z
21. I (x) = asin x cos xdx. Eseguiamo il cambio di variabile:

ξ = cos x =⇒ dξ = cos xdx

Quindi:
Z

I (ξ) = aξ dξ = +C
ln a
asin x
=⇒ I (x) = − +C
ln a
Z
x√n−1 dx
22. I (x) = n n
x +1
dx. Eseguiamo il cambio di variabile:

ξ = xn + 1 =⇒ dξ = nxn−1 dx

Quindi:
Z
1 dξ 1 1 1
1− n
In (ξ) = 1/n
= 1ξ +C
nξ n1− n
1 n 1
=⇒ In (x) = · (xn + 1)1− n + C
n n −q 1
1 n n
= (x + 1)n−1 + C
n−1
Z Z
sin2 ax
23. I (x) = tan axdx = cos
2
2 ax dx =

Z Z Z
1−cos2 ax
= cos2 ax
dx
dx = cos2 ax − dx = a1 tan ax − x
Z √
3
24. I (x) = 1+ln x
x
dx. Eseguiamo il cambio di variabile:

dx
ξ = ln x =⇒ dξ =
x
Quindi:
Z q
33
I (ξ) = (1 + ξ)1/3 dξ = (1 + ξ)4 + C
4
q
33
=⇒ I (x) = (1 + ln x)4 + C
4
3.5 Integrazione per introduzione sotto il segno di integrale 55

Z

25. I (x) = tan x − 1 √dx
x−1
. Eseguiamo il cambio di variabile:

√ dx
ξ= x − 1 =⇒ dξ = √
2 x−1

Quindi:
Z Z
sin ξ d (− cos ξ)
I (ξ) = 2 dξ = −2 = −2 ln |sin ξ| + C
cos ξ cos ξ

=⇒ I (x) = −2 ln sin x − 1 + C
Z
earctan x +x ln(1+x2 )+1
26. I (x) = 1+x2
dx = I1 (x) + I2 (x) + I3 (x), essendo:
Z
earctan x
I1 (x) = dx
1 + x2
Z
x ln (1 + x2 )
I2 (x) = dx
1 + x2
Z
dx
I3 (x) = dx
1 + x2

Calcolo di I1 (x)
Eseguiamo il cambio di variabile:
dx
ξ = arctan x =⇒ = dξ
1 + x2
Quindi:
Z
I1 (ξ) = 2 eξ dξ = eξ + C1

=⇒ I1 (x) = earctan x + C1

Calcolo di I2 (x)
Eseguiamo il cambio di variabile:
 2xdx
ξ = ln 1 + x2 =⇒ dξ =
1 + x2
Quindi:
Z
1 1
I2 (ξ) = ξdξ = ξ 2 + C2
2 4
1 2 
=⇒ I2 (x) = ln 1 + x2 + C2
4
3.5 Integrazione per introduzione sotto il segno di integrale 56

Calcolo di I3 (x)
Z
dx
I3 (x) = = arctan x + C3
1 + x2

Quindi:
1 2 
I (x) = earctan x + ln 1 + x2 + arctan x + C
4
Z Z
d(sin x+cos x)
27. I (x) = sin x−cos x
sin x+cos x
dx =− sin x+cos x
= − ln |sin x + cos x| + C
Z Z Z
x2 x2 −2+2

28. I (x) = x2 −2
dx = x2 −2
dx = 1+ 2
x2 −2
dx
Z Z  √  √ √
ln x−
√2 2
ln x−
dx 1 √2
= dx + 2 x2 −2
=x+2 √
2 2 x+ 2
+C =x+ 2 x+ 2

Z
2
29. I (x) = esin x
sin 2xdx. Eseguiamo il cambio di variabile:

ξ = sin2 x =⇒ dξ = sin 2xdx

Quindi:
Z
I (ξ) = eξ dξ = eξ + C
2
=⇒ I (x) = esin x
+C
Z
30. I (x) = √5−3x dx
4−3x2
= 5I1 (x) − 3I2 (x), essendo:
Z
dx
I1 (x) = √
2
Z 4 − 3x
dx
I2 (x) = √
4 − 3x2

Calcolo di I1 (x)
Z
1 dx
I1 (x) = √ r
3 2
√2 − x2
3
√ !
1 3
= √ arcsin x + C1
3 2
3.5 Integrazione per introduzione sotto il segno di integrale 57

Calcolo di I2 (x)
Eseguiamo il cambio di variabile:
1
ξ = 4 − 3x2 =⇒ xdx = − dξ
6

Quindi:
Z
1 dξ 1p
I2 (ξ) = − √ =− ξ + C2
6 ξ 3
1√
=⇒ I2 (x) = − 4 − 3x2 + C2
3

Da ciò segue:
√ !
5 3 1√
I (x) = √ arcsin x + 4 − 3x2 + C
3 2 3

3.5.4 Calcolare gli integrali:


Z Z Z

1) sin 2π
T
+ φ0 dt 5) sin x cos x

4
dx 9) x2 cosh (x3 + 3) dx
Z Z 2−sin x Z
3tanh x
2) dx
x(4−ln2 x)
6) arcsin
√ x+x dx
1−x2
10) cosh 2 dx
x
Z Z
arccos( x2 )
3) √
4−x2
dx 7) cos 2x
4+cos2 2x
dx
Z Z r √
ln(x+ x2 +1)
4) e− tan x cosdx2 x 8) 1+x2
dx

Z

1. I (t) = sin 2π
T
+ φ0 dt. Eseguiamo il cambio di variabile:

2π 2π T
τ= + φ0 =⇒ dτ = dt =⇒ dt = dτ
T T 2π

Quindi:
Z  
2π T T 2π
I (τ ) = sin τ dτ = − cos τ + C =⇒ I (t) = − cos + φ0 + C
T 2π 2π T
Z
2. I (x) = dx
x(4−ln2 x)
. Eseguiamo il cambio di variabile:

dx
ξ = ln x =⇒ = dξ
x
3.5 Integrazione per introduzione sotto il segno di integrale 58

Quindi:
Z

I (ξ) =
4 − ξ2
Z

=
2 − ξ2
2

1 2 + ξ
= ln +C
4 2 − ξ

1 2 + ln x
I (x) = ln +C
4 2 − ln x
Z
arccos( x2 )
3. I (x) = √
4−x2
dx. Eseguiamo il cambio di variabile:

x dx
ξ = arccos =⇒ √ = −dξ
2 4 − x2

Quindi:
Z
I (ξ) = − ξdξ
1
= − ξ2 + C
2
1h  x i2
I (x) = − arccos +C
2 2
Z
4. I (x) = e− tan x cosdx2 x . Eseguiamo il cambio di variabile:

dx
ξ = − tan x =⇒ dξ = −
cos2 x

Quindi:
Z
I (ξ) = − eξ dξ

= −eξ + C
I (x) = −etan x + C
Z
5. I (x) = sin x cos x

4
dx. Eseguiamo il cambio di variabile:
2−sin x

1
ξ = sin2 x =⇒ sin x cos xdx = dξ
2
3.5 Integrazione per introduzione sotto il segno di integrale 59

Quindi:
Z
1 dξ
I (ξ) = p
2 2 − ξ2
 
1 ξ
= arcsin √ +C
2 2
 2 
1 sin x
I (x) = arcsin √ +C
2 2
Z Z Z
6. I (x) = arcsin
√ x+x dx
1−x2
= arcsin

1−x2
x
dx + √ x
1−x2
dx

Poniamo:
Z Z
def arcsin x def x
I1 (x) = √ dx; I2 (x) = √ dx
1−x 2 1 − x2

Calcolo di I1 (x)
Eseguiamo il cambio di variabile:
dx
ξ = arcsin x =⇒ √ = dξ
1 − x2

Quindi:
Z
1
I1 (ξ) = ξdξ
2
1
= ξ 2 + C1
2
1
I1 (x) = (arcsin x)2 + C1
2

Calcolo di I2 (x)
Eseguiamo il cambio di variabile:
xdx dξ
ξ = 1 − x2 =⇒ √ =−
1−x 2 2

Quindi:
Z
1 dξ
I2 (ξ) = − √
2 ξ
p
= − ξ + C2

I1 (x) = − 1 − x2 + C2
3.5 Integrazione per introduzione sotto il segno di integrale 60

L’integrale I (x) è:
1 √
I (x) = (arcsin x)2 − 1 − x2 + C
2
Z Z
7. I (x) = cos 2x
4+cos2 2x
dx = cos 2x
5−sin2 2x
dx. Eseguiamo il cambio di variabile:


ξ = sin 2x =⇒ cos 2xdx =
2
Quindi:
Z
1 dξ
I (ξ) =
2 5 − ξ2

1 5 + ξ

= √ ln √ +C
4 5 5 − ξ
√ !
1 5 + sin 2x
I (x) = ln √ +C
2 5 − sin 2x
Z r √
ln(x+ x2 +1)
8. I (x) = 1+x2
dx. Eseguiamo il cambio di variabile:
 √  dx
ξ = ln x + x2 + 1 =⇒ √ = dξ
1 + x2
Quindi:
Z p
I (ξ) = ξdξ
2p 3
ξ +C =
r  3
2 √ 
I (x) = ln3 x + x2 + 1 + C
3
Z
9. I (x) = x2 cosh (x3 + 3) dx. Eseguiamo il cambio di variabile:


ξ = x2 + 3 =⇒ x2 dx =
3
Quindi:
Z
1
I (ξ) = cosh ξdξ
3
1
= sinh ξ + C
3
1 
I (x) = sinh x3 + 3 + C
3
3.6 Integrazione per sostituzione 61

Z
3tanh x
10. I (x) = cosh2 x
dx. Eseguiamo il cambio di variabile:

dx
ξ = tanh x =⇒ = dξ
cos2 x
Quindi:
Z
I (ξ) = 3ξ dξ


= +C
ln 3
3tanh x
I (x) = +C
ln 3

3.6 Integrazione per sostituzione


Assegnato l’integrale:
Z
f (x) dx (3.2)

si esegue la sostituzione:

x = φ (ξ) (3.3)
In tal modo l’integrale (3.2) diventa:
Z
f [φ (ξ)] φ′ (ξ) dξ (3.4)

La scelta della funzione φ (ξ) deve essere tale che (3.4) è riconducibile agli integrali
fondamentali.
Esempio 1
Z

I (x) = x x − 1dx

Poniamo:

ξ= x − 1,
da cui:

x = ξ2 + 1
dx = 2ξdξ
3.6 Integrazione per sostituzione 62

L’integrale diventa:
Z  
 1 5 1 3
I (ξ) = 2 ξ + 1 ξ 2 dξ = 2
2
ξ + ξ +C
5 3
Ripristinando la variabile x:
2 2
I (x) = (x − 1)5/2 + (x − 1)3/2 + C
5 3
Esempio 2
Z
dx
I (x) = √
5x − 2
Poniamo:

ξ = 5x − 2
da cui:

dξ = 5dx
L’integrale diventa:
Z
1 dξ 2p
I (ξ) = √ = ξ+C
5 ξ 5
Ripristinando la variabile x:
2√
I (x) = 5x − 2 + C
5
Esempio 3
Z
xdx
I (x) = √
1 + x4
Poniamo:

ξ = x2
da cui:

dξ = 2xdx
L’integrale diventa:
Z
1 dξ 1 p
2


I (ξ) = p = ln ξ + ξ + 1 + C
2 1+ξ 2 2
Ripristinando la variabile x:
3.6 Integrazione per sostituzione 63

1  2 √ 4 
I (x) = ln x + x + 1 + C
2
***
In molti casi è conveniente eseguire le sostituzioni trigonometriche. Precisa-
mente, se l’integrando contiene uno dei seguenti radicali:
√ √ √
a2 − x 2 , x 2 − a2 , x 2 + a2 ,
si eseguono le sostituzioni:

√ √
a2 − x2 , x = a sin ξ =⇒ a2 − x2 = a cos ξ
√ a √
x 2 − a2 , x = =⇒ x2 − a2 = a tan ξ
cos ξ
√ √ a
x2 + a2 , x = a tan ξ =⇒ x2 + a2 =
cos ξ
Esempio 4
Z √
x2 + 1
I (x) = dx
x2
Poniamo:

x = tan ξ
da cui:
√ 1
x2 + 1 =
cos ξ
L’integrale diventa:
Z

I (ξ) = 2
sin ξ cos ξ
Z
sin2 ξ + cos2 ξ
= dξ
sin2 ξ cos ξ
= I1 (ξ) + I2 (ξ) ,
essendo:

Z
dξ 1
I1 (ξ) = = ln + tan ξ + C1
cos ξ cos ξ
Z Z
cos ξ d (sin ξ) 1
I1 (ξ) = 2 dξ = 2 =− + C2 ,
sin ξ sin ξ sin ξ
3.6 Integrazione per sostituzione 64

Quindi:

1 1
I (ξ) = ln + tan ξ − +C
cos ξ sin ξ
Ripristinando la variabile x e osservando che

1 p √
= 1 + tan2 ξ = 1 + x2
cos ξ
r r
1 1 1
= 1+ 2
= 1 + 2,
sin ξ tan ξ x
si ottiene:
√ √1 + x2
2
I (x) = ln x + 1 + x − +C
x

3.6.1 Calcolare gli integrali:


Z Z Z
10 2x
1) √dx
x x2 −2
6) x (2x + 5) dx 11) √ex
e +1
dx
Z Z Z
sin3 x
2) dx
ex +1
7) 1−x
√ dx
1+ x
12) √
cos x
dx
Z Z Z
7
3) 2
x (5x − 3) dx 8) √dx
x 2x+1
13) √dx
x 1+x2
dx
Z Z
4) √xdx
x+1
9) √ dx
ex −1
Z Z
5) √cos xdx 10) ln 2x dx
ln 4x x
1+sin2 x

3.6.2 Soluzioni
Z
1. I (x) = √dx
x x2 −2
. Procediamo per sostituzione:

1 dx dξ
ξ= =⇒ =−
x x ξ
3.6 Integrazione per sostituzione 65

Quindi:
Z

I (ξ) = − q √ 2
1− ξ 2
Z √ 
1 d ξ 2
= −√ q √ 2
2 1− ξ 2
1  √ 
= − √ arcsin ξ 2 + C
2
√ !
1 2
I (x) = − √ arcsin +C
2 x
Z
2. I (x) = dx
ex +1
. Procediamo per sostituzione:

dx dξ
x = − ln ξ =⇒ = −
ex + 1 ξ+1

Quindi:
Z

I (ξ) = −
ξ+1
= − ln |ξ + 1| + C

I (x) = − ln 1 + e−x + C
Z
7
3. I (x) = x (5x2 − 3) dx. Procediamo per sostituzione:


5x2 − 3 = ξ =⇒ xdx =
10
Quindi:
Z
1
I (ξ) = ξ 7 dξ
10
1
= ξ8 + C
80
1 8
I (x) = 5x2 − 3 + C
80
Z
4. I (x) = √xdx .
x+1
Procediamo per sostituzione:

√ dx
ξ= x + 1 =⇒ √ = 2dξ
x+1
3.6 Integrazione per sostituzione 66

Quindi:
Z

I (ξ) = 2 ξ 2 − 1 dξ
2
= ξ 3 − 2ξ + C
q 3
2 √
I (x) = (x + 1)3 − 2 x + 1 + C
3
Z
5. I (x) = √cos xdx . Procediamo per sostituzione:
2
1+sin x

ξ = sin ξ =⇒ cos xdx = dξ

Quindi:
Z

I (ξ) = 2 p
1 + ξ2
p
2
= ln ξ + 1 + ξ + C
p
I (x) = ln sin x + 1 + sin2 x + C

Z
6. I (x) = x (2x + 5)10 dx. Poniamo

ξ = 2x + 5 =⇒ dξ = 2dx

Quindi:
Z 

ξ 5 10
1
I (ξ) = − ξ dξ
2
2 2
1 5
= ξ 12 − ξ 11 + C
 24 44 
1 1 5
I (x) = (2x + 5)12 − (2x + 5)11 + C
4 12 11
Z Z √ √ Z Z √
(1− x)(√1+ x) √
7. 1−x
√ dx
1+ x
= 1+ x
dx = dx − xdx = x − 3
2
x3 + C
Z
8. I (x) = √dx
x 2x+1
. Poniamo

√ dx
ξ= 2x + 1 =⇒ dξ = √
2x + 1
3.6 Integrazione per sostituzione 67

Quindi:
Z

I (ξ) = 2 2
ξ −1

ξ − 1
= ln
+C
ξ + 1

2x + 1 − 1
I (x) = ln √ +C
2x + 1 + 1
Z
9. I (x) = √ dx
ex −1
. Poniamo

√ dx dξ
ξ= ex − 1 =⇒ √ =2 2
x
e −1 ξ +1

Quindi:
Z

I (ξ) = 2
+1 ξ2
= arctan ξ + C
√ 
I (x) = 2 arctan ex − 1 + C
Z Z
10. I (x) = ln 2x dx
ln 4x x
= ln 2+ln x dx
ln 4+ln x x
. Poniamo

dx
ξ = ln x =⇒ = dξ
x
Quindi:
Z
ln 2 + ξ
I (ξ) = dξ
ln 4 + ξ
Z  
ln 2 − ln 4
= 1+ dξ
ln 4 + ξ
Z
d (ξ + ln 4)
= ξ − ln 2
ln 4 + ξ
= ξ − ln 2 ln |ξ + ln 4| + C
I (x) = ln x − ln 2 ln |ln x + ln 4| + C
Z
2x
11. I (x) = √ex
e +1
dx. Poniamo

√ e2x dx 
ξ= ex + 1 =⇒ √ x = 2 ξ 2 − 1 dξ
e +1
3.6 Integrazione per sostituzione 68

Quindi:
Z

I (ξ) = 2 ξ 2 − 1 dξ
 
1 3
=2 ξ −ξ +C
3
2 √
I (x) = (ex − 2) ex + 1 + C
3
Z
sin3 x
12. I (x) = √
cos x
dx. Poniamo

sin3 x 1 − ξ2
ξ = cos x =⇒ √ dx = − √ dξ
cos x ξ

Quindi:
Z

I (ξ) = ξ 3/2 − ξ −1/2 dξ
2
= ξ 5/2 − 2ξ 1/2 + C
5 
√ 1 2
I (x) = 2 cos x cos x − 1 + C
5
Z
13. I (x) = √dx
x 1+x2
dx. Poniamo

1 dx dξ
ξ= =⇒ =−
x x ξ

Quindi:
Z

I (ξ) = − p
1 + ξ2
p

= − ln ξ + ξ 2 + 1 + C

x
I (x) = − ln √ +C
1 + x2 + 1

***
3.6 Integrazione per sostituzione 69

3.6.3 Calcolare i seguenti integrali utilizzando le sostituzioni


trigonometriche
Z Z √
2 x2 +1
1) √x dx
1−x2
5) x
dx
Z Z
3
2) √x dx
2−x2
6) √dx
x2 4−x2
Z Z

x2 −a2

3) x
dx 7) 1 − x2 dx
Z Z
4) √dx
x x2 −1
8) √ dx
x(1−x)

Z
2
1. I (x) = √x dx .
1−x2
Poniamo

x2 dx
x = sin ξ =⇒ √ = sin2 ξdξ
1−x 2

Quindi:
Z
I (ξ) = sin2 ξdξ
Z
1
= (1 − cos 2ξ) dξ
2
Z Z 
1 1
= dξ − cos 2ξd (2ξ)
2 2
ξ 1
= − sin 2ξ
2 4
x√
I (x) = arcsin x − 1 − x2 + C
2
Z
3
2. I (x) = √x dx .
2−x2
Poniamo

√ x3 dx
ξ= 2 sin ξ =⇒ √ = 23/2 sin3 ξdξ
2 − x2
3.6 Integrazione per sostituzione 70

Quindi:
Z
3/2
I (ξ) = 2 sin2 ξ sin ξdξ
Z
3/2

=2 cos2 ξ − 1 d (cos ξ)
 
3/2 1 3
=2 cos ξ − cos ξ + C
3
 
3/2 1 2
q q
=2 1 − sin ξ 1 − sin2 ξ − 1 − sin2 ξ + C
3
"  r r #
2 2 2
1 x x x
I (x) = 23/2 1− 1− − 1− +C
3 2 2 2
1 √
= − 4 + x2 2 − x2 + C
3
Z √
x2 −a2
3. I (x) = x
dx. Poniamo

a 1√ 2 a
x= =⇒ tan ξ = x − a2 ; dx = sin ξdξ
cos ξ a cos2 ξ
Quindi:
Z
sin2 ξ
I (ξ) = a dξ
cos ξ
Z
1 − cos2 ξ
=a dξ
cos ξ
Z Z 

=a − cos ξdξ
cos ξ
 
1
− sin ξ + C
= a ln tan ξ +
cos ξ
 √ 2 √ 2 
x − a2 + x x − a2
I (x) = a ln − +C

a x
Z
4. I (x) = √dx
x x2 −1
. Anziché eseguire una sostituzione trigonometrica, è
conveniente porre:
1 dx dξ
x= =⇒ =−
ξ x ξ
Z

I (ξ) = − p = − arccos ξ
1 − ξ2
1
I (x) = − arccos + C
x
3.6 Integrazione per sostituzione 71

Z √
x2 +1
5. I (x) = x
dx. Poniamo


x = tan ξ =⇒ dx =
cos2 ξ

Quindi:
Z

I (ξ) =
sin ξ cos2 ξ
Z
sin2 ξ + cos2 ξ
= dξ
sin ξ cos2 ξ
Z Z
sin ξ dξ
= 2
dξ +
cos ξ sin ξ
Z Z
d (cos ξ) dξ
=− 2
+
cos ξ sin ξ

1 1
= + ln − cot ξ + C
cos ξ sin ξ

Osservando che:

1 p x2 + 1
= 1 + tan2 ξ =
sin ξ x
1 √
= x2 + 1,
cos ξ

si ottiene: √
√ x2 + 1 − 1

I (x) = x2 + 1 + ln +C
x
Z
6. I (x) = √dx
x2 4−x2
. Anziché eseguire una sostituzione trigonometrica, è
conveniente porre:
1 dx
x= =⇒ 2 = −dξ
ξ x
Z
ξ
I (ξ) = − dξ p
4ξ 2 − 1
Z
1 d (4ξ 2 − 1)
=− p
8 4ξ 2 − 1
1p 2
=− 4ξ − 1 + C
4 √
4 − x2
I (x) = − +C
4x
Z Z
2
3.7 sin xdx, cos2 xdx 72

Z

7. I (x) = 1 − x2 dx. Poniamo
 √ 
x = sin ξ =⇒ dx = cos ξdξ, 1 − x2 = cos ξ

Quindi:
Z
I (ξ) = cos2 dξ
Z
cos 2ξ + 1
= dξ
2
 Z Z 
1 1
= cos 2ξd (2ξ) + dξ
2 2
 
1 1
= sin 2ξ + ξ + C
2 2
1 x√
I (x) = arcsin x + 1 − x2 + C
2 2
Z
8. I (x) = √ dx
. Poniamo
x(1−x)

 p 
2
x = sin ξ =⇒ dx = 2 sin ξ cos ξdξ, x (1 − x) = sin ξ cos ξ

Quindi:
Z
I (ξ) = 2 dξ

= 2ξ + C

I (x) = 2 arcsin x + C
Z Z
3.7 sin2 xdx, cos2 xdx

Dagli esercizi precedenti, risulta:

Z
1 1
sin2 xdx =(x − sin x cos x) + C = (2x − sin 2x) + C (3.5)
2 4
Z
1 1
cos2 xdx = (x + sin x cos x) + C = (2x + sin 2x) + C
2 4
3.8 Integrazione per parti 73

3.8 Integrazione per parti


Proposizione. Se f (x) e g (x) sono funzioni derivabili, sussiste la formula di
integrazione per parti:
Z Z
f (x) g (x) dx = f (x) g (x) − g (x) f ′ (x) dx,

(3.6)

che può essere riscritta come:


Z Z
f (x) dg (x) = f (x) g (x) − g (x) df (x) (3.7)

Dimostrazione. È immediata: basta applicare l’operatore di derivazione ad ambo


i membri della (3.7).
Esempio 1

Z Z   Z
x2 x2 1 dx
x ln xdx = ln xd = ln x − x2
2 2 2 x
x2 1
= ln x − x2 + C
2 4
x2
= (2 ln x − 1) + C
4
Esempio 2

Z Z Z
x
e cos xdx = e d (sin x) = e sin x − ex sin xdx
x x

Z
= e sin x − ex d (− cos x)
x

Z
= e sin x + e cos x − ex cos xdx,
x x

R
risolvendo rispetto a ex cos xdx:
Z
ex
ex cos xdx = (sin x + cos x) + C
2
3.8 Integrazione per parti 74

3.8.1 Calcolare gli integrali:


Z Z Z
1) ln xdx 6) x
ex
dx 11) (x2 + 5x + 6) cos 2xdx
Z Z Z
2) arctan xdx 7) −x
x · 2 dx 12) x2 ln xdx
Z Z Z
3) arcsin xdx 8) 2 3x
x e dx 13) ln2 xdx
Z Z Z
4) x sin xdx 9) xe 3 −x/3
dx 14) ln x
x3
dx
Z Z Z
5) x cos 3xdx 10) x sin x cos xdx 15) ln
√ x dx
x

3.8.2 Soluzioni
Z Z Z
1. I (x) = ln xdx = x ln x − xd (ln x) = x ln x − dx = x (ln x − 1) + C
Z Z
2. I (x) = arctan xdx = x arctan x− xdx
1+x2
= x arctan x − 12 ln (1 + x2 ) + C
Z Z Z
d(1−x2 )
3. I (x) = arcsin xdx = x arcsin x− √xdx
= x arcsin x +1−x2
1
2

1−x2

= x arcsin x + 1 − x2 + C
Z Z Z
4. I (x) = x sin xdx = xd (− cos x) = −x cos x + cos xdx

= −x cos x + sin x + C
Z Z Z
 1
5. I (x) = x cos 3xdx = xd 3 sin 3x = 3 x sin 3x − 9 sin 3xd (3x)
1 1


= 31 x sin 3x + 13 cos 3x + C
Z Z Z Z
6. I (x) = ex dx = xe dx = xd (−e ) = −xe + e−x dx = −xe−x −
x −x −x −x
Z
e−x d (−x)
Z Z
= xe + e dx = −xe − e−x d (−x) = −xe−x − e−x + C = − x+1
−x −x −x
ex
+C
Z Z  −x  Z
7. I (x) = −x
x·2 dx = x·d − ln 2 = − ln 2 x·2 + ln 2 2−x dx = − 2x+1
2 1 −x 1
x ln 2 +

C
3.8 Integrazione per parti 75

Z
8. I (x) = x2 e3x dx. Anzichè integrare per parti, è conveniente utilizzare il
metodo dei coefficienti indeterminati. Assegnato l’integrale:
Z
In,m (x) = pn (x) emx dx,

essendo pn (x) un polinomio di grado n:


n
X
pn (x) = ak x k
k=0

risulta:
In,m (x) = qn (x) emx , (3.8)

qui è:
n
X
qn (x) = bk xk
k=0

Applichiamo l’operatore di derivazione ad ambo i membri della (3.8):


Z
D pn (x) emx dx = D [qn (x) emx ] ,

ottenendo:
pn (x) = qn′ (x) + mqn (x) (3.9)

Il principio di identità dei polinomi applicato alla (3.9) conduce ad un sis-


tema di equazioni lineari che permette di ricavare i coefficienti indeterminati
bk , e quindi l’integrale In,m (x).
Nel caso in esame è:
Z

I2,3 (x) = x2 e3x dx = b2 x2 + b1 x + b0 e3x

x2 = 3b2 x2 + (3b1 + 2b2 ) x + b2 + 3b0

Affinchè sia verificata l’ultima, deve essere:

3b2 = 1
3b1 + 2b2 = 0
b2 + 3b0 = 0,

da cui i coeffcienti indeterminati:


 
1 2 2
(b2 , b1 , b0 ) = ,− , ,
3 9 27
3.8 Integrazione per parti 76

quindi l’integrale:
e3x 
I2,3 (x) = 9x2 − 6x + 2
27
Z
9. I (x) = x3 e−x/3 dx. Procedendo come nell’esercizio precedente:
Z
 x
x3 e−x/3 dx = b3 x2 + b2 x2 + b1 x + b0 e− 3 ,

donde:
 1 
x3 e−x/3 = 3b3 x2 + 2b2 x + b1 e−x/3 − b3 x3 + b2 x2 + b1 x + b0 e−x/3
 3  
b3 3 b2 2 b1 b2
= − x + 3b3 − x + 2b2 − x + b1 −
3 3 3 3

da cui:
b3 + 3 = 0
b2
3b3 − =0
3
b1
2b2 − =0
3
b0 − 3b1 = 0

Z

x3 e−x/3 dx = −3 x3 + 9x2 + 54x + 162 e−x/3
Z Z
10. I (x) = x sin x cos xdx = 1
x sin 2xdx
2
Z  Z 
1 cos 2x
 1 1 1
= 2 xd − 2 = 2 − 2 x cos 2x + 4 cos 2xd (2x)

= 41 (sin 2x − 2x cos 2x) + C


Z
11. I (x) = (x2 + 5x + 6) cos 2xdx. Anziché procedere per un’integrazione
per parti, conviene applicare il metodo dei coefficienti indeterminati. Più
in generale, consideriamo l’integrale:
Z
def
In,α (x) = pn (x) cos (αx) dx (3.10)

L’integrale (3.10) ha la forma:


In,α (x) = qn (x) cos (αx) + rn (x) sin (αx) , (3.11)
3.8 Integrazione per parti 77

essendo:
n
X
pn (x) = ak x k
k=1
Xn
qn (x) = bk xk
k=1
n
X
rn (x) = c k xk
k=1

Applicando l’operatore di derivazione ad ambo i membri della (3.11) ed


eseguendo le dovute semplificazioni:
1   
In,α (x) = 3 (a1 + 2a2 x) α cos (αx) + (a0 + a1 x) α2 + a2 −2 + α2 x2 sin (αx)
α
Nel caso in esame è:
1  
I2,2 (x) = (5 + 2x) cos (2x) + 2x2 + 10x + 11 sin (2x)
4
Z Z  3 Z
x3 3
12. I (x) = x ln xdx = ln xd 3 = 3 ln x− 3 x2 dx = x9 (3 ln x − 1)+C
2 x 1

Z Z

13. I (x) = ln xdx = x ln x − 2 ln xdx = x ln2 x − 2 ln x + 2 + C
2 2

Z Z Z Z
 
14. I (x) = ln x
x3
dx = ln xd − 2x12 = ln xd − 2x12 = − 2x12 ln x + 1
2
1 dx
x2 x

= − 2x12 ln x − 41 x12 + C = − 1+ln


4x2
x
+C
Z Z Z
√ √ √ dx
15. I (x) = ln
√ x
x
dx = ln xd (2 x) = 2 x ln x − 2 xx
Z
√ √
= 2 x ln x − 2 x−1/2 dx = 2 x (ln x − 2) + C

***

3.8.3 Calcolare gli integrali:


Z Z Z
1) ln
√ x dx
x
5) xdx
sin2 x
9) eax sin bxdx
Z Z
R
2) x arctan xdx 6) x cos xdx
sin2 x
10) sin (ln x) dx
Z Z
3) x arcsin xdx 7) ex sin xdx
Z Z
√ 
4) ln x + 1 + x2 dx 8) 3x cos xdx
3.8 Integrazione per parti 78

3.8.4 Soluzioni
Z Z Z
√ √ √ dx
1. I (x) = ln
√ x dx
=
x
ln xd (2 x) = 2 x ln x − 2 xx

= 2 x (ln x − 2) + C
Z Z  2 Z
x2 x2
2. I (x) = x arctan xdx = arctan xd x2 = 2
arctan x − 1
2 x2 +1
dx

Poniamo:
Z
x2
J (x) = dx
x2 + 1
Z 2
x +1−1
= dx
x2 + 1
Z Z
dx
= dx − 2
x +1
= x − arctan x + C1 ,

donde:
x2 + 1 x
arctan x − + C
I (x) =
2 2
Z Z  2 Z
x2 2
3. I (x) = x arcsin xdx = arcsin xd 2 = 2 arcsin x − 2
x 1 √x dx
1−x2

Poniamo:
Z
x2 dx
J (x) = √
1 − x2
Z 2
x −1+1
= √ dx
1 − x2
Z Z
x2 − 1 dx
= √ dx + √
Z1 − x2 1 − x2

=− 1 − x2 dx + arcsin x

R√
Calcoliamo a parte 1 − x2 dx:
Z √ Z
1− x2 dx = sin2 ξdξ
x=sin ξ
Z
1
= (1 − cos 2ξ) dξ
2
 
1 1 1 √ 
= ξ − sin 2ξ + C1 = arcsin x − x 1 − x2 + C1 ,
2 2 ξ=arcsin x 2
3.8 Integrazione per parti 79

donde:
x2 1 √ 
arcsin x −
I (x) = arcsin x − x 1 − x2 + C
2 4
Z Z
√  √ 
4. I (x) = ln x + 1 + x dx = x ln x + 1 + x − √1+x
2 2 x
2 dx

Z
√  d(1+x2 ) √  √
2
= x ln x + 1 + x − √ = x ln x + 1 + x 2 − 2 1 + x2 + C
1+x 2

Z Z Z
5. I (x) = xdx
sin2 x
= xd (− cot x) = −x cot x + cot xdx = −x cot x +
Z
cos x
sin x
dx

= −x cot x + ln |sin x| + C
Z Z Z
6. I (x) = x cos xdx
sin2 x
= x cos xd (− cot x) = −x cos x cot x+ cot x (cos x − x sin x) dx

Poniamo:
Z
J (x) = cot x (cos x − x sin x) dx
Z
cos2 x − x cos x sin x
= dx
sin x
Z Z
cos2 x
= dx − x cos xdx
sin x
Z  Z 
1 − sin2 x
= dx − x sin x − sin xdx
sin x
Z
dx x
= − x sin x = ln tan − x sin x + C,
sin x 2

donde:
x

I (x) = −x cos x cot x + ln tan − x sin x + C
2
x x
=− + ln tan + C
sin x 2
Z Z Z
7. I (x) = e sin xdx = e d (− cos x) = −e cos x + cos xd (ex )
x x x

Poniamo:
Z
J (x) = ex cos xdx
Z
= ex d (sin x) = ex sin x − I (x) ,
3.8 Integrazione per parti 80

da cui:
ex
(sin x − cos x) + C
I (x) =
2
Z Z Z
8. I (x) = 3 cos xdx = 3 d (sin x) = 3 sin x − sin xd (3x ) = 3x sin x −
x x x
Z
1
ln 3
3x sin xdx

Poniamo:
Z Z
x
J (x) = 3x d (− cos x)
3 sin xdx =
Z
x 1
= −3 cos x + 3x cos xdx
ln 3

Quindi
I (x) = 3x sin x − ln 3 [−3x cos x + I (x) ln 3]

Risolvendo rispetto a I (x):

3x (cos x ln 3 + sin x)
I (x) = +C
1 + ln3
Z
9. Ia,b (x) = eax sin bxdx

Abbiamo
Z  
1 ax
Ia,b (x) = e d − cos bx
b
Z
1 a
= − eax cos bx + eax cos bxdx
b b

Poniamo:
Z Z  
ax 1 ax
Ja,b (x) = e cos bxdx = e d sin bx
b
1 a
= eax sin bx − Ia,b (x) ,
b b

donde:
eax (a sin bx − b cos bx)
Ia,b (x) = +C
a2 + b 2
3.8 Integrazione per parti 81

R R
10. I (x) = sin (ln x) dx = x sin (ln x) − cos (ln x) dx = x sin (ln x) − J (x)
essendo:
Z
J (x) = cos (ln x) dx

= x cos (ln x) + I (x) ,

quindi:
x
I (x) = [x sin (ln x) − cos (ln x)] + C
2

3.8.5 Formule ricorrenti


In questa sezione determiniamo alcune formule ricorrenti utilizzando l’integrazioni
per parti.
Z
(±) dx
In (x) = (3.12)
(a ± x2 )n
2

(+)
Calcoliamo In (x):

Z
1 a2 + x 2 − x 2
In(+) (x) = 2 dx
a (a2 + x2 )n
1 h (+) i
= 2 In−1 (x) − Jn (x) ,
a
essendo:
Z
x2
Jn (x) = dx
(a2 + x2 )n
Osserviamo che:
 
x d 1 1
= − ,
(a2 + x2 )n dx 2 (n − 1) (a2 + x2 )n−1
quindi:
x 1 1 (+)
Jn (x) = − n−1 + In−1 (x)
2 (n − 1) (a + x )
2 2 2 (n − 1)
Perciò:
 
1 x 1 2n − 3 (+)
In(+) (x) = 2 + I (x)
a 2 (n − 1) (a2 + x2 )n−1 2 (n − 1) n−1
(−)
Similmente si perviene a In (x):
3.8 Integrazione per parti 82

 
1 x 1 2n − 3 (±)
In(±) (x) = 2 + I (x) (3.13)
a 2 (n − 1) (a2 ± x2 )n−1 2 (n − 1) n−1
Si osservi che la (3.13) è valida per n 6= 1:

(+) 1 x
I1 (x) =
arctan + C
a a
(−) 1 x
I1 (x) = arctan h + C
a a
Esempi:

 
(+) 1 x 1 x
I2 (x) = + arctan +C
a2 2 (a + x2 ) 2a a
 
(+) 1 x 3 (+)
I3 (x) = + I (x)
a2 4 (a + x2 )2 4 2
  x 
1 x (3x2 + 5a2 ) 3
= + arctan +C
8a4 (a + x2 )2 a a
Z
dx
(1 + x2 )5/2
Z
x 2 dx
= +
3 (1 + x2 )3/2 3 (1 + x2 )3/2
Z
dx x
= + C1 =⇒
(1 + x2 )3/2 (1 + x2 )1/2
Z
dx x 2 x
= + +C
(1 + x2 )5/2 3 (1 + x2 )3/2 3 (1 + x2 )1/2

Calcoliamo:
Z
n
In(±) (x) = a2 ± x 2 dx (3.14)
(+)
Calcoliamo In (x):
Z
n
In(+) (x) = a2 + x 2 dx

Sviluppiamo l’integrando nel seguente modo:

n  n−1
a2 + x 2 = a2 + x 2 a2 + x 2
n−1 n−1
= a2 a2 + x 2 + x 2 a2 + x 2 ,
3.8 Integrazione per parti 83

donde:
(+)
In(+) (x) = a2 In−1 (x) + Jn (x) ,
essendo:
Z
n−1
Jn (x) = x 2 a2 + x 2 dx

Osserviamo che:
 
2

2 n−1 d 1 2

2 n
x a +x = a +x
dx 2n
Ciò implica che possiamo eseguire un’integrazione per parti:
x 2 n 1
Jn (x) = a + x2 − In(+) (x)
2n 2n
Quindi:
2n + 1 (+) x 2 n (+)
In (x) = a + x2 + a2 In−1 (x)
2n 2n
(−)
Analogo risultato per In (x). In definitiva abbiamo ottenuto la seguente re-
lazione:

x n 2na2 (±) 1
In(±) (x) = a2 ± x 2 + In−1 (x) , ∀n 6= − (3.15)
2n + 1 2n + 1 2
Esempio:

Z
(+) 3/2
I3/2 (x) = 9 + x2 dx (3.16)
x 3/2 27 (+)
= 9 + x2 + I1/2 (x)
Z4 4
(+)  1/2
I1/2 (x) = 9 + x2 dx
x 1/2 9 h 1/2 i
= 9 + x2 + ln 9 + 9 + x2 + C1 =⇒
2 2
(+) x 3/2 x 1/2 9 h 1/2 i
I3/2 (x) = 9 + x2 + 9 + x2 + ln 9 + 9 + x2 +C
4 2 2
In maniera simile si dimostrano le seguenti:
3.8 Integrazione per parti 84

Z  Z 
dx 1 x 1 2n − 3 dx
=− 2 +
(x2 − a2 )n a 2 (n − 1) (x2 − a2 )n−1 2 (n − 1) (x2 − a2 )n−1
(3.17)
Z Z
n x n 2n n−1
x2 − a2 dx = x 2 − a2 − x 2 − a2 dx
2n + 1 2n + 1
Ricaviamo ora una formula ricorrente per l’integrale:
Z
In (x) = xn eax dx (3.18)

Abbiamo:

Z  
1 axn
In (x) = x d e
a
Z
1 n ax n
= x e − xn−1 eax dx
a a
Cioè:
1 n
In (x) = xn eax − In−1 (x)
a a
Ad esempio:

Z
I3 (x) = x3 eax dx
1 3
= x3 eax − I2 (x)
a a
1 2 ax 2
I2 (x) = x e − I1 (x)
a a
1 2 ax 2
I1 (x) = x e − I0 (x)
a a
1 ax
I0 (x) = xe + C0
a
da cui:
eax 3 3 2 2

I3 (x) = a x − 3a x + 6ax − 6 +C
a4
3.8 Integrazione per parti 85

3.8.6 Calcolare i seguenti integrali applicando il metodo


più opportuno
Z Z Z √
ln2 x arcsin x
1) 3
x sin xdx 7) x2
dx 13) √
1−x
dx
Z Z Z
√ ln(ln x)
2) x ln (2x) dx 8) x
dx 14) x tan2 2xdx
Z Z Z
3 −x2 sin2 x
3) xe dx 9) 2
x arctan 3xdx 15) ex
dx
Z Z

4) e x
dx 10) x (arctan x)2 dx
Z Z
5) (x2 − 2x + 3) ln xdx 11) (arcsin x)2 dx
Z Z

6) x ln 1+x
1−x
dx 12) arcsin
x2
x
dx

3.8.7 Soluzioni
Z
1. I (x) = x3 sin xdx

Poniamo:
Z 3
X 3
X
3 k
x sin xdx = sin x bk x + cos x c k xk
k=1 k=1

Derivando primo e secondo membro, e risolvendo rispetto ai coefficienti


bk , c k :
Z
 
x3 sin xdx = −x x2 − 6 cos x + 3 x2 − 2 sin x + C

Z Z Z
√ 
2. I (x) = x ln (2x) dx = ln (2x) d 2 3/2
3
x = 2 3/2
3
x ln (2x) − 2
3
x1/2 dx

= 32 x3/2 ln (2x) − 49 x3 + C,
donde:
2√ 3
x [3 ln (2x) − 2] + C
I (x) =
9
Z Z Z Z
3 −x2 −x2 −x2 2
3. I (x) = x e dx = x xe dx = 2 x xe dx = 2 x2 e−x d (x2 )
2 1 2 1

Z
= 2 ξe−ξ dξ. Calcoliamo a parte quest’integrale utilizzando il metodo dei
ξ=x
coefficienti indeterminati:
Z
ξe−ξ dξ = (b0 + b1 ξ) e−ξ + C
3.8 Integrazione per parti 86

Derivando primo e secondo membro:

ξe−ξ = (b0 + b1 ξ) e−ξ ,

da cui:
Z
(b0 , b1 ) = (−1, −1) =⇒ ξe−ξ dξ = − (1 + ξ) e−ξ + C

Ripristinando la variabile x:
1  2
I (x) = − 1 + x2 e−x + C
2
Z

4. I (x) = e x
dx. Eseguiamo il cambio di variabile:

ξ= x =⇒ dx = 2ξdξ,

donde:
Z  Z 
I (ξ) = 2 ξe dξ = 2 ξe − e dξ = 2 (ξ − 1) eξ + C,
ξ ξ ξ

ripristinando la variabile x:
√  √
I (x) = 2 x−1 e x+C
Z
5. (x2 − 2x + 3) ln xdx = 13 x3 ln x − 91 x3 − (ln x) x2 + 12 x2 + 3x ln x − 3x + C
Z
 x2 −1

6. x ln 1−x
1+x
dx = 2
ln 1−x
1+x
+C
Z
ln2 x

7. x2
dx =−− 1
x
ln2 x + 2 ln x + 2 + C
Z
ln(ln x)
8. x
dx = ln x [ln (ln x) − 1] + C
Z
 
9. x2 arctan 3xdx = 13 x3 arctan 3x − 1
18
x2 − 91 ln (1 + 9x2 ) + C
Z
10. x (arctan x)2 dx = x2 +1
2
(arctan x)2 − x arctan x + 21 ln (1 + x2 ) + C
Z
√ 
11. (arcsin x)2 dx = arcsin x x arcsin x + 2 1 − x2 − 2x + C
3.9 Integrali contenenti un trinomio di secondo grado 87

Z
12. arcsin x
− x1
√x
x2
dx = arcsin x + ln 1+ 1−x2 + C
Z √
arcsin x √ √ √
13. √
1−x
dx = −2 1 − x arcsin x + 2 x + C
Z
14. x tan2 2xdx = 12 x tan 2x − 12 x2 − 18 ln (1 + tan2 2x) + C
Z
sin2 x
15. ex
dx = 1 − sin x−2 cos x
5 ex
sin x − 2
5ex
+C

3.9 Integrali contenenti un trinomio di secondo


grado
Consideriamo gli integrali del tipo:
Z
mx + n
F (x) = dx, (3.19)
ax2 + bx + c
essendo m, n, a, b, c ∈ R. Senza perdita di generalità, consideriamo a > 0.
Poniamo:

mx + n = A (2ax + b) + B, (3.20)
da cui:
m mb
A= , B =n− , (3.21)
2a 2a
donde:

Z Z  Z
mx + n m 2ax + b mb dx
2
dx = 2
dx + n − 2
ax + bx + c 2a ax + bx + c 2a ax + bx + c
 
m 2 mb
= ln ax + bx + c + C1 + n − G (x) ,
2a 2a

essendo:
Z
dx
G (x) =
ax2 + bx + c
Per calcolare G (x) procediamo nel modo seguente:

ax2 + bx + c = a (x + k)2 + l, (3.22)


da cui si ricava:
3.9 Integrali contenenti un trinomio di secondo grado 88

b b2 ∆
k= , l =c− =− , (3.23)
2a 4a 4a
essendo

∆ = b2 − 4ac, (3.24)
il discriminante del trinomio.
Distinguiamo i tre casi:

1. ∆ < 0
2. ∆ > 0
3. ∆ = 0

Iniziamo con il caso 1:


Z
1 dx
G (x) = p a 2
l 1+ (x + k)
l
Eseguiamo il cambio di variabile:
r r
a l
(x + k) = ξ =⇒ dx = dξ (3.25)
l a
L’integrale G (ξ) diventa:
r Z
1 dξ 1
G (ξ) = 2
= √ arctan ξ + C2
al 1+ξ al
Ripristinando la variabile x:
r  
1 a b
G (x) = √ arctan x+ + C2
al l 2a
Osserviamo che:

a 4a2 ∆
=− a·l =− , (3.26)
l ∆ 4
quindi:  
2 2ax + b
G (x) = √ arctan √ + C2 (3.27)
−∆ −∆
Finalmente l’integrale (3.19):

   
m  2 mb 2ax + b
F (x) = ln ax2 + bx + c + √ n− arctan √ + C (3.28)
2a −∆ 2a −∆
3.9 Integrali contenenti un trinomio di secondo grado 89

Esempio 7. Calcoliamo:
Z
x+2
F (x) = dx
x2 − 3x + 10
Qui è ∆ = −31, quindi applichiamo la (3.28):
 
1  7 2x − 3
F (x) = ln x2 − 3x + 10 + √ arctan √ +C
2 31 31
Nel caso speciale m = 0, n = 1:
Z  
dx 2 2ax + b
F (x) = =√ arctan √ +C (3.29)
ax2 + bx + c −∆ −∆
***

Nel caso 2, la (3.22) si scrive:

ax2 + bx + c = a (x + k)2 − |l| , (3.30)


giacché l < 0. Quindi

Z
dx
G (x) =
a (x + k)2 − |l|
Z
1 d (x + k)
=
a (x + k)2 − |l|
a
q
|l|
1 x + k − a
= p ln
q + C2
2 a |l| x + k + |l|
a


1 2ax + b − ∆
= √ ln √ + C2 ,
∆ 2ax + b + ∆
Esempio 8. Z
dx
G (x) =
x2 − 2x − 3
Qui è: ∆ = 16, quindi:
1 x − 3
G (x) = ln +C
4 x + 1
cioè:

2ax + b − √∆
 
m 2 1 mb
(3.31)

F (x) = ln ax + bx + c + √ n− ln √ +C
2a ∆ 2a 2ax + b + ∆
3.9 Integrali contenenti un trinomio di secondo grado 90

***

Caso 3 (∆ = 0): l = 0, per cui

Z
dx
G (x) =
a (x + k)2
Z
1 d (x + k)
=
a (x + k)2
2
=− + C2 ,
2ax + b
cioè:
 
m 2
 mb 2
F (x) = ln ax + bx + c − n − +C (3.32)
2a 2a 2ax + b
Esempio 9.
Z
3x + 2
F (x) = dx
x2 − 4x + 4
3  8
= ln x2 − 4x + 4 − +C
2 x−2
Riassumendo:

   
m 2
 2 mb 2ax + b
∆ < 0 =⇒ F (x) = ln ax + bx + c + √ n− arctan √ +C
2a −∆ 2a −∆
  √
m 2 1 mb 2ax + b − ∆
∆ > 0 =⇒ F (x) = ln ax + bx + c + √ n− ln √ +C
2a ∆ 2a 2ax + b + ∆
 
m  mb 2
∆ = 0 =⇒ F (x) = ln ax2 + bx + c − n − +C
2a 2a 2ax + b

3.9.1 Calcolare gli integrali:


Z Z Z
x2 −1
1) x2 −x−1
dx 4) 3x−2
x2 −4x+5
dx 7) dx
3x2 −x+1
Z Z
(x−2)2
2) dx
x2 +2x+5
5) x2 +3x+4
dx
Z Z
x2
3) dx
x2 +2x
6) x2 −6x+10
dx
3.9 Integrali contenenti un trinomio di secondo grado 91

3.9.2 Soluzioni
Z
x2 −1
R  R
1. F (x) = x2 −x−1
dx = 1+ x
dx = x +
x2 −x−1
x
x2 −x−1
dx
h √ i
= x + 12 ln |x2 − x − x| + √15 ln 2x−1+
2x−1−√ 5
5
+C
Z

2. F (x) = dx
x2 +2x+5
= 21 arctan x+1
2
+C
Z
x
3. F (x) = dx
x2 +2x
= 21 ln x+2 +C

Z
4. F (x) = 3x−2
x2 −4x+5
dx = 23 ln (x2 − 4x + 5) + 4 arctan (x − 2) + C
Z h  i
(x−2)2
5. F (x) = x2 +3x+4
dx =x− 7
2
ln (x2 + 3x + 4) − √6
7
arctan 2x+3

7
+C
Z
x2
6. F (x) = x2 −6x+10
dx = x + 3 ln (x2 − 6x + 10) + 8 arctan (x − 3) + C
Z √  
7. F (x) = dx
3x2 −x+1
= 2 11
11
arctan 6x−1

11
+C

***

Consideriamo integrali del tipo:


Z
mx + n
H (x) = √ dx, (3.33)
ax2 + bx + c
Qui vanno distinti i casi:

1. a > 0

2. a < 0

Consideriamo a > 0
Applicando l’artificio (3.20):

Z  Z
m 2ax + b mb dx
H (x) = √ dx + n − √
2a ax2 + bx + c 2a ax2 + bx + c
 
m√ 2 mb
= ax + bx + c + C1 + n − K (x) ,
a 2a

essendo:
3.9 Integrali contenenti un trinomio di secondo grado 92

Z
dx
K (x) = √
+ bx + cax2
Per calcolare K (x) applichiamo l’artificio (3.22):
Z
dx
K (x) = q ,
2
a (x + k) + l
cosicchè vanno distinti i tre casi:

1. ∆ < 0
2. ∆ > 0
3. ∆ = 0

Iniziamo con il caso 1:


Z
1 dx
K (x) = √ q
p a 2
l 1+ (x + k)
l
p
Eseguendo il cambio di variabile x → ξ = al (x + k):
Z
1 dξ 1 p
K (ξ) = √ p = √ ln ξ + ξ + 1 + C2
2
a 1+ξ 2 a
Ripristinando la variabile x:
r r
1 a a
2
K (x) = √ ln
(x + k) + 1 + (x + k) + C2
a l l
Per le (3.26):
2ax + b
ξ= √ ,
−∆
donde:
s
2
1 2ax + b (2ax + b)
K (x) = √ ln √
+ 1+ + C2
a −∆ |∆|
Finalmente l’integrale (3.33):

s
  2
m√ 1 mb 2ax + b (2ax + b)
H (x) = ax2 + bx + c + √ n− ln √
+ 1+ +C
a a 2a −∆ |∆|
(3.34)
3.9 Integrali contenenti un trinomio di secondo grado 93

Ad esempio:

Z
x+3
H (x) = √ dx
x2 + 2x + 2



2 2
= x + 2x + 2 + 2 ln x + 1 + x + 2x + 2 + C

Caso 2 (∆ > 0) vale la (3.30), donde:


Z
1 dx
K (x) = p s
|l| q 2
a
|l|
(x + k) − 1

Eseguiamo il cambio di variabile:


r
a
ξ= (x + k)
|l|

Z
1 dx
K (ξ) = √ p
a ξ2 − 1
1 p

= √ ln ξ + ξ − 1 + C2
2
a

Per le (3.26):
2ax + b
ξ= √ ,

donde:
s
2
1 2ax + b (2ax + b)
K (x) = √ ln √ + − 1 + C2
a ∆ ∆
Quindi:

s
  2
m√ 2 mb 1 2ax + b (2ax + b)
H (x) = ax + bx + c + n − √ ln √ + − 1 + C
a 2a a ∆ ∆
(3.35)
Caso 3 (∆ = 0):
3.9 Integrali contenenti un trinomio di secondo grado 94

Z
dx
K (x) = q
a (x + k)2
1
= √ ln |x + k| + C2
a

1 2ax + b
= √ ln + C2
a 2a
Quindi:
 
m√ 2 mb 1 2ax + b
H (x) = ax + bx + c + n − √ ln +C (3.36)
a 2a a 2a
Consideriamo a < 0
 
m√ 2 mb
H (x) = ax + bx + c + C1 + n − K (x)
a 2a
Qui è:
Z
dx
K (x) = q
− |a| (x + k)2 + l
con k e l dati dalla (3.23). Consideriamo solo il caso ∆ > 0, giacché per ∆ ≤ 0
l’integrando è immaginario.
Z
1 dx
K (x) = √ s  2
l q
|a|
1− l
(x + k)

Eseguendo il cambio di variabile


r
|a|
ξ= (x + k)
l
otteniamo:
1
K (ξ) = p arcsin ξ + C2
|a|
Ripristinando la variabile x:
r
4a2 2ax + b 2ax + b
ξ= =− √ =⇒ (3.37)
∆ 2a ∆ 
1 2ax + b
=⇒ K (x) = − p arcsin √ + C2 ,
|a| ∆
3.9 Integrali contenenti un trinomio di secondo grado 95

donde:

   
m√ 2 1 mb 2ax + b
H (x) = ax + bx + c − p n− arcsin √ +C (3.38)
a |a| 2a ∆
Esempi
Z
√ √ 
1. H (x) = √ 2x−3
−2x2 +3x+2
dx = − −2x2 + 3x + 2 − 43 2 arcsin 4x−3
5
+C
Z  
2. H (x) = √ dx
−x2 +2x+5
= arcsin x−1

6
+C

3.9.3 Calcolare gli integrali


Z Z
1) √ dx
x−x2
4) √ −x+2 dx
2−x−x2
Z Z
2) √ 2x−8 dx
1−x−x2
5) √ 10x−4
2+5x−7x2
dx
Z Z
3) √ 3x+2
1−4x−6x2
dx 6) √ −2x+3
14+5x−7x2
dx

3.9.4 Soluzioni
Z
1. H (x) = √ dx
x−x2
= arcsin (2x − 1) + C
Z
√  
2. H (x) = √ 2x−8 dx
1−x−x2
= −2 1 − x − x2 − 9 arcsin 2x+1

5
+C
Z
√ √ h√ i
3. H (x) = √ 3x+2
1−4x−6x2
dx = − 12 1 − 4x − 6x2 + 6
6
arcsin 10
5
(3x + 1) + C
Z
√ 
4. H (x) = √ −x+2 dx
2−x−x2
= 2 − x − x2 + 25 arcsin 2x+1
3
+C
Z
√ √ 
5. H (x) = √ 10x−4
2+5x−7x2
dx = − 10
7
2 + 5x − 7x2 − 3 7
49
arcsin 14x−5
9
+C
Z
−2x+3
6. H (x) = √14+5x−7x 2 dx

p √ √ 
= 72 (14 + 5x − 7x2 ) + 16
49
14
7 arcsin 417 417 x − 5
14
+C

Esercizio 10. Calcolare:


Z
dx
K (x) = p
x2 + px + q
3.9 Integrali contenenti un trinomio di secondo grado 96

2 2
Soluzione 11. x2 + px + q = x + p2 + 4q−p 4
Risulta ∆ = p2 − 4q, per cui se ∆ < 0 abbiamo:
Z
dx
K (x) = v (
u  2 )
u 4q−p2 p

t
4
1+ √ 2 x+ 2
4q−p2

Poniamo: 
2 p
ξ=p x+ ,
4q − p2 2
donde: p

K (ξ) = ln ξ + 1 + ξ 2
Cioè:
" #
2 p p

K (x) = ln p x + + x2 + px + q + C1
4q − p 2 2
p p
2
= ln x + + x + px + q + C,
2
 
essendo C = ln √ 2 2 + C1 .
4q−p
Se ∆ > 0:  p 2 |4q − p2 |
x2 + px + q = x + − ,
2 4
donde: Z
dx
K (x) = v (
u  2 )
u |4q−p2 | 
t
4
√ 2 2 x + p2 − 1
|4q−p |

Eseguendo il cambio di variabile:


2  p
ξ=p x+ ,
|4q − p2 | 2

otteniamo: p

K (ξ) = ln ξ + ξ − 1 + C1
In funzione di x:
" #
2 p p 2

K (x) = ln p x + + x + px + q + C1
|4q − p2 | 2
p p

= ln x + + x2 + px + q + C,
2
3.9 Integrali contenenti un trinomio di secondo grado 97

 
essendo C = ln √ 2 2 + C1 .
|4q−p |
Se ∆ = 0:  p 2
x2 + px + q = x + ,
2
donde: Z
dx p
K (x) = 2 = ln x + + C
x+ p 2
2

***

Consideriamo gli integrali del tipo:


Z
dx
L (x) = √ , (3.39)
(mx + n) ax2 + bx + c
essendo m, n, a, b, c ∈ R tali che (m, n) 6= (0, 0).
Gli integrali di questo tipo si riducono a quelli del tipo (3.33) eseguendo il cambio
di variabile:
1
ξ=
mx + n
Esempio
Z
dx
L (x) = √
(x + 1) x2 + 1
Esegiamo il cambio di variabile:
1 dξ
x+1= =⇒ dx = − 2 ,
ξ ξ
donde:

Z

L (ξ) = − p
2ξ 2
− 2ξ + 1
1 p

= − √ ln 2ξ − 1 + 4ξ 2 − 4ξ + 2 + C
2
Ripristinando la variabile indipendente:

1 1 − x + p2 (1 + x2 )

L (x) = − √ ln +C
2 1 + x
3.9 Integrali contenenti un trinomio di secondo grado 98

3.9.5 Calcolare gli integrali


Z Z
1) √dx
x 1−x2
3) dx

(x−1) x2 −2
Z Z
2) √ dx
x x2 +x−1
4) dx

(x+1) x2 +2x

3.9.6 Soluzioni
Z
1. L (x) = √dx
x 1−x2
. Eseguiamo il cambio di variabile:

1
x= ,
ξ

donde:
Z

L (ξ) = − p
ξ2 − 1
p
2
= − ln ξ + ξ − 1 + C

Ripristinando la variabile x:

x
L (x) = ln √ +C
1 + 1 − x2
Z
2. L (x) = √ dx
x x2 +x−1
. Eseguiamo il cambio di variabile:

1
x= ,
ξ

donde: Z

L (ξ) = − p
−ξ 2 + ξ + 1

Applicando la (3.37):
 
−2ξ + 1
L (ξ) = arcsin √ +C
5

Ripristinando la variabile x:
 
x−2
L (x) = arcsin √ +C
x 5
3.9 Integrali contenenti un trinomio di secondo grado 99

Z
3. L (x) = dx

(x−1) x2 −2
. Eseguiamo il cambio di variabile:

1
x−1= ,
ξ

donde: Z

L (ξ) = − p
−ξ 2 + 2ξ + 1

Applicando la (3.37):
 
−ξ + 1
L (ξ) = arcsin √ +C
2

Ripristinando la variabile x:
 
x−2
L (x) = arcsin √ +C
2 (x − 1)
Z
4. L (x) = dx

(x+1) x2 +2x
. Eseguiamo il cambio di variabile:

1
x+1= ,
ξ

donde:
Z

L (ξ) = − p
1 − ξ2
= − arcsin ξ + C

Ripristinando la variabile x:
 
1
L (x) = − arcsin +C
x+1

***

Consideriamo gli integrali del tipo:


Z √
M (x) = ax2 + bx + cdx (3.40)

Qui vanno distinti i casi:


3.9 Integrali contenenti un trinomio di secondo grado 100

1. a > 0

2. a < 0

Consideriamo a > 0
Ricaviamo dal trinomio un quadrato completo applichando l’artificio (3.22):
Z q
M (x) = a (x + k)2 + ldx

Eseguiamo il cambio di variabile:

ξ =x+k (3.41)
Quindi:
Z r
√ l
M (ξ) = a ξ 2 + dξ (3.42)
a
Calcoliamo l’integrale a secondo membro della (3.42), ponendo α = l/a. Inte-
grando per parti:

Z p Z
p ξ 2 dξ
ξ2 + αdξ = ξ ξ2 p+α−
ξ2 + α
Z
p (ξ 2 + α) − α
= ξ ξ2 + α − p dξ
ξ2 + α
Z p Z
p
2 2

=ξ ξ +α− ξ + αdξ + α p ,
ξ2 + α

donde:

Z p Z
ξp 2 α dξ
ξ 2 + αdξ = ξ +α+ p
2 2 ξ2 + α
ξp 2 α p

= ξ +α+ ln ξ + ξ 2 + α + C1
2 2
Osserviamo che:

2ax + b
ξ =x+k =
2a
∆ 2 ax2 + bx + c
α=− =⇒ ξ + α =
4a2 a
Quindi:
3.9 Integrali contenenti un trinomio di secondo grado 101

√ r
2ax + b √ 2 ∆ a 2ax + b ax2 + bx + c
M (x) = ax + bx + c − ln + + C2
4a 8a2 2a a
√  
2ax + b √ 2 ∆ a √ 2ax + b √
2

= ax + bx + c − ln a
+ ax + bx + c − ln a + C2

4a 8a2 2a

Incorporando ln a nella costante di integrazione:

∆ a √
C = C2 + ln a,
8a2
otteniamo:


2ax + b √ 2 ∆ a √ 2ax + b √ 2
M (x) = ax + bx + c − ln a + ax + bx + c +C
4a 8a2 2a
(3.43)
Esempi Z
√ √ √ 
M (x) = x2 + 2x − 3dx = x+1
2
x 2 + 2x − 3 − 2 ln 1 + x + x 2 + 2x − 3

+C Z

M (x) = 3x2 + x + 2dx
1
h √ √
1+6x √ i

2 2
= 72 6 (6x + 1) 3x + x + 2 + 23 3 ln 2 3 + 3x + x + 2 + C

Z

M (x) = 25x2 + 4x + 7dx
1
 √ √ 
= 250 5 (2 + 25x) 25x2 + 4x + 7 + 171 ln 52 + 5x + 25x2 + 4x + 7 + C
Consideriamo a < 0
L’integrale diventa (3.40)
p Z q
M (x) = |a| β 2 − (x + k)2 dx,

essendo
l ∆
β2 = = 2
|a| 4a
da ciò segue affinchè l’integrando sia reale, deve essere ∆ > 0. Eseguendo il
cambio di variabile (3.41):
p Z p
M (ξ) = |a| β 2 − ξ 2 dξ (3.44)

Integriamo per parti:


3.9 Integrali contenenti un trinomio di secondo grado 102

Z p Z
2 2
p
2 2
ξ2
β − ξ dξ = ξ β − ξ + p dξ
β2 − ξ2
Z
p
2 2
(β 2 − ξ 2 ) − β 2
=ξ β −ξ − p dξ
β2 − ξ2
Z p Z
p
2 dξ
= ξ β2 − ξ2 − β 2 − ξ 2 dξ + β p
β2 − ξ2
Z  
p
2 2
p
2 2 2 ξ
=ξ β −ξ − β − ξ dξ + β arcsin ,
β

da cui:
Z p 2
 
ξ p β ξ
β 2 − ξ 2 dξ = β2 − ξ2 + arcsin + C1
2 2 β
Sostituendo nella (3.44) e ripristinando la variabile x:
p  
2ax + b √ 2 ∆ |a| 2ax + b
M (x) = ax + bx + c − arcsin √ +C
4a 8a2 ∆
Esempi Z
√ x+1
√ 
1+x

M (x) = −x2
− 2x + 1dx = 2
−x2
− 2x + 1 + arcsin +C√
2
Z
√ √
M (x) = x − x2 dx = 2x−1
4
x − x2 + 18 arcsin (2x − 1) + C
Z
√ √ 
M (x) = 2 − x − x2 dx = 2x+1
4
2 − x − x 2 + 9 arcsin 2x+1 + C
8 3

***

Riassumendo:

Z
mx + n
dx (3.45)
ax2 + bx + c
   



2
+ √−∆ n− mb
arctan 2ax+b
2a

−∆
+ C, se ∆ < 0
m 2  √
= ln ax + bx + c +

+ √1∆ n − mb
2ax+b−√∆
ln 2ax+b+

+ C, se ∆ > 0
2a 
 2a ∆
2
se ∆ = 0

− 2ax+b + C,
3.10 Coppia di integrali notevoli 103

Z
mx + n
√ dx (3.46)
ax2 + bx + c
  2ax+b q
(2ax+b)2


 √1n−
a
mb
2a
ln √−∆ + 1 + |∆| + C, se a > 0, ∆ < 0




 2ax+b q 4a(ax2 +bx+c)
m√ 2  
mb √1
n − 2a ln √∆ +
+ C, se a > 0, ∆ > 0
= ax + bx + c + a ∆
a   2ax+b



mb √1
n − 2a a
ln 2a + C, se a > 0, ∆ = 0

   
− √1 n − mb arcsin 2ax+b se a < 0, ∆ > 0


2a


+ C,
|a|

Z
dx
√ (3.47)
(mx + n) ax2 + bx + c
si riconduce alla forma (3.46)
mediante la sostituzione
1
ξ=
mx + n

Z √
ax2 + bx + cdx (3.48)

∆ a
√ √ 2ax+b √
− 2 + bx + c + C, se a > 0
2 ln
a + ax
2ax + b √ 2 √

8a 2a
= ax + bx + c +  
4a  − ∆8a2|a| arcsin 2ax+b
√ + C, se a < 0

3.10 Coppia di integrali notevoli


Dai calcoli precedenti segue l’esistenza della coppia di integrali notevoli:

Z √
x√ 2 α √
(3.49)

x2 + αdξ = x + α + ln x + x2 + α + C
2 2
Z p  
2 2
x p
2 2
β2 x
β − x dξ = β −x + arcsin +C
2 2 β
3.11 Esercizi riepilogativi sugli integrali contenenti un trinomio di
secondo grado 104

3.11 Esercizi riepilogativi sugli integrali contenen-


ti un trinomio di secondo grado
3.11.1 Calcolare i seguenti integrali
Z Z Z

1) √dx
x x2 −1
11) √ x+2 dx
4x−x2
21) 4x2 − 4x + 5dx
Z Z Z
2) dx
2x2 +2x+5
12) √x+2 dx
x2 +9
22) √dx
x x2 −5
Z Z Z
3) x+1
x2 −4x+8
dx 13) 2x−3
4x2 −11
dx 23) 2x−3
x2 +6x+13
dx
Z Z Z
dy
4) y 2 +10y+30
14) √ x+2
x2 +2x−3
dx 24) x−1
3x2 −4x+3
dx
Z Z Z
5) √x+3 dx
1−x2
15) 2−x
4x2 +4x−3
dx 25) √ x
27+6x−x2
dx
Z Z Z

6) 2x−7
x2 +9
dx 16) 25 − x2 dx 26) √ 5−4x
12x−4x2 −8
dx
Z Z Z
√ √
7) √ dx
20+8x−x2
17) 3− 4x2 dx 27) 16 − 9x2 dx
Z Z Z
√ √
8) √ dx
28−12x−x2
18) x2 − 36dx 28) x2 − 16dx
Z Z Z
√ √
9) √ x+3
5−4x−x2
dx 19) 3x2 + 5dx 29) 4x2 + 9dx
Z Z Z
√ √
10) 2x+3
9x2 −12x+8
dx 20) 3 − 2x − x2 dx 30) x2 − 2x − 3dx

3.11.1.1 Soluzioni
Z
1. I (x) = x√dxx2 −1
. Eseguiamo il cambio di variabile:

1
x= ,
ξ

donde: Z

I (ξ) = − p = − arcsin ξ + C
1 − ξ2

Ripristinando la variabile x:
 
1
I (x) = − arcsin +C
x
Z

2. dx
2x2 +2x+5
= 13 arctan 2x+1
3
3.11 Esercizi riepilogativi sugli integrali contenenti un trinomio di
secondo grado 105

Z

3. x+1
x2 −4x+8
dx = 12 ln |x2 − 4x + 8| + 32 arctan x−2
2
+C
Z  
dy y+5
4. y 2 +10y+30
= √1
5
arctan √
5
+C
Z

5. √x+3 dx
1−x2
= − 1 − x2 + 3 arcsin x + C
Z

6. 2x−7
x2 +9
dx = ln (x2 + 9) − 73 arctan x
3
+C
Z

7. √ dx
20+8x−x2
= arcsin x−4
6
+C
Z

8. √ dx
28−12x−x2
= arcsin x+6
8
+C
Z
√ 
9. √ x+3
5−4x−x2
dx = − 5 − 4x − x2 + arcsin x+2
3
+C
Z

10. 2x+3
9x2 −12x+8
dx = 19 ln |9x2 − 12x + 8| + 13
18
arctan 3
2
x −1 +C
Z
 √
11. √ x+2 dx
4x−x2
= 4 arcsin 1
2
x − 1 − 4x − x2 + C
Z
√ √
12. √x+2 dx
x2 +9
= x2 + 9 + 2 ln x + x2 + 9 + C
Z √
13. 2x−3
= 14 ln |4x2 − 11| − √3 ln 2x−
√11
4x2 −11
dx 4 11 2x+ 11
+C
Z
√ √
14. √ x+2
x2 +2x−3
dx = ln 1 + x + x2 + 2x − 3 + x2 + 2x − 3 + C
Z
15. 2−x
4x2 +4x−3
dx =

= − 18 ln |4x2 + 4x − 3| + 5
16
ln 2x−1
2x+3
+C
= − 18 (ln |2x + 3| + ln |2x − 1|) + 5
16
(ln |2x − 1| − ln |2x + 3|) + C
7 3
= − 16 ln |2x + 3| + 16 ln |2x − 1| + C
Z
√ √
16. 25 − x2 dx = x2 25 − x2 + 25 2
arcsin x5 + C
Z
√ √  
17. 3 − 4x2 dx = x
2
3 − 4x2 + 43 arcsin 2x

3
+C
3.11 Esercizi riepilogativi sugli integrali contenenti un trinomio di
secondo grado 106

Z
√ √ √
18. x2 − 36dx = x
2
x2 − 36 − 18 ln x + x2 − 36 + C
Z
√ √ √ √ √
19. 3x2 + 5dx = x
2
3x2 + 5 + 5 3
6
ln 3x + 3x2 + 5 + C
Z
√ √ 
20. 3 − 2x − x2 dx = x+1
2
3 − 2x − x2 + 2 arcsin 1+x
2
+C
Z
√ √ √
21. 4x2 − 4x + 5dx = 2x−1
4
4x2 − 4x + 5+ln 2x − 1 + 4x2 − 4x + 5 +C
Z √ 
22. √dx
x x2 −5
= √1
5
arcsin x
5
+C
Z

23. 2x−3
x2 +6x+13
dx = ln |x2 + 6x + 13| − 29 arctan x+3
2
+C
Z √  
24. x−1
3x2 −4x+3
dx = 61 ln |3x2 − 4x + 3| − 5
15
arctan 3x−2

5
+C
Z
√ 
25. √ x
27+6x−x2
dx = − 27 + 6x − x2 + 3 arcsin x−3
6
+C
Z

26. √ 5−4x
12x−4x2 −8
dx = 21 arcsin (3 − 2x) + 2 3x − x2 − 2 + C
Z
√ √ 
27. 16 − 9x2 dx = x
2
16 − 9x2 + 38 arcsin 3
4
x +C
Z
√ √ √
28. x2 − 16dx = 21 x x2 − 16 − 8 ln x + x2 − 16 + C
Z
√ p √
29. 4x2 + 9dx = 21 x (4x2 + 9) + 94 ln 2x + 4x2 + 9 + C
Z
√ √ √
30. x2 − 2x − 3dx = x−1
2
x2 − 2x − 3 − 2 ln −1 + x + x2 − 2x − 3 + C

3.11.2 Calcolare i seguenti integrali


Z Z

1) 12 + 4x − x2 dx 5) √ dx
4−9x2
Z Z

2) x2 + 4xdx 6) sin 8x
9+sin4 4x
dx
Z Z

3) x2 − 8xdx 7) xdx
x4 −4x2 +3
Z Z

4) 6x − x2 dx 8) cos x
sin2 x−6 sin x+12
dx
3.11 Esercizi riepilogativi sugli integrali contenenti un trinomio di
secondo grado 107

3.11.3 Soluzioni
Z
√ √ 
1. 12 + 4x − x2 dx = x−2
2
12 + 4x − x2 + 8 arcsin x−2
4
+C
Z
√ √ √
2. x2 + 4xdx = x+2
2
x2 + 4x − 2 ln 2 + x + x2 + 4x + C
Z
√ √ √
3. x2 − 8xdx = x−4
2
x2 − 8x − 8 ln x − 4 + x2 − 8x + C
Z
√ √ 
4. 6x − x2 dx = x−3
2
6x − x2 + 29 arcsin −1 + 13 x + C
Z
5. √ dx
4−9x2
= 13 arcsin 32 x + C
Z Z
6. I (x) = sin 8x
9+sin4 4x
dx =2 sin 4x cos 4x
9+sin4 4x
dx. Eseguiamo il cambio di variabile:

ξ = sin 4x,

donde:
Z
1 ξdξ
I (ξ) =
2 9 + (ξ 2 )2
Z
1 d (ξ 2 )
=
4 9 + (ξ 2 )2
Z
1 d (ξ 2 )
=  2 2
36
1 + ξ3
 2
1 ξ
= arctan +C
12 3

Ripristinando la variabile x:
 
1 sin2 4x
I (x) = arctan +C
12 3
Z
7. I (x) = xdx
x4 −4x2 +3
. Eseguiamo il cambio di variabile:

ξ = x2 ,
3.12 Integrazione delle funzioni razionali 108

donde:
Z
1 dξ
I (ξ) = 2
2 ξ − 4ξ + 3

1 ξ − 3
= ln +C
4 ξ − 1

Ripristinando la variabile x:

1 x2 − 3
I (x) = ln 2 +C
4 x − 1
Z
8. I (x) = cos x
sin2 x−6 sin x+12
dx. Eseguiamo il cambio di variabile:

ξ = sin x,

donde:
Z
1 dξ
I (ξ) =
2 ξ 2 − 6ξ + 12
 
1 ξ−3
= √ arctan √ +C
3 3
Ripristinando la variabile x:

1 sin x − 3
I (x) = √ arctan √ +C
3 3

3.12 Integrazione delle funzioni razionali


Consideriamo l’integrale indefinito:
Z
f (x) dx, (3.50)

essendo:

p (x)
,
f (x) = (3.51)
q (x)
con p (x), q (x) polinomi in x di grado m e n rispettivamente:
m
X n
X
p (x) = k
ak x , q (x) = bk xk (3.52)
k=0 k=0

Definizione. La funzione f (x) espressa dalla (3.51) è una funzione razionale.


Più precisamente, è una funzione razionale propria se m < n; una funzione
razionale impropria se m ≥ n.
3.12 Integrazione delle funzioni razionali 109

3.12.1 Funzioni razionali proprie


Sotto ragionevoli ipotesi, la funzione (3.51) ammette una decomposizione in
frazioni parziali, cioè somme di fattori del tipo:
Ai Ai,j Ax + B Ai x + Bi
, νj , ,
x − ξi (x − ξi ) ax + bx + c (ax2 + bx + c)i
2

Ad esempio, se q (x) ha r radici reali distinte:

ξ1 , ξ2 , ..., ξr
Se νk la molteciplità di ξk :
r
X
νk = n,
k=1

donde:
r
Y
q (x) = (x − ξk )νk (3.53)
k=1

In tal caso la decomposizione in frazioni parziali è


νi
r X
X Ai,j
f (x) = j, (3.54)
i=1 j=1
(x − ξ i )
essendo Ai,j coefficienti indeterminati. Esplicitando il secondo membro della
(3.54):

A1,1 A1,2 A1,ν1


f (x) = + 2 + ... + (3.55)
x − ξ1 (x − ξ1 ) (x − ξ1 )ν1
A2,1 A2,2 A2,ν2
+ + 2 + ... +
x − ξ2 (x − ξ2 ) (x − ξ1 )ν2
Ar,1 Ar,2 Ar,νr
... + + 2 + ... +
x − ξr (x − ξr ) (x − ξr )νr

Nel caso speciale r = n (cioè q (x) ammette n radici reali e distinte ciascuna di
molteplicità 1), la decomposizione (3.54) è:
n
X Ai A1 A2 An
f (x) = = + + ... + (3.56)
i=1
x − ξi x − ξ1 x − ξ2 x − ξn
La (3.56) esprime una decomposizione in fattori lineari distinti.
3.12 Integrazione delle funzioni razionali 110

Esempio 12.
1 1
f (x) = =
x2
−4 (x − 2) (x + 2)
A1 A1
= +
x−2 x−2
(A1 + A2 ) x + 2 (A1 − A2 )
=
x2 − 4
Per il principio di identità dei polinomi:

A1 + A2 = 0 (3.57)
2 (A1 − A2 ) = 1

La soluzione del sistema (3.57) è:


 
1 1
(A1 , A2 ) = ,− ,
4 4

donde la demposizione in fattori lineari distinti è:


1 1
f (x) = −
4 (x − 2) 4 (x + 2)

Nel caso generale r ∈ {1, 2, ...n − 1}, si applica la (3.54) che esprime una decom-
posizione in fattori lineari ripetuti.

Esempio 13.
p (x) = x, q (x) = x3 + x2 − x − 1
Cerchiamo le radici reali di q (x):

q (x) = 0 ⇐⇒ x = 1, −1

Precisamente:

ξ1 = 1, (ν1 = 1)
ξ2 = −1, (ν2 = 2) ,

per cui:
q (x) = (x − 1) (x + 1)2
La decomposizione si scrive:
x A1,1 A2,1 A2,2
2 = + +
(x − 1) (x + 1) x − 1 x + 1 (x + 1)2
3.12 Integrazione delle funzioni razionali 111

Per non appesantire la notazione, ridefiniamo i coefficienti della decomposizione:


x A B1 B2
2 = + +
(x − 1) (x + 1) x − 1 x + 1 (x + 1)2

Abbiamo:
(A + B1 ) x2 + (2A + B2 − 1) x + A − B1 − B2 = 0
da cui il sistema lineare:

A + B1 + 0 = 0
2A + 0 + B2 = 1
A − B1 − B2 = 0

che è compatibile e determinato:


 
1 1 1
(A, B1 , B2 ) = ,− ,
4 4 2
Perciò:
x 1 1 1
2 = − +
(x − 1) (x + 1) 4 (x − 1) 4 (x + 1) 2 (x + 1)2
Quindi l’integrale:
Z
x 1 x − 1 1
2 = ln − +C
(x − 1) (x + 1) 4 x+1 2 (x + 1)

Esempio 14. Z
p (x)
I (x) = dx,
q (x)
essendo:
p (x) = 1, q (x) = x3 − 2x2 + x = x (x − 1)2 ,
donde:
p (x) A B1 B2
= + +
q (x) x x − 1 (x − 1)2
Ciò implica:

A + B1 + 0 = 0
2A + B1 − B2 = 0
A+0+0=1

La soluzione è
(A, B1 , B2 ) = (1, −1, 1)
x
Quindi I (x) = ln x−1 − 1
x−1
+C
3.12 Integrazione delle funzioni razionali 112

Se q (x) contiene dei fattori irriducibili ax2 + bx + c, a ciascuno di essi corrisponde


una frazione parziale propria:
Ax + B
,
ax2 + bx + c
con A, B coefficienti indeterminati. In tal caso si ha una decomposizione in
fattori quadratici distinti.

Esempio 15. Sia:


x3 + x2 + x + 2
f (x) =
x4 + 3x2 + 2
La decomposizione è:
Ax + B Cx + D
f (x) = + 2
x2 + 2 x +1
Deve essere:

A+0+C +0=1
0+B+0+D =1
A + 0 + 2C + 0 = 1
0 + B + 0 + 2D = 2

Tale sistema lineare è compatibile e determinato:

(A, B, C, D) = (1, 0, 0, 1)

Quindi:
x 1
f (x) = + 2
x2 +1 x +1
L’integrale vale:
Z
1 
f (x) dx = ln x2 + 1 + arctan x + C
2
β
Se q (x) contiene dei fattori irriducibili (ax2 + bx + c) , a ciascuno di essi cor-
risponde una somma di β frazioni parziali proprie:
β
X Ai x + Bi
,
i=1
(ax2 + bx + c)i

con Ai , Bi coefficienti indeterminati. In tal caso si ha una decomposizione in


fattori quadratici ripetuti.
3.12 Integrazione delle funzioni razionali 113

Esempio 16. Sia:


2x2 + 3
f (x) =
(x2 + 1)2
Abbiamo:
Ax + B Cx + D
f (x) = +
2
x +1 (x2 + 1)2
Otteniamo il sistema di equazioni lineari:
A+0+0+0=0
0+B+0+0=2
A+0+C +0=0
0 + B + 0 + D = 3,
che è compatibile e determinato, con soluzione:
(A, B, C, D) = (0, 2, 0, 1)
donde:
2 1
f (x) = +
x2 + 1 (x2 + 1)2
Ciò implica che l’integrale è:
Z Z
dx
I (x) = f (x) dx = 2 arctan x +
(x2 + 1)2
Calcoliamo l’integrale a secondo membro attraverso il cambio di variabile:
x = tan ξ,
per cui: Z Z
dx 1
2 = cos2 ξdξ = (ξ + sin ξ cos ξ) + C
(x2 + 1) 2
Per ripristinare la variabile x è necessario esplicitare:
sin (arctan x) , cos (arctan ξ)
A tale scopo osserviamo che:
r
1
cos x = 2
,
tan x + 1
quindi:
1
cos (arctan ξ) = √ (3.58)
x2 + 1
p
sin (arctan x) = 1 − cos2 (arctan x)
x
=√
x2 + 1
3.12 Integrazione delle funzioni razionali 114

Dalle (3.58) otteniamo:


x
sin ξ cos ξ = ,
x2 +1
quindi l’integrale:
Z
dx 1 x
2 = arctan x + 2
+ C1 (3.59)
(x2 + 1) 2 2 (x + 1)

Finalmente: Z
5 x
f (x) dx = arctan x + 2
+C
2 2 (x + 1)
Esempio 17. Sia:

x5 − x4 + 4x3 − 4x2 + 8x − 4
f (x) =
(x2 + 2)3

Abbiamo:
Ax + B Cx + D Ex + F
f (x) = + 2 +
2
x +2 (x2 + 2) (x2 + 2)3
Eseguendo i calcoli:

A+0+0+0+0+0=1
0 + B + 0 + 0 + 0 + 0 = −1
4A + 0 + C + 0 + 0 + 0 = 4
0 + 4B + 0 + D + 0 + 0 = −4
4A + 0 + 2C + 0 + E + 0 = 8
0 + 4B + 0 + 2D + 0 + F = −4,

la cui unica soluzione è:

(A, B, C, D, E, F ) = (1, −1, 0, 0, 4, 0)

Quindi:
x−1 4x
f (x) = +
x + 2 (x2 + 2)3
2

L’integrale:
Z Z Z Z
x dx xdx
f (x) dx = dx − +4
2
x +2 2
x +2 (x2 + 2)3
 
1 2
 1 x 1
= ln x + 2 − √ arctan √ − +C
2 2 2 (x + 2)2
2

La tabella seguente riassume i vari casi:


3.12 Integrazione delle funzioni razionali 115

Fattore Frazioni parziali N


A
ax + b ax+b
1
(ax + b)α A
(ax+b)i
, i= 1, 2, ..., α α
Ax+B
ax2 + bx + c ax2 +bx+c
1
β
(ax2 + bx + c) Ax+B
(ax2 +bx+c)i
, i = 1, 2, ..., β β

Se il polinomio q (x) ha radici reali e complesse multiple, per il calcolo dell’inte-


grale (3.50) si applica la seguente formula di riduzione:
Z Z
p (x) X (x) Y (x)
dx = + dx (3.60)
q (x) q1 (x) q2 (x)
Nella (3.60) q1 (x) è il massimo comune denominatore di q (x) e q ′ (x) = dx
d
q (x),
mentre X (x), Y (x) sono polinomi a coefficienti indeterminati di grado r − 1,
essendo r il grado di q1 (x). Il polinomio q2 (x) è:

q (x)
q2 (x) =
q1 (x)
2
Esempio 18. p (x) = 1, q (x) = (x3 − 1) . Abbiamo q ′ (x) = 6x2 (x3 − 1), per
cui q1 (x) = q2 (x) = x3 − 1. Per la (3.60)
Z Z
dx Ax2 + Bx + C Dx2 + Ex + F
= +
(x3 − 1)2 x3 − 1 x3 − 1
Derivando primo e secondo membro:
1 (2Ax + B) (x3 − 1) − 3x2 (Ax2 + Bx + C) Dx2 + Ex + F
= +
(x3 − 1)2 (x3 − 1)2 x3 − 1
Dx5 + (E − A) x4 + (F − 2B) x3 + (−D − 3C) x2 + (−E − 2A) x − F − B
=
(x3 − 1)2
Da cui:

0+0+0+D+0+0=0 (3.61)
−A + 0 + 0 + 0 + E + 0 = 0
0 − 2B + 0 + 0 + 0 + F = 0
0 + 0 − 3C − D + 0 + 0 = 0
−2A + 0 + 0 + 0 − E + 0 = 0
0−B+0+0+0−F =1

Dalla prima e dalla quarta equazione del sistema (3.61):

D = C = 0,
3.12 Integrazione delle funzioni razionali 116

donde otteniamo il sistema:

−A + 0 + E + 0 = 0 (3.62)
0 − 2B + 0 + F = 0
−2A + 0 − E + 0 = 0
0−B+0−F =1

Dalla prima e dalla terza:


A=E=0
Quindi:
1 2
B=− ,F =−
3 3
Riassumendo:
1 2
A = 0, B = − , C = 0, D = 0, E = 0, F = −
3 3
donde: Z
1 x 2 dx
I (x) = − 3 −
3x −1 3 x3−1
Poniamo:
Z
def dx
J (x) = 3
x −1
Z
def p̃ (x)
= dx
q̃ (x)

Dove: 
p̃ (x) = 1, q̃ (x) = x3 − 1 = (x − 1) x2 + x + 1
cioè q̃ (x) ha radici complesse. In tal caso la riduzione è:
Z
dx L Mx + N
2
= + 2
(x − 1) (x + x + 1) x−1 x +x+1

Da cui:  
1 1 2
(L, M, N ) = ,− ,− ,
3 3 3
cioè: Z
1 1 x+2
J (x) = ln |x − 1| − dx
3 3 x2 +x+1
L’integrale a secondo membro si calcola con la (3.45):
Z  
x+2 1 2
 √ 2x + 1
dx = ln x + x + 1 + 3 arctan √ + C2
x2 + x + 1 2 3
3.12 Integrazione delle funzioni razionali 117

Quindi l’integrale J (x):


√  
1 1 2
 3 2x + 1
J (x) = ln |x − 1| − ln x + x + 1 − arctan √ + C1
3 6 3 3
L’integrale I (x):
 2  √  
x 1 x +x+1 2 3 2x + 1
I (x) = + ln + arctan √ +C
3 (1 − x3 ) 9 (x − 1)2 9 3
Z
dx
3.12.2 (x2 +1)
2

Dai calcoli precedenti segue un integrale che compare spesso:


Z  
dx 1 x
= + arctan x + C (3.63)
(x2 + 1)2 2 x2 + 1
***

3.12.3 Calcolare i seguenti integrali


Z Z Z
5x2 +6x+9
1) dx
(x+a)(x+b)
, (a 6= b) 5) (x−3)2 (x+1)2
dx 9) dx
(x2 −4x+3)(x2 +4x+5)
Z Z Z
x2 −8x+7
2) dx
(x−1)(x+2)(x+3)
6) (x2 −3x−10)2
dx 10) dx
x3 +1
Z Z Z
2x2 +41x−91
3) (x−1)(x+3)(x−4)
dx 7) 2x−3
(x2 −3x+2)3
dx 11) dx
x4 +1
Z Z Z
x5 −7x+1
4) dx
x(x+1)2
8) (x2 −3x+2)3
dx 12) dx
x4 +x2 +1

3.12.4 Soluzioni
Z
1. I (x) = dx
(x+a)(x+b)
. Abbiamo:

1 A B
= +
(x + a) (x + b) x+a x+b
(A + B) x + (Ab + Ba)
=
(x + a) (x + b)
da cui:  
1 1
(A, B) = ,−
b−a b−a
Quindi:
1 x + a
I (x) = ln +C
b − a x + b
3.12 Integrazione delle funzioni razionali 118

Z
2. I (x) = dx
(x−1)(x+2)(x+3)
. Abbiamo:

1 A B C
= + +
(x − 1) (x + 2) (x + 3) x−1 x+2 x+3

Quindi il sistema di equazioni lineari:

A+B+C =0
5A + 2B − C = 0
6A − 3B − 2C = 1,

che è compatibile e determinato:


 
1 1 1
(A, B, C) = ,− ,
12 3 4

Quindi:
1 1 1
I (x) = ln |x − 1| − ln |x + 2| + ln |x + 3| + C
12 3 4
Z
2x2 +41x−91
3. I (x) = (x−1)(x+3)(x−4)
dx. Abbiamo:

2x2 + 41x − 91 A B C
= + +
(x − 1) (x + 3) (x − 4) x−1 x+2 x+3

Quindi il sistema di equazioni lineari:

A+B+C =2
−A − 5B + 2C = 41
−12A + 4B − 3C = −91,

che è compatibile e determinato:

(A, B, C) = (4, −7, 5)

Quindi:
I (x) = 4 ln |x − 1| − 7 ln |x + 3| + 5 ln |x − 4| + C
3.12 Integrazione delle funzioni razionali 119

Z
4. I (x) = dx
x(x+1)2
. Qui è:

p (x) ≡ 1, q (x) ≡ x (x + 1)

La (3.54) porge:
1 A1,1 A2,1 A2,2
= + +
x (x + 1) x x + 1 (x + 1)2
def A B1 B2
= + +
x x + 1 (x + 1)2

Otteniamo il sistema di equazioni lineari:

A + B1 + 0 = 0
2A + B1 + B2 = 0
A + 0 + 0 = 1,

la cui soluzione è:
(A, B1 , B2 ) = (1, −1, −1)

Quindi:
1 x
I (x) = + ln
+C
x+1 x + 1
Z
5x2 +6x+9
5. I (x) = (x−3)2 (x+1)2
dx. Qui è:

p (x) ≡ 5x2 + 6x + 9, q (x) ≡ (x − 3)2 (x + 1)2

La (3.54) porge:

p (x) A1,1 A1,2 A2,1 A2,2


= + 2 + +
q (x) x − 3 (x − 3) x + 1 (x + 1)2
def A B C D
= + 2 + +
x − 3 (x − 3) x + 1 (x + 1)2

Otteniamo il sistema di equazioni lineari:

A+0+C +0=0
−A + B − 5C + D = 5
−5A + 2B + 3C − 6D = 6
−3A + B + 9C + 9D = 9,
3.12 Integrazione delle funzioni razionali 120

Tale sistema è compatibile e determinato, con soluzione


 
9 1
(A, B, C, D) = 0, , 0,
2 2

Quindi:
5x + 3
I (x) = − +K
(x − 3) (x + 1)
Z
x2 −8x+7
6. I (x) = (x2 −3x−10)2
dx. Qui è:

2
p (x) ≡ x2 − 8x + 7, q (x) ≡ x2 − 3x − 10

Le radici di q (x) = 0 e le relative moltepicità sono:

ξ1 = 5, (ν1 = 2)
ξ2 = −2, (ν2 = 2)

donde:
q (x) = (x − 5)2 (x + 2)2

La (3.54) porge:
2 i ν
p (x) XX Ai,j
=
q (x) i=1 j=1
(x − ξi )j
2  
X Ai,1 Ai,2 Ai,νi
= + 2 + ... +
i=1
x − ξ i (x − ξ i ) (x − ξi )νi
A1,1 A1,2 A2,1 A2,2
= + 2 + +
x − ξ1 (x − ξ1 ) x − ξ2 (x − ξ2 )2

Per non appesantire la notazione, ridefiniamo:


def
(A1,1 , A1,2 , A2,1 , A2,2 ) = (A, B, C, D)

per cui:

p (x)
=
q (x)
A (x − 5) (x + 2)2 + B (x + 2)2 + C (x − 5)2 (x + 2) + D (x − 5)2
=
(x − 5)2 (x + 2)2
3.12 Integrazione delle funzioni razionali 121

cioè:
(A + C) x3 + (−A + B − 8C + D) x2 + (−16A + 4B + 5C − 10D) x
+ (−20A + 4B + 50C + 25D)
= x2 − 8x + 7

Per il principio di identità dei polinomi deve essere:


A+0+C +0=0
−A + B − 8C + D = 1
−16A + 4B + 5C − 10D = −8
−20A + 4B + 50C + 25D = 7

Tale sistema è compatibile e determinato. La soluzione è:


 
30 8 30 27
(A, B, C, D) = ,− ,− ,−
343 49 343 49

Quindi:
   
p (x) 30 1 1 1 8 1
= − − +
q (x) 343 x−5 x+2 49 (x − 5)2 (x + 2)2

L’integrale:

30 x − 5 19x − 151
I (x) = ln − +K
343 x+2 49 (x2 − 3x − 10)
Z
7. I (x) = 2x−3
(x2 −3x+2)3
dx. L’integrazione è immediata:
Z
1 d (x2 − 3x + 2)
I (x) = dx
2 (x2 − 3x + 2)3
1
=− +K
2 (x2 − 3x + 2)2
Z
x5 −7x+1
8. I (x) = (x2 −3x+2)3
dx. Qui è:
3
p (x) ≡ x5 − 7x + 1, q (x) ≡ x2 − 3x + 2

Le radici di q (x) = 0 e le relative moltepicità sono:


ξ1 = 2, (ν1 = 3)
ξ2 = 1, (ν2 = 3)
3.12 Integrazione delle funzioni razionali 122

donde:
q (x) = (x − 2)3 (x − 1)3

La (3.54) porge:
2
i ν
p (x) XX Ai,j
=
q (x) i=1 j=1
(x − ξi )j
2  
X Ai,1 Ai,2 Ai,νi
= + + ... +
i=1
x − ξi (x − ξi )2 (x − ξi )νi
A1,1 A1,2 A1,3 A2,1 A2,2 A2,3
= + 2 + 3 + + 2 +
x − ξ1 (x − ξ1 ) (x − ξ1 ) x − ξ2 (x − ξ2 ) (x − ξ2 )3

Per non appesantire la notazione, ridefiniamo:


def
(A1,1 , A1,2 , A1,3 , A2,1 , A2,2 , A2,3 ) = (A, B, C, D, E, F )

per cui:
p (x)
=
q (x)
1
= · {A (x − 2)2 (x − 2)3 + B (x − 2) (x − 1)3 + C (x − 1)3
q (x)
+ D (x − 1)2 (x − 2)3 + E (x − 1) (x − 2)3 + F (x − 2)3 }

Da cui:

p (x) 19 16 25 5 17 26
= 3 + 2 − + 3 + 2 +
q (x) (x − 2) (x − 2) x − 2 (x − 1) (x − 1) x−1

L’integrale:
9 − 14x 38 − 33x
I (x) = 2 + 2 − 25 ln |−2 + x| + 26 ln |x − 1| + K
2 (x2 − 3x + 2) x − 3x + 2
Z
9. I (x) = 1
(x2 −4x+3)(x2 +4x+5)
. Qui è:
 
p (x) ≡ 1, q (x) ≡ x2 − 4x + 3 x2 + 4x + 5

Le radici reali di q (x) sono:

ξ1 = 3, ξ2 = 1
3.12 Integrazione delle funzioni razionali 123

con molteplicità:
ν1 = ν2 = 1

Quindi:
p (x) A B Cx + D
= + + 2 , (3.64)
q (x) x − 3 x − 1 x + 4x + 5
giacché x2 + 4x + 5 ha radici complesse coniugate. La (3.64) implica

A+B+C +0=0
3A + B − 4C + D = 0
A − 7B + 3C − 4D = 0
−5A − 15B + +0 + 3d = 1,

la cui unica soluzione è:


 
1 1 2 3
(A, B, C, D) = ,− , ,
52 20 65 26

onde:
1 1
I (x) = ln |x − 3| − ln |x − 1| + J (x) ,
52 20
essendo: Z
1 4x + 15
J (x) = ,
130 x2 + 4x + 5
che si calcola attraverso la (3.45), ottenendo:
1 2 7
J (x) = ln x + 4x + 5 + arctan (x + 2) + K1
65 130

Finalmente:
1 1 1 7
I (x) = ln |x − 3|− ln |x − 1|+ ln x2 + 4x + 5 + arctan (x + 2)
52 20 65 130
Z
10. I (x) = x3dx+1 . Qui è:

p (x) ≡ 1, q (x) ≡ x3 + 1

Risulta: 
q (x) = (x + 1) x2 − x + 1 ,

donde:
p (x) A Bx + C
= + 2 ,
q (x) x+1 x −x+1
3.12 Integrazione delle funzioni razionali 124

che conduce al sistema:

A+B+0=0
−A + B + C = 0
A+0+C =1

compatibile e determinato:
 
1 1 2
(A, B, C) = ,− ,
3 3 3

cioè
p (x) 1 1 1 x−2
= −
q (x) 3 x + 1 3 x2 − x + 1
Quindi l’integrale:
1 1
I (x) = ln |x + 1| − J (x) ,
3 3
essendo: Z
x−2
J (x) = dx,
x2 −x+1
che si calcola attraverso la (3.45), ottenendo:
 
1  √ 2x − 1
J (x) = ln x2 − x + 1 − 3 arctan √ + K1
2 3

Finalmente:
 
1 1  1 2x − 1
I (x) = ln |x + 1| − ln x2 − x + 1 + √ arctan √ +K
3 6 3 3
Z
11. I (x) = dx
x4 +1
. Qui è:

2

4

2
√ 
p (x) = 1, q (x) = x + 1 = x + 2x + 1 x − 2x + 1

Quindi:
√  √ 
p (x) (Ax + B) x2 − 2x + 1 + (Cx + D) x2 + 2x + 1
=
q (x) q (x)
3.12 Integrazione delle funzioni razionali 125

da cui il sistema:

A+0+C +0=0
√ √
− 2A + B + 2C + D = 0
√ √
A − 2B + C + 2D = 0
0+B+0+D =1

compatible e determinato
 
1 1 1 1
(A, B, C, D) = √ , ,− √ ,
2 2 2 2 2 2

Quindi l’integrale:
1
I (x) = √ [J1 (x) − J2 (x)] ,
2 2

essendo:
Z √ Z √
x+ 2 x− 2
J1 (x) = √ dx, J2 (x) = √ dx
x2 + 2x + 1 x2 − 2x + 1

che si calcolano attraverso la (3.45), ottenendo:


1  2 √  √ 
J1 (x) = ln x + 2x + 1 + arctan 2x + 1 + C1
2
1 2 √

√ 
J2 (x) = ln x − 2x + 1 − arctan 2x − 1 + C2
2

Finalmente:
" √ √ √
#
1 x2 + 2x + 1  
I (x) = √ ln √ + 2 arctan 2x + 1 − 2 arctan 2x − 1
4 2 x2 − 2x + 1
(3.65)
Z
12. I (x) = x4 +x
dx
2 +1 . Qui è:

 
p (x) = 1, q (x) = x4 + x2 + 1 = x2 + x + 1 x2 − x + 1

Quindi:

p (x) (Ax + B) (x2 − x + 1) + (Cx + D) (x2 + x + 1)


=
q (x) q (x)
3.12 Integrazione delle funzioni razionali 126

da cui il sistema:

A+0+C +0=0
−A + B + C + D = 0
A−B+C +D =0
0+B+0+D =1

compatible e determinato
 
1 1 1 1
(A, B, C, D) = , ,− ,
2 2 2 2

Quindi l’integrale:
1
I (x) = [J1 (x) − J2 (x)] ,
2
essendo:
Z √ Z √
x+ 2 x− 2
J1 (x) = √ dx, J2 (x) = √ dx
x2 + 2x + 1 x2 − 2x + 1

che si calcolano attraverso la (3.45), ottenendo:


 
1 2
 1 2x + 1
J1 (x) = ln x + x + 1 + √ arctan √ + K1
2 3 3
 
1 2
 1 2x − 1
J2 (x) = ln x − x + 1 − √ arctan √ + K2
2 3 3

Finalmente:
" r  #
1 x2 + x + 1 1 2x + 1 2x − 1
I (x) = ln + √ arctan √ + arctan √ +K
2 x2 − x + 1 3 3 3

***

3.12.5 Calcolare i seguenti integrali


Z Z Z
x2 +1
1) x+1
x2 −5x+6
dx 5) x+1
x2 −3x+2
dx 9) x3 −4x2 +5x−2
dx
Z Z Z
x2 +1 x2 2x2 +1
2) x3 −x2 −x+1
dx 6) (x+2)(x−1)2
dx 10) x3 −2x2 +x−2
dx
Z Z Z
(x+2)2 x2 +2
3) x3 −1
dx 7) (x−1)3
dx 11) x+1
x3 −1
dx
Z Z Z
x3 +2x2 +1 x2 −1 x2 −2x
4) x5 −x4 +2x3 −2x2 +x−1
dx 8) (x−2)(x2 +1)
dx 12) (2x−1)(x2 +1)
dx
3.12 Integrazione delle funzioni razionali 127

3.12.6 Soluzioni
Z
1. I (x) = x+1
x2 −5x+6
dx. Qui è:

p (x) = 1, q (x) = x2 − 5x + 6 = (x − 3) (x − 2)

Quindi:
p (x) A B
= +
q (x) x−3 x−2
da cui
(A, B) = (4, −3)

L’integrale:
I (x) = 4 ln |x − 3| − 3 ln |x − 2| + C
Z
x2 +1
2. I (x) = x3 −x2 −x+1
dx. Qui è:

p (x) = x2 + 1, q (x) = x3 − x2 − x + 1 = (x − 1)2 (x + 1)

Le radici di q (x) = 0 e le relative moltepicità sono:

ξ1 = 1, (ν1 = 2)
ξ2 = −1, (ν2 = 1)

La (3.54) porge:
2i ν
p (x) XX Ai,j
=
q (x) i=1 j=1
(x − ξi )j
2  
X Ai,1 Ai,2 Ai,νi
= + + ... +
i=1
x − ξi (x − ξi )2 (x − ξi )νi
A1,1 A1,2 A2,1
= + 2 +
x − ξ1 (x − ξ1 ) x − ξ2

Per non appesantire la notazione, ridefiniamo:


def
(A1,1 , A1,2 , A2,1 ) = (A, B, C)

per cui:
p (x) A B C
= + 2 +
q (x) x − 1 (x − 1) x+1
3.12 Integrazione delle funzioni razionali 128

che conduce al sistema

A+0+C =1
0 + B − 2C = 0
−A + B + C = 1,

compatibile e determinato:
 
1 1
(A, B, C) = , 1,
2 2

L’integrale:
p 1
I (x) = ln |x2 − 1| − +K
x−1
Z
(x+2)2
3. I (x) = x3 −1
dx. Qui è:

p (x) = x2 + 4x + 4, q (x) = (x − 1) x2 + x + 1

Quindi:
p (x) A Bx + C
= + 2
q (x) x−1 x +x+1
da cui

A+B+0=1
A−B+C =4
A+0−C =4

La soluzione è
(A, B, C) = (3, −2, −1)

Perciò:
p (x) 3 2x + 1
= − 2
q (x) x−1 x +x+1
d
3 (x2 + x + 1)
= − dx 2
x−1 x +x+1

Quindi:

I (x) = 3 ln |x − 1| − ln x2 + x + 1 + K
!
|x − 1|3
= ln +K
x2 + x + 1
3.12 Integrazione delle funzioni razionali 129

Z
x3 +2x2 +1
4. I (x) = x5 −x4 +2x3 −2x2 +x−1
dx. Qui è:

 2
p (x) = x3 +2x2 +1, q (x) = x5 −x4 +2x3 −2x2 +x−1 = x2 − 1 x2 + 1

Quindi:
p (x) A Bx + C Dx + E
= + 2 +
q (x) x−1 x +1 (x2 + 1)2
da cui il sistema:

A+B+0+0+0=0
0−B+C +0+0=1
2A + B − C + D + 0 = 2
0−B−C −D+E =0
A+0−C +0−E =1

compatibile e determinato

(A, B, C, D + E) = (1, −1, 0, 1, 0)

Ciò implica:
p (x) 1 x x
= − 2 +
q (x) x − 1 x + 1 (x2 + 1)2

Quindi l’integrale:
1  1
I (x) = ln |x − 1| − ln x2 + 1 − +K
2 2 (x2 + 1)
Z
5. I (x) = x+1
x2 −3x+2
dx. Qui è:

p (x) = x + 1, q (x) = x2 − 3x + 2 = (x − 2) (x − 1)

Quindi:
p (x) A B
= +
q (x) x−2 x−1
Affinché ciò sia vero, deve essere:

A = 3, B = −2

Ciò implica:
I (x) = 3 ln |x − 2| − 2 ln |x − 1| + K
3.12 Integrazione delle funzioni razionali 130

Z
x2
6. I (x) = (x+2)(x−1)2
dx. Qui è:

p (x) = x2 , q (x) = (x + 2) (x − 1)2

Quindi:
p (x) A B1 B2
= + +
q (x) x + 2 x − 1 (x − 1)2
Affinché ciò sia vero, deve essere:
A + B1 + 0 = 1
−2A + B1 + B2 = 0
A − 2B1 + 2B2 = 0

L’unica soluzione di tale sistema è


4 5 1
A= , B= , C=
9 9 3
Ciò implica:
4 5 1
I (x) = ln |x + 2| + ln |x − 1| − +C
9 9 3 (x − 1)
Z
x2 +2
7. I (x) = (x−1)3
dx. Qui è:

p (x) = x2 + 2, q (x) = (x − 1)3

Quindi:
p (x) A B C
= + 2 +
q (x) x − 1 (x − 1) (x − 1)3
Affinché ciò sia vero, deve essere:
A+0+0=1
−2A + B + 0 = 0
A−B+C =2

L’unica soluzione di tale sistema è


A = 1, B = 2, C = 3

Ciò implica:
2 3
I (x) = ln |x − 1| − − +K
x − 1 2 (x − 1)2
3.12 Integrazione delle funzioni razionali 131

Z
x2 −1
8. I (x) = (x−2)(x2 +1)
dx. Qui è:

p (x) = x2 − 1, q (x) = (x − 2) x2 + 1

Quindi:
p (x) A Bx + C
= + 2
q (x) x−2 x +1
Affinché ciò sia vero, deve essere:
A+B+0=1
0 − 2B + C = 0
A + 0 − 2C = −1

L’unica soluzione di tale sistema è


 
3 2 4
(A, B, C) = , ,
5 5 5

Ciò implica:
3 1  4
I (x) = − ln |x − 2| + ln x2 + 1 + arctanx + C
5 5 5
Z
x2 +1
9. I (x) = x3 −4x2 +5x−2
dx. Qui è:

p (x) = x2 + 1, q (x) = x3 − 4x2 + 5x − 2 = (x − 1)2 (x − 2)

Quindi:
p (x) A B C
= + +
q (x) x − 2 x − 1 (x − 1)2
Affinché ciò sia vero, deve essere:
A+B+0=1
−2A − 3B + C = 0
A + 2B − 2C = 1

L’unica soluzione di tale sistema è


(A, B, C) = (5, −4, −2)

Ciò implica:
2
I (x) = 5 ln |x − 2| + − 4 ln |x − 1| + K
x−1
3.12 Integrazione delle funzioni razionali 132

Z
2x2 +1
10. I (x) = x3 −2x2 +x−2
dx. Qui è:

p (x) = 2x2 − 1, q (x) = x3 − 2x2 + x − 2 = (x − 2) x2 + 1

Quindi:
p (x) A Bx + C
= + 2
q (x) x−2 x +1
da cui

A+B+0=2
0 − 2B + C = 0
A + 0 − 2C = 1

La soluzione è  
9 1 2
(A, B, C) = , ,
5 5 5
Perciò:
p (x) 9 1 x+2
= + · 2
q (x) 5 (x − 2) 5 x + 1
L’integrale:
9 1
I (x) = ln |x − 2| + J (x) ,
5 5
essendo:
Z
x+2
J (x) = dx
x2 + 1
Z Z
x dx
= 2
dx + 2 2
x +1 x +1
1 2
= ln x + 1 + 2 arctan x + C1 ,
2

donde:
9 1  2
I (x) = ln |x − 2| + ln x2 + 1 + arctan x + C
5 10 5
Z
11. I (x) = x+1
x3 −1
dx. Qui è:

p (x) = x + 1, q (x) = x3 − 1 = (x − 1) x2 + x + 1
3.12 Integrazione delle funzioni razionali 133

Quindi:
p (x) A Bx + C
= + 2
q (x) x−1 x +x+1
da cui

A+B+0=0
A−B+C =0
A+0−C =1

La soluzione è
(A, B, C) = (1, −1, 0)
Perciò:
p (x) 1 x
= − 2
q (x) x−1 x +x+1
L’integrale:
I (x) = ln |x − 1| − J (x) ,

essendo:
Z
x
J (x) = dx
x2 +x+1
 
1  1 2x + 1
= ln x2 + x + 1 − √ arctan √ + K1 ,
2 3 3

Si conclude che:
 
|x − 1| 1 2x + 1
I (x) = ln √ + √ arctan √ +K
x2 + x + 1 3 3
Z
x2 −2x
12. I (x) = (2x−1)(x2 +1)
dx. Qui è:

p (x) = x2 − 2x, q (x) = (2x − 1) x2 + 1

Quindi:
p (x) A Bx + C
= + 2
q (x) 2x − 1 x +1
da cui

A + 2B + 0 = 1
0 − B + 2C = −2
A+0−C =0
3.12 Integrazione delle funzioni razionali 134

La soluzione è  
3 4 3
(A, B, C) = − , ,−
5 5 5
Perciò:
p (x) 3 1 4x − 3
=− + · 2
q (x) 5 (2x − 1) 5 x + 1
L’integrale:
3 1
I (x) = − ln |2x − 1| + J (x) ,
10 5
essendo:
Z
4x − 3
J (x) = dx
x2 + 1

= 2 ln x2 + 1 − 3 arctan x + C1 ,

Si conclude che:
3 2  3
I (x) = − ln |2x − 1| + ln x2 + 1 − arctan x + C
10 5 5

***

3.12.7 Calcolare i seguenti integrali


Z Z Z
5x2 +11x−2 x2 −2x+3 2x2 +12
1) (x+5)(x2 +9)
dx 5) (x−1)2 (x2 +1)
dx 9) x4 +12x2 +16
dx
Z Z Z
2) x−1
9x2 −18x+7
dx 6) x−3
(x−2)(x2 +x+1)2
dx 10) dx
x3 +x
Z Z Z
x3 +x+1 2x3
3) x4 +x2
dx 7) dx
x(x2 +1)
11) (x2 +1)2
dx
Z Z Z
x3 +2x2 −2x x3 +x−1
4) dx
(x2 −1)2
8) x4 +4
dx 12) (x2 +1)2
dx

3.12.8 Soluzioni
Z
5x2 +11x−2
1. I (x) = (x+5)(x2 +9)
dx. Qui è:

p (x) = 5x2 + 11x − 2, q (x) = (x + 5) x2 + 9

Quindi:
p (x) A Bx + C
= + 2
q (x) x+5 x +9
3.12 Integrazione delle funzioni razionali 135

da cui

A+B+0=5
0 + 5B + C = 11
9A + 0 + 5C = −2

La soluzione è
(A, B, C) = (2, 3, −4)
Perciò:
p (x) 2 3x − 4
= + 2
q (x) x+5 x +9
L’integrale:
I (x) = 2 ln |x + 5| + J (x) ,

essendo:
Z
3x − 4
J (x) = dx
x2 + 9
3  4 x
= ln x2 + 9 − arctan + K1 ,
2 3 3

Si conclude che:
3  4 x
2
I (x) = 2 ln |x + 5| + ln x + 9 − arctan +K
2 3 3
R
2. I (x) = 9x2 −18x+7
x−1
dx. Anziché procedere per decomposizione in frazioni
semplici, è preferibile procedere nel modo seguente
Z
1 d (9x2 − 18x + 7)
I (x) =
18 9x2 − 18x + 7
1 2
= ln 9x − 18x + 7 + K
18
R x3 +x+1
3. I (x) = x4 +x2
dx. Qui è:

p (x) = x3 + x + 1, q (x) = x4 + x2 = x2 x2 + 1

Quindi:
p (x) A B Cx + D
= + 2+ 2
q (x) x x x +1
3.12 Integrazione delle funzioni razionali 136

da cui

A+0+C +0=1
0+B+0+D =0
A+0+0+0=1
0+B+0+0=1

La soluzione è
(A, B, C, D) = (1, 1, 0, −1)
Perciò:
p (x) 1 1 1
= + 2− 2
q (x) x x x +1
L’integrale:
1
I (x) = ln |x| − − arctan x + K,
x
R
4. I (x) = dx
(x2 −1)2
. Qui è:
2
p (x) = 1, q (x) = x2 − 1 = (x − 1)2 (x + 1)2

Quindi:
p (x) A B C D
= + 2 + +
q (x) x − 1 (x − 1) x + 1 (x + 1)2

da cui

A+0+C +0=0
A+B−C +D =0
−A + 2B − C − 2D = 0
−A + B + C + D = 1

La soluzione è  
1 1 1 1
(A, B, C, D) = − , , ,
4 4 4 4
Perciò:
p (x) 1 1 1 1
=− + 2 + +
q (x) 4 (x − 1) 4 (x − 1) 4 (x + 1) 4 (x + 1)2

L’integrale:  
1 x + 1
− 2x
I (x) = ln +K
4 x − 1 x2 − 1
3.12 Integrazione delle funzioni razionali 137

R x2 −2x+3
5. I (x) = (x−1)2 (x2 +1)
dx. Qui è:

p (x) = x2 − 2x + 3, q (x) = (x − 1)2 x2 + 1

Quindi:
p (x) A B Cx + D
= + 2 +
q (x) x − 1 (x − 1) (x2 + 1)
da cui

A+0+C +0=0
−A + B − 2C + D = 1
A + 0 + C − 2D = −2
−A + B + 0 + D = 3

La soluzione è
(A, B, C, D) = (−1, 1, 1, 1)
Perciò:
p (x) 1 1 x+1
=− + 2 +
q (x) x − 1 (x − 1) (x2 + 1)
L’integrale:
1
I (x) = − ln |x − 1| + + J (x) ,
1−x
essendo:
Z
x+1
J (x) = dx
x2 + 1
Z Z
x dx
= 2
dx + 2
x +1 x +1
1 
= ln x2 + 1 + arctan x
2
Quindi: √
1 x2 + 1
I (x) = + ln + arctan x + K
1−x |x − 1|
R
6. I (x) = x−3
(x−2)(x2 +x+1)2
dx. Qui è:
2
p (x) = x − 3, q (x) = (x − 2) x2 + x + 1

Quindi:
p (x) A Bx + C Dx + E
= + 2 +
q (x) x − 2 x + x + 1 (x + x + 1)2
2
3.12 Integrazione delle funzioni razionali 138

da cui

A+B+0+0+0=0
2A − B + C + 0 + 0 = 0
3A − B − C + D + 0 = 0
2A − 2B − C − 2D + E = 1
A + 0 − 2C + 0 − 2E = −2

La soluzione è
 
1 1 3 1 10
(A, B, C, D, E) = − , , , ,
49 49 49 7 7

Perciò:
p (x) 1 1 1 x+3 1 x + 10
=− + +
q (x) 49 x − 2 49 x + x + 1 7 (x + x + 1)2
2 2

L’integrale:
1 1 1
I (x) = − ln |x − 2| + J1 (x) + J2 (x) ,
49 49 7

essendo:
Z  
x+3 1  5 2x + 1
J1 (x) = 2
dx = ln x2 + x + 1 + √ arctan √ + K1
x +x+1 2 3 3
Z  
x + 10 19x + 8 38 2x + 1
J2 (x) = 2 dx = + √ arctan √ + K2
(x2 + x + 1) 3 (x2 + x + 1) 3 3 3

Quindi:
1
h
42(19x+8)
√ 
2x+1

2
i
I (x) = 882 x2 +x+1
+ 562 3 arctan √
3
− 18 ln |x − 2| + 9 ln (x + x + 1) +
+K
R
7. I (x) = dx
x(x2 +1)
. Qui è:

p (x) = 1, q (x) = x x2 + 1

Quindi:
p (x) A Bx + C
= + 2
q (x) x x +1
3.12 Integrazione delle funzioni razionali 139

da cui

A+B+0=0
0+0+C =0
A+0+0=1

La soluzione è
(A, B, C) = (1, −1, 0)
Perciò:
p (x) 1 x
= − 2
q (x) x x +1
L’integrale:
1 
I (x) = ln |x| − ln x2 + 1 + K
2
R x3 +2x2 −2x
8. I (x) = x4 +4
dx.Abbiamo:
Z Z
x3 2x2 − 2x
I (x) = dx + dx (3.66)
x4 + 4 x4 + 4
1 
= ln x4 + 4 + J (x) ,
4

essendo: Z
2x2 − 2x
J (x) = dx
x4 + 4
Abbiamo:
2x2 − 2x Ax + B Cx + D
4
= 2 + 2
x +4 x − 2x + 2 x + 2x + 2
da cui:

A+0+C +0=0
2A + B − 2C + D = 1
2A + 2B + 2C − 2D = −1
0 + 2B + 0 + 2D = 0

La soluzione di tale sistema è:


1 1 1 1
A = ,B = − ,C = − ,D =
4 4 4 4

Quindi:
1
J (x) = [G1 (x) − G2 (x)] ,
4
3.12 Integrazione delle funzioni razionali 140

essendo: Z
x−1
G1,2 = dx
x2 ± 2x + 2
Calcolando i due integrali:
1 2
G1 (x) = ln x − 2x + 2 + C1
2
1 
G2 (x) = ln x2 + 2x + 2 − 2 arctan (x + 1) + C2
2

Finalmente:
1 4
 1 |x2 − 2x + 2|
I (x) = ln x + 4 + ln 2 + arctan (x + 1) + C
4 4 x + 2x + 2
R 2x2 +12
R x2 +6
9. I (x) = x4 +12x2 +16
dx =2 x4 +12x2 +16
dx. Qui è:
 √  √ 
p (x) = x2 + 6, q (x) = x4 + 12x2 + 16 = x2 + 6 + 2 5 x2 + 6 − 2 5

Quindi:
p (x) Ax + B Cx + D
= √ + √
q (x) 2 2
x +6+2 5 x +6−2 5
da cui il sistema:

A+0+C +0=0
0+B+0+D =1
 √   √ 
6−2 5 A+0+ 6+2 5 C +0=0
 √   √ 
A+ 6−2 5 B+0+ 6+2 5 D =6

La soluzione è:  
1 1
(A, B, C, D) = 0, , 0, ,
2 2
donde: Z Z
dx dx
I (x) = √ + √
6 + 2 5 + x2 6 − 2 5 + x2
3.12 Integrazione delle funzioni razionali 141

Calcoliamo (α > 0)
Z Z
dx 1 dx
2
=  2
α+x α √x
1+ α
 
Z d √x
1 α
=√  2
α √x
1+ α
 
1 x
= √ arctan √ + const
α α

Quindi:
   
1 x 1 x
I (x) = q √  arctan q

√  +q

√  arctan q

√ 

2 3+ 5 2 3+ 5 2 3− 5 2 3− 5

R
10. I (x) = dx
x3 +x
. Qui è:

p (x) = 1, q (x) = x3 + x

Quindi:
p (x) A Bx + C
= + 2
q (x) x x +1
da cui il sistema:

A+B+0=0
0+0+C =0
A+0+0=1

La soluzione è:
(A, B, C) = (1, −1, 0) ,

donde:
Z Z
dx x
I (x) = − 2
dx
x x +1
|x|
= ln √ +C
x2 + 1
3.12 Integrazione delle funzioni razionali 142

R 2x3
11. I (x) = (x2 +1)2
dx. Qui è:
2
p (x) = 2x3 , q (x) = x2 + 1

Quindi:
p (x) Ax + B Cx + D
= 2 +
q (x) x +1 (x2 + 1)2
da cui il sistema:

A+0+0+0=2
0+B+0+0=0
A+0+C +0=0
0+B+0+D =0

La soluzione è:
(A, B, C, D) = (2, 0, −2, 0) ,

donde:
Z Z
2x 2x
I (x) = dx − dx
2
x +1 (x + 1)2
2
Z Z
d (x2 + 1) d (x2 + 1)
= −
x2 + 1 (x2 + 1)2
 1
= ln x2 + 1 + 2 +C
x +1
R x3 +x−1
12. I (x) = (x2 +1)2
dx. Procedendo come nel n. 11:

A+0+0+0=1
0+B+0+0=0
A+0+C +0=1
0 + B + 0 + D = −1,

la cui soluzione è:
(A, B, C, D) = (1, 0, 0, −1) ,

donde: Z
√ dx
I (x) = ln x2 +1−
(x2 + 1)2
3.12 Integrazione delle funzioni razionali 143

L’integrale a secondo membro è dato dalla (3.63), che riscriviamo:


Z  
dx 1 x
= + arctan x + C1 (3.67)
(x2 + 1)2 2 x2 + 1

per cui:  
√ 1 x
I (x) = ln x2 + 1 − + arctan x + C
2 x2 + 1

3.12.9 Calcolare i seguenti integrali


Z Z Z
x4 +8x3 −x2 +2x+1 x3 +x2 +x+3
1) (x2 +x)(x3 +1)
dx 5) (x2 +1)(x2 +3)
dx 9) dx
(x4 −1)2
Z Z Z
x3 +x2 −5x+15
2) (x2 +5)(x2 +2x+3)
dx 6) xdx
(x−2)2
10) dx
(x+1)2 (x2 +1)
Z Z
x2 −3x−1
3) dx
e2x −ex
7) x3 +x2 −2x
dx
Z Z
4) sin x
cos x(1+cos2 x)
dx 8) dx
(x+1)2 (x2 −1)2

3.12.10 Soluzioni
Z
x4 +8x3 −x2 +2x+1
1. I (x) = (x2 +x)(x3 +1)
dx. Qui è:

  
p (x) = x4 +8x3 −x2 +2x+1, q (x) = x2 + x x3 + 1 = x (x + 1)2 x2 − x + 1

Quindi:
p (x) A B C Dx + E
= + + 2 + 2
q (x) x x + 1 (x + 1) x −x+1

da cui

A+B+0+D+0=1
A + 0 + C + 2D + E = 8
0 + 0 − C + D + 2E = −1
A+B+C +0+E =2
A+0+0+0+0=1

La soluzione è
(A, B, C, D, E) = (1, −2, 3, 2, 0)
Perciò:
p (x) 1 2 3 2x
= − + 2 + 2
q (x) x x + 1 (x + 1) x −x+1
3.12 Integrazione delle funzioni razionali 144

L’integrale:
Z
3 2x
I (x) = ln |x| − 2 ln |x + 1| − + dx
x+1 x2 −x+1

Calcoliamo a parte l’integrale a secondo membro:


Z Z
2x 2x − 1 + 1
2
dx = dx
x −x+1 x2 − x + 1
Z
2 1
= ln x − x + 1 +

2
dx
x −x+1
Z
4 1
= ln x2 − x + 1 + h i dx
3 1 + √2 x − 1 
3 2
 
2
 2 2x − 1
= ln x − x + 1 + √ arctan √ + C1 ,
3 3

donde:
 
|x3 − x2 + x| 3 2 2x − 1
I (x) = ln 2 − + √ arctan √ +C
(x + 1) x+1 3 3
R x3 +x2 −5x+15
2. I (x) = (x2 +5)(x2 +2x+3)
dx. Qui è:
 
p (x) = x3 + x2 − 5x + 15, q (x) = x2 + 5 x2 + 2x + 3

Quindi:
p (x) Ax + B Cx + D
= 2 + 2
q (x) x +5 (x + 2x + 3)
da cui

A+0+C +0=1
2A + B + 0 + D = 1
3A + 2B + 5C + 0 = −5
0 + 3B + 0 + 5D = 15

La soluzione è
(A, B, C, D) = (0, −5, 1, 6)

Da ciò segue:
Z Z
dx x+6
I (x) = −5 2
+ dx
x +5 x2 + 2x + 3
3.12 Integrazione delle funzioni razionali 145

Calcoliamo a parte i due integrali:


 
Z Z d √x
dx 1 5
2
=√  2
x +5 5 √x
1+ 5
 
1 x
= √ arctan √ +C
5 5

Il secondo integrale si calcola con la (3.45):


Z  
x+6 √ 5 x + 1
dx = ln x2 + 2x + 3 + √ arctan √ +C
x2 + 2x + 3 2 2
Finalmente:
   
√ 5 x+1 1 x
I (x) = ln x + 2x + 3 + √ arctan √
2 − √ arctan √ +C
2 2 5 5
R
3. I (x) = e2xdx
−ex
. Eseguiamo il cambio di variabile:
dy
y = ex =⇒ dx = ,
y
per cui: Z
dy
I (y) =
y2 (y − 1)
Procediamo per decomposizione in frazioni semplici:
1 A B C
= + +
y 2 (y − 1) y y2 y − 1

Il principio di identità dei polinomi conduce al sistema di Cramer:


A+0+C =0
−A + B + 0 = 0
0 − B + 0 = 1,

con soluzione:
(A, B, C) = (−1, −1, 1)
Quindi:
1 y − 1
I (y) = + ln
+C
y y
Ripristinando la variabile x:

I (x) = e−x + ln 1 − e−x + C
3.12 Integrazione delle funzioni razionali 146

R
4. I (x) = sin x
cos x(1+cos2 x)
dx. Eseguiamo il cambio di variabile:

y = cos x =⇒ dy = − sin xdx,

donde: Z
dy
I (y) = −
y (1 + y 2 )
Procediamo per decomposizione in frazioni semplici:
1 A By + C
= + 2
y 2 (1 + y 2 ) y y +1

Il principio di identità dei polinomi conduce al sistema di Cramer:

A+B+0=0
0+0+C =0
A + 0 + 0 = 1,

con soluzione:
(A, B, C) = (1, −1, 0)

Quindi:
|y|
−I (y) = ln p +C
1 + y2
Ripristinando la variabile x:

1 + cos2 x
I (x) = ln +C
|cos x|
R x3 +x2 +x+3
5. I (x) = (x2 +1)(x2 +3)
dx. Qui è:
 
p (x) = x3 + x2 + x + 3, q (x) = x2 + 1 x2 + 3

Quindi:
p (x) Ax + B Cx + D
= 2 + 2
q (x) x +1 x +3
da cui

A+0+C +0=1
0+B+0+D =1
3A + 0 + C + 0 = 1
0 + 3B + 0 + D = 3
3.12 Integrazione delle funzioni razionali 147

La soluzione è
(A, B, C, D) = (0, 1, 1, 0)
Da ciò segue:
1 
I (x) = arctan x + ln x2 + 3 + C
2
R
6. I (x) = xdx
(x−2)2
. Qui è:

p (x) = x, q (x) = (x − 2)2

Quindi:
p (x) A B
= +
q (x) x − 2 (x − 2)2
da cui
A=1
B − 2A = 0

La soluzione è
(A, B) = (1, 2)
Da ciò segue:
2
I (x) = ln |x − 2| − +C
x−2
R x2 −3x−1
7. I (x) = x3 +x2 −2x
dx. Qui è:
p (x) = x2 − 3x − 1, q (x) = x3 + x2 − 2x = x (x + 2) (x − 1)

Quindi:
p (x) A B C
= + +
q (x) x x+2 x−1
da cui
A+B+C =1
A − B + 2C = −3
2A + 0 + 0 = 1

La soluzione è  
1 3
(A, B, C) = , , −1
2 2
Da ciò segue: q
x (x + 2)3
I (x) = ln +C
|x − 1|
3.12 Integrazione delle funzioni razionali 148

R
8. I (x) = dx
(x+1)2 (x2 −1)2
. Qui è:
2
p (x) = 1, q (x) = (x + 1)2 x2 − 1 = (x + 1)4 (x − 1)2

Quindi:

p (x) A B C D E F
= + 2 + 3 + 4 + 4 +
q (x) x + 1 (x + 1) (x + 1) (x + 1) (x + 1) (x − 1)2

da cui

(A + E) x5 + (A + B + 3E + F ) x4
+ (−2A + C + 2E + 4F ) x3 +
+ (−2A − 2B − C + D − 2E + 6F ) x2 +
+ (A − C − 2D − 3E + 4F ) x+
+ (A + B + C + D − E + F )
=1

Per il principio di identità dei polinomi:

A+0+0+0+E+0=0
A + B + 0 + 0 + 3E + F = 0
−2A + 0 + C + 0 + 2E + 4F = 0
−2A − 2B − C + D − 2E + 6F = 0
A + 0 − C − 2D − 3E + 4F = 0
A+B+C +D−E+F =1

La soluzione è
 
1 3 1 1 1 1
(A, B, C, D, E, F ) = , , , ,− ,
8 16 4 4 8 16

Da ciò segue:
Z Z Z
1 dx 3 dx 1 dx
I (x) = + 2 +
8 x + 1 16 (x + 1) 4 (x + 1)3
Z Z Z
1 dx 1 dx 1 dx
+ 4 − +
4 (x + 1) 8 x − 1 16 (x − 1)2

1 x + 1 3x3 + 6x2 + x − 4
= ln − +C
8 x − 1 12 (x + 1)3 (x − 1)
3.12 Integrazione delle funzioni razionali 149

R
9. I (x) = dx
(x4 −1)2
. Qui è:
2
p (x) = 1, q (x) = (x − 1)2 (x + 1)2 x2 + 1

Quindi:

p (x) A B C D Ex + F Gx + H
= + 2 + + 2 + +
q (x) x − 1 (x − 1) x + 1 (x + 1) x2 + 1 (x2 + 1)2

da cui

A+0+C +0+E+0+0+0=0
A+B−C +D+0+F +0+0=0
A + 2B + C − 2D − E + 0 + G + 0 = 0
A + 3B − C + 3D + 0 − F + 0 + H = 0
−A + 4B − C − 4D − E + 0 − 2G + 0 = 0
−A + 3B + C + 3D + 0 − F + 0 − 2H = 0
−A + 2B − C − 2D + E + 0 + G + 0 = 0
−A + 2B + C + D + 0 + F + 0 + H = 1

La soluzione è
 
−3 1 3 1 1
(A, B, C, D, E, F, G, H) = , , , 0, , 0,
16 16 16 4 4

Da ciò segue:
Z Z Z
3 dx 1 dx 3 dx
I (x) = − + 2 +
16 x − 1 16 (x − 1) 16 x + 1
Z Z Z
1 dx 1 dx 1 dx
+ 2 + +
16 (x + 1) 4 x + 1 4 (x2 + 1)2
2
Z
3 x + 1 1 1 1 1 1 1 dx
= ln + arctan x − − + + const
16 x−1 4 16 x − 1 16 x + 1 4 (x + 1)2
2

R
L’integrale dx
(x2 +1)2
è dato dalla (3.63) donde:
 
1 1 + x 4x
I (x) = 3 ln + 6 arctan x − + const
16 1 − x x4 − 1
3.12 Integrazione delle funzioni razionali 150

R
10. I (x) = dx
(x+1)2 (x2 +1)
. Qui è:

p (x) = 1, q (x) = (x + 1)2 x2 + 1

Quindi:
p (x) A B Cx + D
= + 2 +
q (x) x + 1 (x + 1) x2 + 1

Per il principio di identità dei polinomi:

A+0+C +0=0
A + B + 2C + D = 0
A + 0 + C + 2D = 0
A + B + 0 + D = 1,

la cui soluzione è:  
1 1 1
(A, B, C, D) = , ,− ,0
2 2 2
Quindi:
p (x) 1 1 x
= + 2 − 2
,
q (x) 2 (x + 1) 2 (x + 1) 2 (x + 1)

da cui l’integrale:
1 1 1 
I (x) = ln |x + 1| − + ln x2 + 1 + C
2 2 (x + 1) 4

3.12.11 Funzioni razionali improprie


Qui il grado del numeratore è ≥ di quello del denominatore.
Esempio
Z 3
x +8
dx
x−2
Eseguendo la divisione dei polinomi, otteniamo:

x3 + 8 16
= x2 + 2x + 4 + ,
x−2 x−2
donde:
Z
x3 + 8 x3
dx = + x2 + 4x + 16 ln |x − 2| + C
x−2 3
3.12 Integrazione delle funzioni razionali 151

3.12.12 Calcolare i seguenti integrali


Z Z Z
x6 −x5 +x4 −x2 +x 5x3 +2 x4
1) x4 −1
dx 5) x3 −5x2 +4x
dx 9) (1−x)3
dx
Z Z
x4 −2x3 +3x2 −x+3 x3 −1
2) x3 −2x2 +3x
dx 6) 4x3 −x
dx
Z Z
x2 +3x−4 x4 −6x3 +12x2 +6
3) x2 −2x−8
dx 7) x3 −6x2 +12x−8
dx
Z Z
x2 −5x+9 x4
4) x2 −5x+6
dx 8) x4 −1
dx

3.12.13 Soluzioni
R x6 −x5 +x4 −x2 +x
1. I (x) = x4 −1
dx. Eseguendo la divisione tra polinomi:
x6 − x5 + x4 − x2 + x 1
4
= x2 − x + 1 + 4 ,
x −1 x −1
donde:
1 1
I (x) = x3 − x2 + x + I1 (x) ,
3 2
essendo: Z
dx
I1 (x) = 4
x −1
Per calcolare I1 (x) occorre applicare il metodo dei coefficienti indetermi-
nati:
1 A B Cx + D
4
= + + 2 ,
x −1 x−1 x+1 x +1
da cui il sistema:
A+B+C +0=0
A−B+0+D =0
A+B−C +0=0
A−B+0−D =1

La cui unica soluzione è:


 
1 1 1
(A, B, C, D) = , − , 0, − ,
4 4 2

per cui:
Z Z Z
1 dx 1 dx 1 dx
I1 (x) = − −
4 x − 1 4 x + 1 2 x2 + 1

1 x − 1 1
= ln − arctan x + C
4 x + 1 2
3.12 Integrazione delle funzioni razionali 152

Finalmente:

1 3 1 2 1 x − 1 1
I (x) = x − x + x + ln − arctan x + C
3 2 4 x + 1 2
R x4 −2x3 +3x2 −x+3
2. I (x) = x3 −2x2 +3x
dx. Risulta:

x4 − 2x3 + 3x2 − x + 3 3−x


3 2
=x+ 3 ,
x − 2x + 3x x − 2x2 + 3x

per cui:
1
I (x) = x2 + I1 (x) ,
2
essendo: Z
3−x
I1 (x) = dx
x3 − 2x2 + 3x
Applichiamo il metodo dei coefficienti indeterminati:
3−x A Bx + C
= + 2 ,
x3 2
− 2x + 3x x x − 2x + 3

ottenendo il sistema:

A+B+0=0
−2A + 0 + C = −1
3A + 0 + 0 = 3,

che ammette l’unica soluzione:

(A, B, C) = (1, −1, 1)

Quindi:
Z Z
dx x−1
I1 (x) = − dx
x x2
− 2x + 3
Z
1 d (x2 − 2x + 3)
= ln |x| −
2 x2 − 2x + 3
1 
= ln |x| − ln x2 − 2x + 3 + C
2

Si conclude che:
1 1 
I (x) = x2 + ln |x| − ln x2 − 2x + 3 + C
2 2
3.12 Integrazione delle funzioni razionali 153

R x2 +3x−4
3. I (x) = x2 −2x−8
dx. Risulta:

x2 + 3x − 4 5x + 4
2
=1+ 2
x − 2x − 8 x − 2x − 8

Perciè:
I (x) = x + I1 (x) ,

essendo: Z
5x + 4
I1 (x) = dx
x2 − 2x − 8
Tale integrale si calcola con la (3.45):

5 2 3 x − 4
I1 (x) = ln x − 2x − 8 + ln +C
2 2 x + 2

= ln (x + 2) (x − 4)4 + C,

donde:
I (x) = x + ln (x + 2) (x − 4)4 + C
R x2 −5x+9
4. I (x) = x2 −5x+6
dx. Abbiamo:
Z 2
x − 5x + 6 + 3
I (x) = dx
x2 − 5x + 6
Z Z
dx
= dx + 3 2
x − 5x + 6

L’integrale a secondo membro si calcola con la (3.45):


Z
dx x − 3
= ln
+ C,
x2 − 5x + 6 x − 2

per cui:
x − 3
I (x) = x + 3 ln
+C
x − 2
R 5x3 +2 R R 2 −20x
5. I (x) = x3 −5x 2 +4x dx = 5 dx + 2+25x
x3 −5x2 +4x
dx. Calcoliamo a parte il
secondo integrale: Z
def 25x2 − 20x + 2
J (x) = dx
x3 − 5x2 + 4x
Metodo dei coefficienti indeterminati:
25x2 − 20x + 2 A B C
= + + ,
x3 − 5x2 + 4x x x−4 x−1
3.12 Integrazione delle funzioni razionali 154

da cui il sistema lineare:

A + B + C = 25
5A + B + 4C = 20
4A + 0 + 0 = 2,

che ammette l’unica soluzione:


 
1 161 7
(A, B, C) = , ,− ,
2 6 3

perciò:
1 161 7
I (x) = 5x + ln |x| + ln |x − 4| − ln |x − 1| + C
2 6 3
R x3 −1 R R 1 R R
x−1
6. I (x) = 4x 1
3 −x dx = 4 dx + 4
4x3 −x
dx = 1
4
dx + 41 x−4
4x3 −x
dx. Calcoliamo
a parte il secondo integrale:
Z
def x−4
J (x) = dx
4x3 − x

Metodo dei coefficienti indeterminati:


Z Z Z
4 7 9
J (x) = dx − dx − dx
x 2 (2x − 1) 2 (2x + 1)
7 9
= 4 ln x − ln (2x − 1) − ln (2x + 1) + C
4 4

Quindi:
1 7 9
I (x) = x + ln x − ln (2x − 1) − ln (2x + 1) + C
4 16 16
R x4 −6x3 +12x2 +6
7. I (x) = x3 −6x2 +12x−8
dx. Abbiamo:

x4 − 6x3 + 12x2 + 6 6 + 8x
3 2
=x+ 3 ,
x − 6x + 12x − 8 x − 6x2 + 12x − 8

per cui: Z
1 2 4x + 3
I (x) = x + 2 dx (3.68)
2 x − 6x2 + 12x − 8
3

Applicando il metodo dei coefficienti indeterminati


4x + 3 11 4
= 3 +
x3 2
− 6x + 12x − 8 (x − 2) (x − 2)2
3.12 Integrazione delle funzioni razionali 155

Quindi Z
4x + 3 5 − 8x
dx = +C
x3 − 6x2 + 12x − 8 2 (x − 2)2
Sostituendo nella (3.68):
x4 − 4x3 + 4x2 − 16x + 10
I (x) = +C
2 (x − 2)2
R x4
8. I (x) = x4 −1
dx. Abbiamo:

x4 1
4
=1+ 4 ,
x −1 x −1
donde: Z
dx
I (x) = x + (3.69)
x4−1
Calcoliamo l’integrale a secondo membro della (3.69) con il metodo dei
coefficienti indeterminati:
1 A B Cx + D
= + + ,
x4 − 1 x−1 x+1 x2 + 1
da cui il sistema lineare:
A+B+C +0=0
A−B+0+D =0
A+B−C +0=0
A − B + 0 − D = 1,

la cui unica soluzione è:


 
1 1 1
(A, B, C, D) = , − , 0, −
4 4 2

Quindi:
1 1 1 1
= − −
x4 −1 2
4 (x − 1) 4 (x + 1) 2 (x + 1)
Perciò: Z
dx 1 x − 1 1
= ln − arctan x + C
x4 − 1 4 x + 1 2
Sostituendo nella (3.69):

1 x − 1 1
I (x) = x + ln − arctan x + C
4 x + 1 2
3.12 Integrazione delle funzioni razionali 156

R x4
9. I (x) = (1−x)3
dx. Abbiamo:

x4 −8x + 6x2 + 3
= −x − 3 + ,
(1 − x)3 (1 − x)3

da cui l’integrale:
Z  
6x2 − 8x + 3
I (x) = −x − 3 − dx (3.70)
(x − 1)3
Z
1 2 6x2 − 8x + 3
= − x − 3x − dx
2 (x − 1)3

Calcoliamo l’integrale a secondo membro della (3.70) con il metodo dei


coefficienti indeterminati:
6x2 − 8x + 3 A B C
3 = + 2 + ,
(x − 1) x − 1 (x − 1) (x − 1)3

da qui il sistema lineare:

A+0+0=6
−2A + B + 0 = −8
A − B + C = 3,

che ammette l’unica soluzione:

(A, B, C) = (6, 4, 1)

Quindi
Z
6x2 − 8x + 3 4 1 1
3 dx = 6 ln |x − 1| − − 2 + +C
(x − 1) x − 1 2 (x − 1) (x − 1)3

Sostituendo nella (3.70)


1 8x − 7
I (x) = − x2 − 3x − 6 ln |x − 1| + +C
2 2 (x − 1)2
Z Z
dx dx
3.13 In (x) = (x2 −1)n
, Jn (x) = (x2 +1)n
157

Z Z
dx dx
3.13 In (x) = n , Jn (x) = n
( x2 −1 ) (x2 +1 )
Iniziamo con il primo eseguendo il cambio di variabile (anziché applicare il metodo
dei coefficienti indeterminati):
x−1
t= (3.71)
x+1
Dalla (3.71) otteniamo:
1+t 2
x= , dx = dt,
1−t (1 − t)2
donde:
Z
1 (1 − t)2(n−1)
In (t) = dt (3.72)
22n−1 tn
La (3.72) può essere utilizzata solo per piccoli valori di n. Ad esempio, per n = 1:
Z
1 dt 1
I1 (t) = = ln |t| + C
2 t 2
Ripristinando la variabile x:

1 x − 1
I1 (x) = ln +C
2 x + 1
Osservazione. Il risultato precedente è equivalente a:
Z
dx
2
= − arctanh x + C
x −1
Per n = 2:

Z
1 (1 − t)2
I2 (x) = dt
8 t2
Z Z Z 
1 dt dt
= −2 + dt
8 t2 t
 
1 1
= − − 2 ln |t| + t + C1
8 t

Ripristinando la variabile x:
Z Z
dx dx
3.13 In (x) = (x2 −1)n
, Jn (x) = (x2 +1)n
158

 
1 x+1 x − 1 x − 1
I2 (x) = − − 2 ln + +C
8 x−1 x + 1 x + 1

x 1 x − 1
= − ln +C
2 (1 − x2 ) 4 x + 1

Per n = 3:

Z
1 (1 − t)4
I3 (x) = dt
32 t3
Z Z Z Z Z 
1 dt dt dt
= −4 +6 − 4 dt + tdt
32 t3 t2 t
 
1 1 4 1 2
= − 2 + + 6 ln |t| − 4t + t + C
32 2t t 2

Ripristinando la variabile x:

" #
1 1 (x + 1)2 x+1 x − 1 x − 1 1 (x − 1)2
I3 (x) = − +4 + 6 ln −4 + +C
32 2 (x − 1)2 x−1 x + 1 x + 1 2 (x + 1)2

x (3x2 − 5) 3 x − 1
= + ln +C
8 (x2 − 1)2 16 x + 1

I rimanenti integrali (n > 3) sono riportati in Appendice A.


Passiamo a Jn (x); per n = 1 è un integrale fondamentale:

J1 (x) = arctan x + C
Per n = 2, abbiamo già calcolato (vedi eq.3.59):
1 x
J2 (x) = arctan x + 2
+C
2 2 (x + 1)
È possibile giungere al medesimo risultato attraverso un’integrazione per parti.
Per ogni n:

Z
x2 + 1 − x2
Jn (x) = dx
(x2 + 1)n
Z Z
x2 + 1 x2
= dx − dx,
(x2 + 1)n (x2 + 1)n
cioè:
Z Z
dx dx
3.13 In (x) = (x2 −1)n
, Jn (x) = (x2 +1)n
159

Jn (x) = Jn−1 (x) − Jn (x) , (3.73)


essendo:
Z
def x2
Jn (x) = dx (3.74)
(x2 + 1)n
Per n = 2:

J2 (x) = J1 (x) − J2 (x)


Qui è:
Z
x2
J2 (x) = dx
(x2 + 1)2
Per calcolare tale integrale, procediamo per parti osservando che:
 
1 2xdx
d = − ,
x2 + 1 (x2 + 1)2
donde:

Z  
1 1
J2 (x) = − xd
2 x2 + 1
 Z 
1 1 dx
=− x 2 −
2 x +1 x2 + 1
 
1 x
=− − J1 (x)
2 x2 + 1
Quindi:
1 x
J2 (x) = J1 (x) + 2
2 2 (x + 1)
R
Ma J1 (x) = dx
x2 +1
= arctan x+const, per cui:
1 x
J2 (x) = arctan x + +C (3.75)
2 2 (x2 + 1)
Passiamo a n = 3; per la (3.73):

J3 (x) = J2 (x) − J3 (x)


Qui è:
Z
x2
J3 (x) = dx
(x2 + 1)3
Z Z
dx dx
3.13 In (x) = (x2 −1)n
, Jn (x) = (x2 +1)n
160

Anche in questo caso procediamo per parti osservando che:


 
1 4xdx
d 2 = − ,
2
(x + 1) (x2 + 1)3
donde:

Z  
1 1
J3 (x) = − xd
4 (x2 + 1)2
1 x 1
=− 2 + J2 (x)
4 (x2 + 1) 4
Quindi:
3 1 x
J3 (x) = J2 (x) +
4 4 (x2 + 1)2
L’integrale J2 (x) è dato dalla (3.75), per cui:

3 3 x 1 x
J3 (x) = arctan x + + +C (3.76)
8 8 x + 1 4 (x + 1)2
2 2

3 x (3x2 + 5)
= arctan x + +C
8 8 (x2 + 1)2
Iterando il procedimento, si trova:
2n − 3 x
Jn (x) = Jn−1 (x) + +C (3.77)
2 (n − 1) 2 (n − 1) (x2 + 1)n−1
Ad esempio per n = 4:

5 x
J4 (x) = J3 (x) + (3.78)
6 6 (x2 + 1)3
5 5 x 5 x x
= arctan x + + 2 + +C
16 16 x2 + 1 24 (x2 + 1) 6 (x2 + 1)3
Per n = 5:

7 x
J5 (x) = J4 (x) + (3.79)
8 8 (x + 1)4
2

35 35 x 35 x 7 x x
= arctan x + + 2 + 3 + +C
128 2
128 x + 1 192 (x2 + 1) 48 (x2 + 1) 8 (x2 + 1)4
Una tabella di valori di Jn (x) è riportata in Appendice A.
3.14 Integrali di funzioni irrazionali 161

3.14 Integrali di funzioni irrazionali


3.14.1 Integrali del “tipo 1”
Hanno un espressione generale:
Z "   p1   p2   pn #
ax + b q1 ax + b q2 ax + b qn
R x, , , ..., dx,
cx + d cx + d cx + d
essendo R una funzione razionale, mentre pi , qi ∈ N − {0}. Gli integrali di questo
tipo si riconducono ad integrali di funzioni razionali attraverso la sostituzione:
ax + b
= tk (3.80)
cx + d
Qui k è il m.c.m. di q1 , q2 , ...qn . Ad esempio, calcoliamo l’integrale:
Z
dx
I (x) = √ √
2x − 1 − 4 2x − 1
La (3.80) è:

2x − 1 = t4 ,
donde:
dx = 2t3 dt
Quindi l’integrale:

Z
t2
I (t) = 2 dt
t−1
Z  
1
=2 t+1+ dt
t−1
= (t + 1)2 + 2 ln |t − 1| + C

Ripristinando la variabile x:

4
2 √
I (x) = 1 + 2x − 1 + 2 ln 4 2x − 1 − 1 + C
3.14 Integrali di funzioni irrazionali 162

3.14.2 Calcolare i seguenti integrali


Z Z √
Zq
3 x−1
1) √x
x−1
dx 5) √3 x+1 dx 9) x x−1
x+1
dx
Z Z √
Z q
x+1+2
2) √
3
xdx
ax+b
6) √
(x+1)2 − x+1
dx 10) 3 x+1
x−1
dx
Z Z √
Z
x
3) √
dx
√ 7) x+2
dx 11) x2 √x+3
2x+3
dx
x+1+ (x+1)3 q
Z Z Z
2− 3 2x+1
4) √ dx√
x+ 3 x
8) dx

(2−x) 1−x
12) q
x−1
2x+1
dx
x+4 x−1

3.14.3 Soluzioni
R 3
1. I (x) = √x
x−1
dx. Qui è q1 = 2, n = 1, perciò: k = 2. La (3.80) porge:

x − 1 = t2

Ciò implica:
dx = 2tdt

Quindi l’integrale in funzione di t:


Z
3
I (t) = 2 t2 + 1 dt
2 6
= t7 + t5 + 2t3 + 2t + K
7 5

Ripristinando la variabile x:
q q q √
2 7 6
I (x) = (x − 1) + (x − 1) + 2 (x − 1)3 + 2 x − 1 + C1
5
7  5 
√ 1 3 3 2
=2 x−1 (x − 1) + (x − 1) + x + C
7 5
R
2. I (x) = √
3
xdx
ax+b
. Qui è q1 = 3, n = 1, perciò: k = 3. La (3.80) porge:

ax + b = t3

Ciò implica:
1 3 
x= t −b
a
3t2
dx = dt
a
3.14 Integrali di funzioni irrazionali 163

Quindi l’integrale in funzione di t:


Z
1 3  3t
I (t) = t −b dt
a a
Z
3 
= 2 t t3 − b dt
a
 
3 1 5 b 2
= 2 t − t + const
a 5 2

Ripristinando la variabile x:
 q q 
3 13 5 2
(ax + b) − b (ax + b) + const
3
I (x) = 2
a 5

R R h 1 3
i
3. I (x) = √
dx
√ = R (x + 1) 2 , (x + 1) 2 . Qui è q1 = q2 = 2,
x+1+ (x+1)3
perciò: k = 2. La (3.80) porge:

x + 1 = t2

Ciò implica:
dx = 2tdt

Quindi l’integrale in funzione di t:


Z
2tdt
I (t) =
t + t3
Z
dt
=2
1 + t2
= 2 arctan t + const

Ripristinando la variabile x:

I (x) = 2 arctan x + 1 + const

R R  1 1
4. I (x) = √ dx√
=
x+ 3 x
R x 2 , x 3 dx. Qui è q1 = 2, q2 = 3, perciò: k = 6.
La (3.80) porge:
x = t6

Ciò implica:
dx = 6t5 dt
3.14 Integrali di funzioni irrazionali 164

Quindi l’integrale in funzione di t:


Z
6t5 dt
I (t) =
t3 + t2
Z
t3
=6 dt
1+t
Z  
2 1
=6 t −t+1−
t+1
3 2
= 2t − 3t + 6t − ln |t + 1| + const

Ripristinando la variabile x:
√ √ √ √
I (x) = 2 x − 3 3 x + 6 6 x − 6 ln 6 x + 1 + const

R √x−1 R  1 1
5. I (x) = √ 3 x+1 dx = R x 2 , x 3 dx. Qui è q1 = 2, q2 = 3, perciò: k = 6.
La (3.80) porge:
x = t6

Ciò implica:
dx = 6t5 dt

Quindi l’integrale in funzione di t:


Z
t3 − 1 5
I (t) = 6t dt
t2 + 1
Z 5 3
t (t − 1)
=6 dt
t2 + 1
Z  
6 4 3 2 t−1
=6 t −t −t +t +t−1− 2
t +1
 Z 
1 7 1 5 1 4 1 3 1 2 t−1
=6 t − t − t + t + t −t−
7 5 4 3 2 t2 + 1
 
1 7 1 5 1 4 1 3 1 2 1 2
=6 t − t − t + t + t − t − ln t + 1 + arctan t + const
7 5 4 3 2 2

Ripristinando la variabile x:
6 √ 6√6 3√3 √
I (x) = x 6 x − x5 − x2 + 2 x
√ √ 7√ 5 2√
+3 3 x − 6 6 x − 3 ln 3 x + 1 + 6 arctan 6 x + const
3.14 Integrali di funzioni irrazionali 165

R √ R h 1 1
i
6. I (x) = x+1+2

(x+1)2 − x+1
dx = R (x + 1)2 , (x + 1) 2 , (x + 1) 2 dx. Qui è:

p1 = 2, q1 = 1
p2 = 1, q1 = 2
p3 = 1, q3 = 2,

donde:
k=2

Il cambio di variabile è:
x + 1 = t2

Ciò implica:
dx = 2tdt

L’integrale: Z 2
t + 2t
I (t) = 2 dt
t4 − t
L’integrando è una funzione razionale propria:
t+2 1 t+1
2
= − 2 ,
(t − 1) (t + t + 1) t−1 t +t+1

per cui:
Z Z 
dt t+1
I (t) = 2 − dt (3.81)
t2 + t + 1
t−1
 Z 
t+1
= 2 ln |t − 1| − dt
t2 + t + 1

L’integrale al secondo termine del secondo membro della (3.81) si calcola


attraverso la (3.45) ottenendo:
Z  
t+1 1 2
 1 2t + 1
dt = ln t + t + 1 + √ arctan √ +C
t2 + t + 1 2 3 3

Sostituendo nella (3.81):


 
 22 2t + 1
I (t) = 2 ln |t − 1| − ln t + t + 1 − √ arctan √ +C
3 3

Ripristinando la variabile x:
√ 2  √ 
x+1−1 2 2 x+1+1
I (x) = ln √ − √ arctan √ +C
x+2+ x+1 3 3
3.14 Integrali di funzioni irrazionali 166

R √ R  1

x
7. I (x) = x+2
dx = R x, x 2 dx. Qui è k = 2. La (3.80) porge:

x = t2

Ciò implica:
dx = 2tdt

Quindi l’integrale in funzione di t:


Z
t2
I (t) = dt
t2 + 2
Z Z 
dt
=2 dt − 2 2
t +2
   
t
 √ Z d √2 
= 6 t − 2  2 
1 + √t2
 
√ t
= 2 t − 2 arctan √
2

Ripristinando la variabile x:
 r 
√ √ x
I (x) = 2 x − 2 arctan + const
2

R R h 1
i
8. I (x) = (2−x)dx√1−x = R x, (1 − x) 2 dx. Qui è q1 = 2, perciò: k = 2. La
(3.80) porge:
1 − x = t2

Ciò implica:
dx = −2tdt

Quindi l’integrale in funzione di t:


Z
dt
I (t) = −2
1 + t2
= −2 arctan t + const

Ripristinando la variabile x:
√ 
I (x) = −2 arctan 1 − x + const
3.14 Integrali di funzioni irrazionali 167

R q x−1 R h  pq1 i
9. I (x) = x x+1 dx = R x, 1−x
1+x
1 dx. Qui è:

p1 = 1, q1 = 2,

donde:
k=2

La (3.80) porge:
x−1
= t2
x+1
Risolvendo rispetto a x:
1 + t2
x=
1 − t2
da cui il differenziale:
4t
dx = dt
(1 − t2 )
Quindi l’integrale in funzione di t:
Z 2 2
t (t + 1)
I (t) = 4 dt
(1 − t2 )3

Applicando il metodo dei coefficienti indeterminati:



1 t + 1 t + 3t2
I (t) = ln + +C
2 t − 1 (t2 − 1)2

Ripristinando la variabile x:
√ √
1 x − 1 + x + 1
I (x) = ln √ √ +
2 x − 1 − x + 1
√ √
(x + 1)2 3 (x − 1) x + 1 + (x + 1) x − 1
+ q +C
4 3
(x + 1)

Rq R h  pq1 i
10. I (x) = 3 x+1
x−1
dx = R x+1
x−1
1 . Qui è:

p1 = 1, q1 = 3,

donde:
k=3
3.14 Integrali di funzioni irrazionali 168

La (3.80) porge:
x+1
= t3
x−1
Risolvendo rispetto a x:
t3 − 1
x=
t3 − 1
da cui il differenziale:
6t2
dx = − 3 dt
(t − 1)
Quindi l’integrale in funzione di t:
Z
t3
I (t) = −6 dt
(t3 − 1)2

Sviluppiamo l’integrando in frazioni parziali:


t3 1 1 t+3 t+1
2 = 2 + − +
(t3 − 1) 9 (t − 1) 9 (t − 1) 9 (t2 + t + 1) 3 (t2 + t + 1)2

Quindi:
Z Z Z
t3 1 1 1 1 t+3 1 t+1
2 dt = ln |t − 1|− − dt− dt
(t3 − 1) 9 2
9t−1 9 t +t+1 3 (t2 + t + 1)2

Risulta:
Z  
t+3 1 2
 5 1 + 2t
dt = ln t + t + 1 + √ arctan √ + C1
t2 + t + 1 2 3 3
Z  
t+1 t−1 2 1 + 2t
dt = + √ arctan √ + C2
(t2 + t + 1)2 3 (t2 + t + 1) 3 3 3

donde:
 2   
1 t +t+1 √ 1 + 2t 6t
I (t) = ln + 2 3 arctan √ + 3 +C
3 (t − 1)2 3 t −1

Ripristinando la variabile x:
 3 (x + 1)2 + 3 (x − 1)2 − 2√
q q
3
1 x2 − 1
I (x) = ln q √ q +
3 3 2 3 2 3 2
(x + 1) + x − 1 − (x − 1)
√ √ ! 
√ 3
x−1+23x+1 p
− 2 3 arctan p − 3 (x2 − 1) (x − 1) + C
3
3
3 (x − 1)
3.14 Integrali di funzioni irrazionali 169

R
11. I (x) = x+3

x2 2x+3
dx. Qui è:

p1 = 1, q1 = 2,

donde:
k=2

La (3.80) porge:
2x + 3 = t2

Risolvendo rispetto a x:
1 2 
x= t −3
2
da cui il differenziale:
dx = tdt

Quindi l’integrale in funzione di t:


Z
t2 + 3
I (t) = 2 dt
(t2 − 3)2

Sviluppiamo l’integrando in frazioni parziali:


t2 + 3 1 6
2 = 2 +
(t2 − 3) t − 3 (t2 − 3)2

L’integrale in funzione di t
2t
I (t) = − +C
t2 −3

Ripristinando la variabile x:

2x + 3
I (x) = − +C
x
q
R 2− 3 2x+1 R h   i
2x+1 1/3 2x+1 1/4
12. I (x) = q
x−1
dx = R x, x−1
, x−1
dx. Qui è k = 6, per
x+4 2x+1
x−1
cui il cambio di variabile è:
2x + 1
= t6
x−1

Risolvendo rispetto a x:
t6 + 1
x=
t6 − 2
3.14 Integrali di funzioni irrazionali 170

Differenziando rispetto a t:

t5 dt
dx = −18
(t6 − 2)

L’integrale diventa:
Z
t7 − 2t5
I (t) = 18 dt,
(t6 − 2) (4t9 + t6 − 8t3 + 1)

che è l’integrale di una funzione razionale propria. Riducendo:


 Z 5 Z 
2 t −t 8t8 + 2t5 − 8t4 + t
I (t) = 18 dt + dt
3 t6 − 2 3 (4t9 + t6 − 8t3 + 1)

Il primo integrale a secondo membro è:


Z 5 " √ !
2 + √
√ #
t −t 1 √ √ 1 + 3
4t2 √ 3
4t2
+ 3
2t4
+ 4 ln t6 − 2
3 3
dt = 2 2 3 arctan √ + 2 ln √
t6 − 2 24 3 3 2
4t − 2
+ C1

Il secondo integrale non si esprime in forma chiusa. Utilizzando il software


Mathematica:
Z 8
8t + 2t5 − 8t4 + t 1 X ln (t − ρ) − 8ρ3 ln (t − ρ) + 2ρ4 ln (t − ρ) + 8ρ7 ln (t − ρ)
dt = ,
4t9 + t6 − 8t3 + 1 6 ρ 6ρ7 + ρ4 − 4ρ

dove la somma è estesa a tutte le radici reali ρ del polinomio 4t9 +t6 −8t3 +1.

3.14.4 Integrali del “tipo 2”


Hanno un espressione generale:
Z
pn (x)
In (x) = √ dx, (3.82)
ax2 + bx + c
essendo pn (x) un assegnato polinomio di grado n:
n
X
pn (x) = ak x k
k=0

Qui è n ≥ 2, poichè per n = 0, 1 l’integrale si calcola con i metodi visti nelle


sezioni precedenti.
3.14 Integrali di funzioni irrazionali 171

Proposizione. L’integrale (3.82) è:


√ Z
dx
In (x) = qn−1 (x) ax2 + bx + c + λ √ , (3.83)
ax2 + bx + c
essendo λ ∈ R, mentre qn−1 (x) è un polinomio di grado n − 1 a coefficienti
indeterminati:
n−1
X
qn−1 (x) = bk xk
k=0

Dimostrazione. Omessa
I coefficienti indeterminati e il numero reale λ si ottengono derivando primo e
secondo membro della (3.83):
n
X
ak x k n−2 n−1
√ X 2ax + b X
√ k=0 = ax2 + bx + c (k + 1) bk+1 x + √ k
bk xk +
ax2 + bx + c k=0
2
2 ax + bx + c k=0
λ
+√ ,
ax2 + bx + c
da cui:

n
X n−2 n−1
k 2
X k
X
2 ak x = 2 ax + bx + c (k + 1) bk+1 x + (2ax + b) bk xk + 2λ (3.84)
k=0 k=0 k=0

Per il principio di identità dei polinomi la n-pla (bn−1 , bn−2 , ..., b0 , λ) è la soluzione
di un sistema di Cramer.
Esempio
Z 4
x + 4x2
I4 (x) = √
x2 + 4
La (3.83) è:
√ Z
dx
I4 (x) = q3 (x) x2 +4+λ √
x2 + 4
Derivando:

x4 + 4x2 √ x (b3 x3 + b2 x2 + b1 x + b0 ) λ
√ = 3b3 x2 + 2b2 x + b1 x2 + 4 + √ +√
x2 + 4 x2 + 4 x2 + 4
Segue il sistema:
3.14 Integrali di funzioni irrazionali 172

4b3 + 0 + 0 + 0 + 0 = 1
0 + 3b2 + 0 + 0 + 0 = 0
12b3 + 0 + 2b1 + 0 + 0 = 4
0 + 8b2 + 0 + b0 + 0 + 0 = 0
0 + 0 + 4b1 + 0 + λ = 0,

la cui unica soluzione è:


 
1 1
(b3 , b2 , b1 , b0 , λ) = , 0, , −2 ,
4 2
donde:

Z
x3 + 2x √ 2 dx
I4 (x) = x +4−2 √
4 x2 + 4
x + 2x √ 2
3  √
2

= x + 4 − 2 ln x + x + 4 + C
4

3.14.5 Calcolare i seguenti integrali


Z Z Z
2 5 6
1) √ x
x2 −x+1
dx 2) √x
1−x2
dx 3) √x
1+x2
dx
Z
4) √ 4x
x2 −4x+2
dx

R 2
1. I (x) = √ x
x2 −x+1
dx. La (3.83) è:
Z √ Z
x2 dx
√ dx = (b1 x + b0 ) x2 − x + 1 + λ √ (3.85)
x2 − x + 1 x2 − x + 1

Derivando primo e secondo membro e riordinando i termini:

2x2 = 4b1 x2 + (2b0 − 3b1 ) x + (−b0 + 2b1 + 2λ)

Per il principio di identità dei polinomi:

2b1 + 0 + 0 = 1 (3.86)
−3b1 + 2b0 + 0 = 0
2b1 − b0 + λ = 0
3.14 Integrali di funzioni irrazionali 173

La soluzione del sistema (3.86) è:


 
1 3 1
(b1 , b0 , λ) = , ,−
2 4 8

Sostituendo nella (3.85):


1 1
I (x) = (2x + 3) − J (x) ,
4 8

essendo: Z
dx
J (x) = √ (3.87)
x2 − x + 1
L’integrale (3.87) si calcola attraverso la (3.46), e si ottiene:

2x − 1 + 2 x2 − x + 1
J (x) = ln
√ + C1

3

= ln 2x − 1 + 2 x2 − x + 1 + C2 ,


avendo incorporato ln 3 nella costante di integrazione. In definitiva:
1 1 √

I (x) = (2x + 3) − ln 2x − 1 + 2 x2 − x + 1 + C
4 8
R 5
2. I (x) = √x
1−x2
dx. La (3.83) è:
Z √ Z
x5 dx
√ dx = q4 (x) 1 − x2 + λ √
1 − x2 1 − x2

Applicando direttamente la (3.84):


3 4
5 2
X k+1
X
2x = 2 1 − x (k + 1) bk+1 x − 2x bk xk + 2λ
k=0 k=0

Si ottiene:
1 4 8
b4 = − , b3 = 0, b2 = − , b1 = 0, b0 = − , λ = 0
5 15 15

da cui:
1 √
I (x) = − 3x4 + 4x2 + 8 1 − x2 + C
15
3.14 Integrali di funzioni irrazionali 174

R 6
3. I (x) = √x
1+x2
dx. La (3.83) è:
Z √ Z
x6 dx
√ dx = q6 (x) 1 + x2 + λ √
1 + x2 1 + x2

Applicando direttamente la (3.84):


4 5
X X
x6 = x2 + 1 (k + 1) bk+1 xk + x bk xk + λ
k=0 k=0

Per il principio di identità dei polinomi:

6b5 + 0 + 0 + 0 + 0 + 0 + 0 = 1
0 + 5b4 + 0 + 0 + 0 + 0 + 0 = 0
5b5 + 0 + 4b3 + 0 + 0 + 0 + 0 = 0
0 + 4b4 + 0 + 3b2 + 0 + 0 + 0 = 0
0 + 0 + 3b3 + 0 + 2b1 + 0 + 0 = 0
0 + 0 + 0 + 2b2 + 0 + b0 + 0 = 0
0 + 0 + 0 + 0 + b1 + 0 + λ = 0

Tale sistema ammette l’unica soluzione:


 
1 5 5 5
(b5 , b4 , b3 , b2 , b1 , b0 , λ) = , 0, − , 0, , 0, − ,
6 24 16 16

donde:
5 5 1
q5 (x) = x − x3 + x5
16 24 6
R √
Ricordando che √ dx
x2 +1
= ln x + x2 + 1 + C1 , si ottiene:

1 √ 5 √
8x5 − 10x3 + 15x x2 + 1 −

I (x) = ln x + x2 + 1 + C
48 16
R
4. I (x) = √ 4x
x2 −4x+2
dx. Si calcola con la (3.46):
√ √
I (x) = 4 x2 − 4x + 2 + 8 ln x − 2 + x2 − 4x + 2 + const

3.14 Integrali di funzioni irrazionali 175

3.14.6 Integrali del “tipo 3”


Hanno un espressione generale:
Z
dx
Jn (x) = √ n (3.88)
(x − x0 ) ax2 + bx + c
Si osservi che per n = 0, 1 l’integrale (3.88) è del tipo (3.46)-(3.47) rispettiva-
mente, donde assumiamo n ∈ N − {0, 1}.
Eseguiamo il cambio di variabile:
1
=t (3.89)
x − x0
L’integrale diventa:
Z
tn−1
Jn (t) = − p dt = −In−1 (t) , (3.90)
αt2 + βt + γ
essendo:

α = ax20 + bx0 + c (3.91)


β = 2ax0 + b
γ=a

e Z
tn−1
In−1 (t) = p dt
αt2 + βt + γ
Abbiamo quindi ricondotto l’integrale al tipo 2, per cui è n ≥ 3. Quindi:

Z
p dt
In−1 (t) = qn−2 (t) αt2 + βt + γ + λ p (3.92)
αt2 + βt + γ
n−3 n−2
n−1 2
X k
X
2t = 2 αt + βt + γ (k + 1) bk+1 t + (2αt + β) bk tk + 2λ
k=0 k=0

3.14.7 Calcolare i seguenti integrali


Z Z Z
x2 +x+1
1) dx

(x+1)3 x2 +2x
2) √dx
x5 x2 −1
3) √
x x2 −x+1
dx
3.14 Integrali di funzioni irrazionali 176

3.14.8 Soluzioni
R
1. dx

(x+1)3 x2 +2x
. Qui è:
α = −1, β = 0, γ = 1

Quindi: Z
t2 dt
J3 (t) = −I2 (t) = − √ (3.93)
1 − t2
Calcoliamo I2 (t):
√ Z
dt
I2 (t) = q1 (t) 1 − t2 + λ √
1 − t2

Per la seconda delle (3.92):



2t2 = 2 1 − t2 b1 + (−2t) (b0 + b1 t) + 2λ

Ciò implica:

−2b1 + 0 + 0 = 1
0 − b0 + 0 = 0
b1 + 0 + λ = 0,

la cui unica soluzione è:


 
1 1
(b1 , b0 , λ) = − , 0,
2 2
R
Osservando che √ dt
1−t2
= arcsin t si ottiene:

1 √ 
I2 (t) = −t 1 − t2 + arcsin t + C2
2

Per la (3.93):
"√ #
1 x2 + 2x 1
J3 (x) = − arcsin +C
2 (x + 1)2 x+1
R
2. √dx
x5 x2 −1
. Qui è:
α = 0, β = 0, γ = 1

Quindi: Z
t4 dt
J5 (t) = −I4 (t) = − √ (3.94)
1 − t2
3.14 Integrali di funzioni irrazionali 177

Calcoliamo I4 (t):
√ Z
dt
I4 (t) = q3 (t) 1 − t2 +λ √
1 − t2
Per la seconda delle (3.92):
  
2t4 = 2 1 − t2 b1 + 2b2 t + 3b3 t2 + (−2t) b0 + b1 t + b2 t2 + b3 t3 + 2λ

Ciò implica:
−4b3 + 0 + 0 + 0 + 0 = 1
0 − 3b2 + 0 + 0 + 0 = 0
3b3 + 0 − 2b1 + 0 + 0 = 0
0 + 2b2 + 0 − b0 + 0 = 0
0 + 0 + b1 + 0 + λ = 0,

la cui unica soluzione è:


 
1 3 3
(b3 , b2 , b1 , b0 , λ) = − , 0, − , 0,
4 8 8
R
Osservando che = arcsin t+const, si ottiene:
√ dt
1−t2

1h 3
√ i
I4 (t) = − 3t + 8t 2
1 − t + arcsin t + C1
8
Per la (3.94):
 
1 2 + 3x2 √ 2 1
J5 (x) = 4
x − 1 − arcsin +C
8 x x
R x2 +x+1
3. √
x x2 −x+1
dx. Spezziamo l’integrale:
Z
x2 + x + 1
F (x) = √ dx
x x2 − x + 1
X3
= Fk (x) ,
k=1

essendo:
Z
xdx
F1 (x) = √ (3.95)
Z −x+1 x2
dx
F2 (x) = √
2
x −x+1
Z
dx
F3 (x) = √
x x2 − x + 1
3.14 Integrali di funzioni irrazionali 178

Calcoliamo separatamente i tre integrali (3.95). Per la (3.46):


√ 1 √

F1 (x) = x2 − x + 1 + ln 2x − 1 + 2 x2 − x + 1 + C1
2



2
F2 (x) = ln 2x − 1 + 2 x − x + 1 + C2

F3 (x) si calcola con la sostituzione x = t−1 ottenendo


Z
dt
F (t) = − √ ,
2
t −t+1

che a parte il segno, è pari a F2 (x), quindi:



F3 (t) = − ln 2t − 1 + 2 t − t + 1 + C3 ,
2

essendo t = x−1 . Ripristinando la variabile x:





2
F3 (x) = ln |x| − ln 2 − x + 2 x − x + 1 + C3

Quindi:
√ 3 √

F (x) = x2 − x + 1 + ln 2 − x + 2 x2 − x + 1 +
2

+ ln |x| − ln 2 − x + 2 x − x + 1 + const
2

3.14.9 Integrali del “tipo 4”


Hanno un espressione generale:
Z
Im,n,p (x) = xm (a + bxn )p dx, (3.96)

essendo m, n, p ∈ Q.

Proposizione (Teorema di Cebyscev) 19. L’integrale (3.96) è esprimibile


attraverso una combinazione finita di funzioni elementari se e solo se è verificata
una delle condizioni:

1. p ∈ Z

2. m+1
n
∈Z
3.14 Integrali di funzioni irrazionali 179


3. m+1
n
+p ∈Z
Nel caso 1 il cambio di variabile è:
x = tq , (3.97)

essendo q il m.c.m. dei denominatori di m e n.


Nel caso 2 il cambio di variabile è:
a + bxn = tk , (3.98)

essendo k il denominatore di p.
Nel caso 3 il cambio di variabile è:
ax−n + b = tk (3.99)

Dimostrazione. Omessa.
Esempio:
Z p
3

1+ 4x
I− 1 , 1 , 1 (x) = √ dx
2 4 3 x
Qui è:
1 1 1
m = − ,n = ,p = ,a = b = 1
2 4 3
Siamo nel caso 2:
1 3
1 + x 4 = t3 =⇒ dx = 12t2 t3 − 1 dt
Ciò implica:
Z
 12
I− 1 , 1 , 1 (t) = 12 t6 − t3 dt = t7 − 3t4 + const
2 4 3 7
Ripristinando la variabile x:
q q
12 √ 7 √ 4
1 + x − 3 1 + 4 x + const
3 3
I− 1 , 1 , 1 (x) = 4
2 4 3 7

3.14.10 Calcolare i seguenti integrali


Z Z Z
2 −3/2 xm dx
1) 3
x (1 + 2x ) dx 2) Fm (x) = per m = −1, 4 3) x4 √dx

4
1+x4 1+x2
Z Z Z
4) √dx
x 3 1+x5
5) x2 (2+x
dx
3 )5/3
6) √ √ 3
dx
√4
3 1+ x3
Z Z √ Z √ x

√ √ 2 √ p4
3
1+ x 4
7) 8) 9)
3
x (1 + 3
x) dx 3
x 1 + x2 dx √
x
dx
3.14 Integrali di funzioni irrazionali 180

3.14.11 Soluzioni
R −3/2
1. x3 (1 + 2x2 ) dx. Qui è m = 3, n = 2, p = − 32 :
Z
−3/2
I3,2,− 3 (x) = x3 1 + 2x2 dx,
2

quindi:
1 tdt
a + bxn = tk ⇐⇒ 1 + 2x2 = t2 =⇒ dx = √ √
2 t2 − 1

L’integrale diventa:
Z  
1 1
I3,2,− 3 (t) = 1 − 2 dt
2 4 t
2
1t +1
= + const
4 t
Ripristinando la variabile x:
1 + x2
I3,2,− 3 (x) = √ + const
2
2 1 + 2x2
R
2. F−1 (x) = √dx
x 4 1+x4
. Qui è m+1
n
= 0, per cui il cambio di variabile:

1 + x4 = t4 ,

per cui: Z
t2
F−1 (t) = dt
(t4 − 1)
Procedendo per decomposizione in frazioni semplici:

1 t + 1 1
F−1 (t) = ln + arctan t + C
4 t − 1 2

Ripristinando la variabile x:

1 4 x−4 + 1 + 1 1 √
4
F−1 (t) = ln √ + arctan x−4 + 1 + C
4 x + 1 − 1 2
4 −4

Procedendo in maniera simile per m = 4:


 q 
√ 4 −4 + 1)3
4 −4
1  x + 1 − 1 √
4
4 (x
F4 (x) = ln √ + 2 arctan x−4 + 1 + q +C
16 4
x−4 + 1 + 1 4 3
(x−4 + 1) − 1
3.14 Integrali di funzioni irrazionali 181

R
3. √dx
x4 1+x2
. Qui è m = −4, n = 2, p = − 21
Z
dx
I−4,2,− 1 (x) = √ ,
2
x4 1 + x2

quindi:
tdt
ax−n + b = tk ⇐⇒ x−2 + 1 = t2 =⇒ dx = −
(t2 − 1)3/2

L’integrale diventa:
Z

I1,4,− 1 (t) = − t2 − 1 dt
4

1
= − t3 + t + const
3

Ripristinando la variabile x:
q

(1 + x2 )3 1 + x2
I1,4,− 1 (x) = − 2
+ + const
4 3x
√ x
1 + x2 
= 2x2 − 1 + const
3x2
R
4. √dx
x 3 1+x5
. Qui è m = −1, n = 5, p = − 13
Z
dx
I−1,5,− 1 (x) = √
3
,
3
x 1 + x5

quindi:

3 t2 dt
a + bxn = tk ⇐⇒ 1 + x5 = t3 =⇒ dx =
5 (t3 − 1)4/5

L’integrale diventa: Z
3 tdt
I−1,5,− 1 (t) =
3 5 t3−1
Abbiamo quindi ricondotto l’integrale 4 all’integrale di una funzione razionale
propria. Riduciamo l’integrando in frazioni semplici:
t A Bt + C
= +
t3 − 1 t − 1 t2 + t + 1
3.14 Integrali di funzioni irrazionali 182

Si ottiene:
A+B+0=0
A−B+C =1
A + 0 − C = 0,

la cui unica soluzione è:


(A, B, C) = (0, 1, 0)

Quindi:
 Z 
3 1 1 t−1
I−1,5,− 1 (t) = ln |t − 1| − + const
3 5 3 3 t2 + t + 1

Calcoliamo l’integrale a secondo membro con la (3.45):


Z  
t−1 1 2
 √ 2t + 1
= ln t + t + 1 − 3 arctan √ + const
t2 + t + 1 2 3
Ripristinando la variabile x:
 

√ √ !
1 3
1 + x 5−1 √ 3
2 1 + x5 + 1 
I−1,5,− 1 (x) =  ln r + 3 arctan √ +C
3 5  q √ 3 
3 (1 + x5 )2 + 3 1 + x5 + 1

R
5. dx
x2 (2+x3 )5/3
. Qui è m = −2, n = 3, p = − 53
Z
dx
I−2,3,− 5 (x) = ,
3
x2 (2 + x3 )5/3

quindi:
!
21/3 t2 dt
ax−n +b = tk ⇐⇒ 2x−3 +1 = t3 =⇒ x= 1/3
, dx = −21/3
(t3 − 1) (t3 − 1)4/3

L’integrale diventa:
Z " 3 2/3 5/3
#
(t − 1) 1 (t3 − 1) t2
I−2,3,− 5 (t) = − · 5/3 · 21/3 dt
3 22/3 2 t5 (t2 − 1)4/3
Z
1 t3 − 1
=− dt
4 t3
 
1 1 −2
=− t+ t +C
4 2
3.14 Integrali di funzioni irrazionali 183

Ripristinando la variabile x:
" #
1/3
1 (2 + x3 ) 1 x2
I−2,3,− 5 (x) = − + +C
3 4 x 2 (2 + x3 )2/3
4 + 3x3
=− +C
8x (2 + x3 )2/3
R
6. √ √
3
dx

4
. Qui è m = − 32 , n = 43 , p = − 13
x3 1+ x3
Z
dx
I− 3 , 3 ,− 1 (x) = √ p 3

4
,
2 4 3
x3 1 + x3
quindi:
t2 dt
ax−n + b = tk ⇐⇒ x−3/4 + 1 = t3 =⇒ dx = −4
(t3 − 1)7/3
L’integrale diventa:
Z
I−2,3,− 5 (t) = −4 tdt
3

= −2t2 + C

Ripristinando la variabile x:
r
3
√4
2
x3 + 1
I−2,3,− 5 (x) = −2 √ +C
3 x
R√ √ 2
7. x (1 + 3
x) dx. Qui è m = 12 , n = 31 , p = 2
Z
√ √ 2
I 1 , 1 ,2 (x) = x 1 + 3 x dx,
2 3

quindi:
x = t6 =⇒ dx = 6t5 dt
L’integrale diventa:
Z

I 1 , 1 ,2 (t) = 6 t12 + 2t10 + t8 dt
2 3
 
1 13 2 11 1 9
=6 t + t + t + const
13 11 9
Ripristinando la variabile x:
 
1 13/6 2 11/6 1 3/2
I 1 , 1 ,2 (x) = 6 x + x + x + const
2 3 13 11 9
3.14 Integrali di funzioni irrazionali 184

R√ p
4

8. x 1 + x2 dx. Qui è m = 13 , n = 32 , p = 41
3 3

Z q √
√ 4 3
I 1 , 2 , 1 (x) = 3
x 1 + x2 dx,
3 3 4

Abbiamo:
m+1
= 1,
n
donde il cambio di variabile è
 3/2 1/2 
n k 2/3 4 4 3 4
a+bx = t ⇐⇒ 1+x = t =⇒ x = t − 1 , dx = 6t t − 1 dt

L’integrale diventa:
Z

I 1 , 1 ,2 (t) = 6 t4 t4 − 1 dt
2 3
 
1 9 1 5
=6 t − t + const
9 5

Ripristinando la variabile x:
" r #
14  √ 9 1 r √ 5
3 4 3
I 1 , 1 ,2 (x) = 6 1 + x2 − 1 + x2 +C
2 3 9 5
r
√ 5 5√3
x2 − 4
4 3
=6 1+ x 2 +C
45
R √
3 √
1+ 4 x
9. √
x
dx. Qui è m = − 12 , n = 41 , p = 31
Z p 3

1+ 4x
I− 1 , 1 , 1 (x) = √ dx,
2 4 3 x

Abbiamo:
m+1
= 2,
n
donde il cambio di variabile è
4 3 
n k 1/4 3 3 2 3
a + bx = t ⇐⇒ 1 + x = t =⇒ x = t − 1 , dx = 12t t − 1 dt

L’integrale diventa:
Z

I 1 , 1 ,2 (t) = 12 t3 t3 − 1 dt
2 3
 
1 7 1 4
= 12 t − t +C
7 4
3.15 Esercizi riepilogativi sugli integrali di funzioni irrazionali 185

Ripristinando la variabile x:
 q 
13 √ 7 1 q
3 √ 4
I 1 , 1 ,2 (x) = 12 1+ x −
4
1+ x
4
+C
2 3 7 4
q
33 √ 7 √ 
= 1+ 4x 44x−3 +C
7

3.15 Esercizi riepilogativi sugli integrali di fun-


zioni irrazionali
3.15.1 Calcolare i seguenti integrali
Z √
Z Z Z
x2 +4x+4+x2 4 1/2
1) x+1
dx 2) dx

x−2 3 x+4
3) x −3
(1 + x ) dx 4) √3
dx
x+x3
Z √
Z Z Z √
x
5) 1+x
dx 6) √
dx

x(1+ x)
7) dx

3+ x+2
8) 1− √3x+2 dx
1+ 3x+2
Z Z Z Z √
4x−x2
9) √ dx
x2 −x+1
10) √ dx
x x2 +x−1
11) √ dx
2 12) x3
dx
Z Z Z p6+x−x Z
√ √
13) dx
(x+1)1/2 +(x+1)1/4
14) √dx
x2 4−x2
15) 1 + xdx 16) x2 1 − xdx
Z √
Z √
Z Z
1− 3 x 3
17) √ √ dx 18) √ 1− x+1
√ dx 19) √x2 −3x+2
dx
20) √ xdx
x+ 4 x x+1+ 3 x+1 −x2 +x+2

Z *** Z Z
√ √ 3 +x
21) x2 + 4x + 13dx 22) −x2 − x + 1dx 23) √−xx4 +3x 2 −2 dx
Z Z √ Z

4 x

24) x+√ 3
x+2
x+2
dx 25) √x+1+x dx 26) −x2 − 14x + 17dx
Z Z Z
p x
27) √ 6x−5
x2 −12x+52
dx 28) x−2
dx 29) x√x2dx+x+1

3.15.2 Soluzioni
R √x2 +4x+4+x2
1. I (x) = x+1
dx. Si riduce facilmente all’integrale di una funzione
razionale impropria:
Z
(x + 2) + x2
I (x) = dx
x+1
Z  
2
= x+ dx
x+1
1
= x2 + 2 ln |x + 1| + const
2
R R 
2. I (x) = x−2dx 3 x+4 =
√ R x, x1/3 dx, cioè è del tipo 1 con (p1 , q1 ) = (1, 3),
donde:
x = t3
3.15 Esercizi riepilogativi sugli integrali di funzioni irrazionali 186

L’integrale diventa:
Z
t2 dt
I (t) = 3 3
t − 2t + 4
Z
t2 dt
=3
(t + 2) (t2 − 2t + 2)

Riduciamo in frazioni semplici:


t2 A Bt + C
2
= + 2
(t + 2) (t − 2t + 2) t + 2 t − 2t + 2

cioè:
(A + B) t2 + (−2A + 2B + C) t + 2A + 2C = t2

Per il principio di identità dei polinomi:


A+B+0=1 (3.100)
−2A + 2B + C = 0
2A + 0 + 2C = 0

Il sistema (3.100) è di Cramer, e risolvendo:


 
2 3 2
(A, B, C) = , ,−
5 5 5

Quindi:  Z 
2 dt 1
I (t) = 3 + J (t) ,
5 t+2 5
essendo: Z
3t − 2
J (t) = dt,
t2 − 2t + 2
che si calcola con la (3.45):
3 
J (t) = ln t2 − 2t + 2 + arctan (t − 1) + const
2
Quindi:
 
2 3  1
I (t) = 3 ln |t + 2| + ln t − 2t + 2 + arctan (t − 1) + const
2
5 10 5

Ripristinando la variabile x:
 
3 √ 3 √ 3 √  √ 
I (x) = 2 ln x + 2 + ln
3 2
x − 2 x + 2 + arctan x − 1 +const
3 3

5 2
3.15 Esercizi riepilogativi sugli integrali di funzioni irrazionali 187

R 1/2
3. x−3 (1 + x4 ) dx. È del tipo 4 con m = −3, n = 4, p = 1
2
Z
1/2
I−3,4, 1 (x) = x−3 1 + x4 dx,
2

Abbiamo:
m+1
+ p = 0,
n
donde il cambio di variabile è
 
−n k −4 2
1/4
2 1 2 −5/4
ax +b = t ⇐⇒ x +1 = t =⇒ x= t −1 , dx = − t t − 1 dt
2

L’integrale diventa:
Z
1 t2
I3,4, 1 (t) = − dt
2 2 t2 − 1
 Z 
1 dt
=− t+ dt
2 t2 − 1
 
1 1 t + 1
=− t + ln +C
2 2 t − 1

Ripristinando la variabile x:
√ √ !
1 x4 + 1 1 x4 + 1 + x2
I−3,4, 1 (x) = − − ln √ +C
2 2 x2 2 x4 + 1 − x2

R R −1/3
4. √
3
dx
x+x3
= x−1/3 (1 + x2 ) dx. È del tipo 4 con m = − 31 , n = 2, p = − 31
Z

I− 1 ,2,− 1 (x) = x−1/3 1 + x2 dx,
3 3

Abbiamo:
m+1
+ p = 0,
n
donde il cambio di variabile è
 
−n k −2 3
−1/2
3 3 2 3 −3/2
ax +b = t ⇐⇒ x +1 = t =⇒ x= t −1 , dx = − t t − 1 dt
2

L’integrale diventa:
Z
3 t
I− 1 ,2,− 1 (t) = − dt
3 3 2 t3 −1
3.15 Esercizi riepilogativi sugli integrali di funzioni irrazionali 188

Riduciamo in frazioni semplici:


t A Bt + C
= +
t3 − 1 t − 1 t2 + t + 1
A (t2 + t + 1) + (Bt + C) (t − 1)
=
t3 − 1
(A + B) t2 + (A − B + C) t + A − C
=
t3 − 1

Per il principio di identità dei polinomi:

A+B+0=0 (3.101)
A−B+C =1
A+0−C =0

Il sistema (3.101) è di Cramer, quindi risolviamolo con l’omonima regola:



1 1 0

∆ = 1 −1 1 = 3
1 0 −1

0 1 0

∆A = 1 −1 1 = 1
0 0 −1

1 1 0

∆B = 1 −1 1 = −1
1 0 0

1 1 0

∆C = 1 −1 1 = 1
1 0 0

Quindi:
∆A 1
A= =
∆ 3
∆B 1
B= =−
∆ 3
∆C 1
C= =
∆ 3

L’integrale diventa:
 Z 
1 t−1
I− 1 ,2,− 1 (t) = − ln |t − 1| + C1 − 2
3 3 2 t +t+1
3.15 Esercizi riepilogativi sugli integrali di funzioni irrazionali 189

L’integrale a secondo membro si calcola con la (3.45), ottenendo:


Z  
t−1 1 2
 √ 2t + 1
= ln t + t + 1 − 3 arctan √ + C2
t2 + t + 1 2 3
Sostituendo nella precedente:
√  
1 1 2
 3 2t + 1
I− 1 ,2,− 1 (t) = − ln |t − 1| + ln t + t + 1 − arctan √ + const
3 3 2 4 2 3
Ripristinando la variabile x:
1 √ 1 q p

3 −2 3 −2 2 −2
I− 1 ,2,− 1 (x) = − ln x + 1 − 1 + ln (x + 1) + (x + 1) + 1
3 3 2 4
√ √ !
3 2 3 x−2 + 1 + 1
− arctan √ +C
2 3
R √
x
5. I (x) = 1+x
dx. Il cambio di variabile è

x = t =⇒ dx = 2tdt

donde:
Z
t2
I (t) = 2 2 dt
t +1
Z 2
t +1−1
=2 dt
t2 + 1
Z Z 
dt
=2 dt −
t2 + 1
= 2 (t − arctan t) + const

Ripristinando la variabile x:
√ √ 
I (x) = 2 x − arctan x + const
R
6. I (x) = √
dx
√ .
x(1+ x)
Il cambio di variabile è

x = t =⇒ dx = 2tdt

donde:
Z
dt
I (t) = 2
t+1
= 2 ln |t + 1| + const

Ripristinando la variabile x:

I (x) = 2 ln x + 1 + const
3.15 Esercizi riepilogativi sugli integrali di funzioni irrazionali 190

R
7. I (x) = dx

3+ x+2
. Eseguiamo il cambio di variabile:

x + 2 = t =⇒ dx = 2tdt

donde:
Z
tdt
I (t) = 2
t+3
= 2 (t − 3 ln |t + 3|) + const

Ripristinando la variabile x:
h√ √ i
I (x) = 2 x + 2 − 3 ln x + 2 + 3 + const

R √
1−√3x+2
8. I (x) = 1+ 3x+2
dx. Eseguiamo il cambio di variabile:
 
√ 1 2  2
3x + 2 = t =⇒ x= t − 2 , dx = tdt
3 3

donde:
Z
2 t − t2
I (t) = dt
3 1+t
Z  
2 2
= −t + 2 − dt + C1
3 1+t
 
2 1 2
= − t + 2t − 2 ln |1 + t| + C1
3 2

Ripristinando la variabile x:
4 h√  √ i
I (x) = −x + 3x + 2 − ln 1 + 3x + 2 + C
3
R
9. I (x) = √ dx
x2 −x+1
. Si calcola direttamente con la (3.46):

I (x) = ln 2x − 1 + 2 x2 − x + 1 + const

R
10. I (x) = √ dx
x x2 +x−1
. Eseguiamo il cambio di variabile:

1
t= ,
x
3.15 Esercizi riepilogativi sugli integrali di funzioni irrazionali 191

ottenendo: Z
dt
I (t) = − √ ,
−t2 +t+1
con la (3.46):
1 − 2t
I (t) = arcsin √ + const
5
Ripristinando la variabile x:
x−2
I (x) = arcsin √ + const
x 5
R
11. I (x) = √ dx
6+x−x2
. Si calcola direttamente con la (3.46):

1 − 2x
I (x) = − arcsin + const
5
2x − 1
= arcsin + const
5
R √
4x−x2
12. I (x) = x3
dx. Può essere scritto come:
Z

I (x) = x−5/2 4 − xdx
Z
= x−m (a + bxn )p dx

Risulta:
m+1
+ p = −1,
n
per cui il cambio di variabile è:

4x−1 − 1 = t2 ,

da cui:
4 8tdt
x= , dx = − 2
t2 +1 (t + 1)
L’integrale diventa:
Z
5/2 2t (−8) · t
I (t) = 4−5/2 t2 + 1 1/2 2 dt
(t2 + 1) (t2 + 1)
Z
−5/2
= −16 · 4 t2 dt
16 −5/2 3
= ·4 t + const
3
3.15 Esercizi riepilogativi sugli integrali di funzioni irrazionali 192

Ripristinando la variabile x:
q
(4x − x2 )3
I (x) = − + const
6x3
R R h 1/2 1/4
i
13. I (x) = (x+1)1/2dx
+(x+1)1/4
= R (x + 1) , (x + 1) , quindi è del tipo 1
con q1 = 2, q2 = 4, donde è k = 4. Il cambio di variabile è:

(x + 1)4 = t

L’integrale diventa:
Z 2
t dt
I (t) = 4
t+1
Z  
1
=4 t−1+ dt
t+1
 
1 2
=t t − t + ln |t + 1| + const
2

Rirpristinando la variabile x:
 
1 1/2 1/4 1/4
(x + 1) − (x + 1) + ln (x + 1) + 1 + const

I (x) = 4
2
R
14. I (x) = √dx
x2 4−x2
. Eseguiamo il cambio di variabile:

2
x=
t

L’integrale diventa:
Z
1 tdt
I (t) = − √
4 t2 − 1
1√ 2
=− t − 1 + const
4

Ripristinando la variabile x:

4 − x2
I (x) = − + const
4x
3.15 Esercizi riepilogativi sugli integrali di funzioni irrazionali 193

Rp √
15. I (x) = 1 + xdx. È del tipo 4 con m = 0, n = 1/2, p = 1/2, per cui il
cambio di variabile è:
1 + x1/2 = t2 ,

differenziando: 
dx = 4t t2 − 1 dt

L’integrale diventa:
Z

I (t) = 4 t2 t2 − 1 dt
 
1 5 1 3
=4 t − t + const
5 3

Ripristinando la variabile x:
 
1 1/2 5/2
 1 1/2 3/2

I (x) = 4 1+x − 1+x + const
5 3
q
4 √ 3 √ 
= 1 + x 3 x − 2 + const
15
R √
16. I (x) = x2 1 − xdx. È del tipo 4 con m = 2, n = 1, p = 1/2, per cui:

1 − x = t2

L’integrale diventa:
Z

I (t) = −2 t6 − 2t4 + t2 dt
 
1 7 2 5 1 3
= −2 t − t + t + const
7 5 3

Ripristinando la variabile x:
q
2 
I (x) = − (1 − x)3 15x2 + 12x + 8 + const
105
R √ R 
1− 3 x
17. I (x) = √ √ dx
x+ 4 x
= R x1/3 , x1/2 , x1/4 , quindi il cambio di variabile è:

x = t12
3.15 Esercizi riepilogativi sugli integrali di funzioni irrazionali 194

L’integrale diventa:
Z 8
t (1 − t4 )
I (t) = 12 dt
t3 + 1
Z  
2 3 5 6 9 t−1
= 12 1−t −t +t +t −t + 2 dt
t −t+1
 
1 3 1 4 1 6 1 6 1 7 1 10
= 12 t − t − t + t + t + t − t + J (t) ,
3 4 6 6 7 10

essendo:
Z
t−1
J (t) =
t2 − t + 1
 
1  1 2t − 1
= ln t2 − t + 1 − √ arctan √ + C1
2 3 3

Sostituendo nella precedente e ripristinando la variabile x:


√ √ √ √ √
12 12
I (x) = 12 12 x − 4 4 x − 3 3 x + 2 x + x7
7 √
6√6 √ √  12 2 12
x−1
− x5 + 6 ln x − x + 1 − √ arctan
6 12
√ +C
5 3 3
R 1− √ 3
x+1
R h 1/2 1/3
i
18. I (x) = √ √
x+1+ 3 x+1
dx = R (x + 1) , (x + 1) dx, quindi il cambio
di variabile è:
x + 1 = t6

L’integrale diventa:
Z  
3 4
 1 4 1 5
I (t) = 6 t −t dt = 6 t − t + const
4 5

Rirpristinando la variabile x:
q q
33 2 66
I (x) = (x + 1) − (x + 1)5 + const
2 5
R
19. I (x) = √x2 −3x+2
dx
. Si calcola direttamente con la (3.46), ottenendo:



2
I (x) = ln 2x − 3 + 2 x − 3x + 2
R
20. I (x) = √ xdx
−x2 +x+2
. Si calcola direttamente con la (3.46), ottenendo:

1 1 − 2x √ 2
I (x) = arcsin − −x + x + 2 + const
2 3
3.15 Esercizi riepilogativi sugli integrali di funzioni irrazionali 195

R√
21. I (x) = x2 + 4x + 13dx. Si calcola direttamente con la (3.48), ottenen-
do:
x + 2√ 2 9 √
x + 4x + 13 + ln x + 2 + x2 + 4x + 13 + const

I (x) =
2 2
R√
22. I (x) = −x2 − x + 1dx. Si calcola direttamente con la (3.48), ottenen-
do:
2x + 1 √ 2 5 2x + 1
I (x) = −x − x + 1 + arcsin √ + const
4 8 5
R 3 +x
23. I (x) = √−xx4 +3x 2 −2 dx. Può essere scritto come:

Z
1 x2 + 1 
I (x) = √ d x2 ,
2 −x4 + 3x2 − 2

per cui è conveniente eseguire il cambio di variabile:


y = x2 ,

donde: Z
1 y+1
I (y) = p dy,
2 −y 2 + 3y − 2
che si calcola con la (3.46), ottenendo:
5 1p 2
I (y) = arcsin (2y − 3) − −y + 3y − 2 + const
4 2
Ripristinando la variabile x:
5  1√
I (x) = arcsin 2x2 − 3 − −x4 + 3x2 − 2 + const
4 2
R x+ √4
x+2
R h 1/3 1/4
i
24. I (x) = √
3
x+2
dx = R x, (x + 2) , (x + 2) dx, per cui il cambio
di variabile è:
x + 2 = t12

Differenziando rispetto a t:
dx = 12t11 dt

L’integrale diventa:
Z

I (t) = 12 t19 + t10 − 2t7 dt
 20 
t t11 1 8
= 12 + − t + const
20 11 4
3.15 Esercizi riepilogativi sugli integrali di funzioni irrazionali 196

Ripristinando la variabile x:
 
2/3 1 1 1/11
I (x) = 12 (x + 2) (x − 1) + (x + 2) + const
20 11
R √
x
25. I (x) = √
x+1+x
dx. Indicando con f (x) la funzione integranda:
√ √
x x+1−1
f (x) = √ ·√
x+1+1 x+1−1
√ √ 
x x+1−1
=
r x
x+1 1
= −√ ,
x x

donde:
Z r Z
x+1 dx
I (x) = dx − √ (3.102)
x x

= J (x) − 2 x + const,

essendo: Z r
x+1
J (x) = dx
x
Per il calcolo di J (x) eseguiamo il cambio di variabile:
r
x+1
= t,
x

da cui:
1 2tdt
x= , dx = −
t2 −1 (t − 1)2
2

Quindi: Z
t2
J (t) = −2 dt
(t2 − 1)2
Riduciamo l’integrando in frazioni semplici:

t2 t2
=
(t2 − 1)2 (t − 1)2 (t2 + 1)
 
1 1 1 1 1
= + + −
4 (t − 1)2 t − 1 (t + 1)2 t + 1
3.15 Esercizi riepilogativi sugli integrali di funzioni irrazionali 197

Integrando:  
1 t + 1 2t
J (t) = ln + + const
2 t − 1 t2 − 1
Ripristinando la variabile x e sostituendo nella (3.102):
r √ √
x+1 √ 1 x + 1 + x
I (x) = x − 2 x + ln √ √ + const
x 2 x + 1 − x
R√
26. I (x) = −x2 − 14x + 17dx. Si calcola direttamente con la (3.48):
x + 7√ 2 x+7
I (x) = −x − 14x + 17 + 33 arcsin √ + const
2 66
R
27. I (x) = √ 6x−5
x2 −12x+52
dx. Si calcola direttamente con la (3.46):

√ x − 6 + x2 − 12x + 52
I (x) = 6 x2 − 12x + 52 + 31 ln + const
4
Rp
28. I (x) = x
x−2
dx. Eseguiamo il cambio di variabile:
x
= t2
x−2

donde:
2t2 4t
x= 2
, dx = − 2 dt
t −1 t −1
L’integrale diventa: Z
t2 dt
I (t) = −4
(t2 − 1)2
Sviluppiamo l’integrando in frazioni semplici:
 
t2 1 1 1 1 1
= + + −
(t2 − 1)2 4 t − 1 (t − 1)2 (t + 1)2 t + 1

Da cui l’integrale:

2t t + 1
+ const
I (t) = 2 + ln
t −1 t − 1

Ripristinando la variabile x:
√ √
p x + x − 2
I (x) = x (x − 2) + ln √
√ + const
x − x − 2
3.16 Integrali di funzioni trigonometriche 198

R
29. I (x) = √ dx
x x2 +x+1
. Si calcola con la (3.47). Il cambio di variabile è:

1
ξ= ,
x

da cui:
Z

I (ξ) = − p
ξ2 + ξ + 1
 p √ 
= − ln 2ξ + 1 + 2 ξ 2 + ξ + 1 − ln 3 + const


Ripristinando la variabile x e incorporando ln 3 nella costante di inte-
grazione:
x
+ const
I (x) = ln

x + 2 + 2 x2 + x + 1

3.16 Integrali di funzioni trigonometriche


3.16.1 Integrali del “tipo 1”
Hanno la seguente espressione:
Z
In1 ,n2 (x) = (sin x)n1 (cos x)n2 dx, (3.103)

essendo n1 , n2 ∈ Z.
Consideriamo ni entrambi positivi. Lo schema di calcolo per In1 ,n2 (x) è legato
alla partità di n1 , n2 . Più precisamente, il caso più immediato è quello in cui ni
è dispari. Senza perdita di generalità, supponiamo che n1 sia dispari:

∃k ∈ N : n1 = 2k + 1,
donde:

Z
In1 (k),n2 (x) = (sin x)2k (cos x)n2 sin xdx
Z
k
=− 1 − cos2 x (cos x)n2 d (cos x)

Eseguiamo il cambio di variabile:

y = cos x (3.104)
Quindi:
3.16 Integrali di funzioni trigonometriche 199

Z
k
In1 (k),n2 (y) = − 1 − y2 y n2 dy,

che si risolve facilmente. Ad esempio:


Z
I (x) = sin3 x cos4 xdx
Z

=− 1 − cos2 x cos4 xd (cos x)

Il cambio di variabile (3.104):


Z

I (y) = − 1 − y 2 y 4 dy
1 1
= − y 5 + y 7 + const
5 7
Ripristinando la variabile x:
1 1
I (x) = − cos5 x + cos7 x + const
5 7
Nel caso n2 = 2k + 1, il cambio di variabile è y = sin x:
Z
In1 ,n2 (k) (x) = (sin x)n1 (cos x)2k cos xdx
Z
k
= (sin x)n1 1 − sin2 x d (sin x)

Cambiando la variabile:
Z
k
In1 ,n2 (k) (y) = y n1 1 − y 2 dy,

che si risolve facilmente.


***
Se n1 e n2 sono entrambi pari, si cerca di trasformare l’integrando utilizzando le
formule trigonometriche:

1
sin2 x =
(1 − cos 2x)
2
1
cos2 x = (1 + cos 2x)
2
1
sin x cos x = sin 2x
2
3.16 Integrali di funzioni trigonometriche 200

Ad esempio:

Z
I (x) = cos2 3x sin4 3xdx
Z
= (cos 3x sin 3x)2 sin2 3xdx
Z
sin2 6x 1 − cos 6x
= · dx
4 2
Z
1 
= sin2 6x − sin2 6x cos 6x dx
8
Z  
1 1 − cos 12x 2
= − sin 6x cos 6x dx
2 2
 
1 x 1 1 3
= − sin 12x − sin 6x + C
2 2 24 18
1 
= 36x − 3 sin 12x − 4 sin3 6x + C
576
***

Consideriamo ora il caso in cui n1 , n2 ≤ 0 : (n1 , n2 ) 6= (0, 0). L’integrale (3.103)


si scrive:
Z
dx
In1 ,n2 (x) = (3.105)
(sin x)|n1 | (cos x)|n2 |
Ricordiamo le formule:

1 1
2 =1+ (3.106)
sin x tan2 x
1
2
= 1 + tan2 x,
cos x
da cui:

1
(sin x)|n1 | =  |n21 |
1
1+ tan2 x
dx  |n22|−2
= 1 + tan2 x d (tan x)
(cos x)|n2 |
Sostituendo in (3.105):
Z   |n21 |
1  |n22|−2
In1 ,n2 (x) = 1+ 1 + tan2 x d (tan x)
tan2 x
3.16 Integrali di funzioni trigonometriche 201

Eseguiamo il cambio di variabile:

y = tan x
Quindi:
|n1 |+|n2 |
Z −1
(y 2 + 1) 2
In1 ,n2 (y) = dy (3.107)
y |n1 |
Osservazione. La (3.107) è valida anche se n1 , n2 sono numeri razionali.

Alcuni esempi: Z
dx
I (x) =
cos4 x
Qui è:

|n1 | = 0, |n2 | = 4,
donde la (3.107):

Z

I (y) = y 2 + 1 dy
y3
= +y+C
3
Ripristinando la variabile x:
1
I (x) = tan3 x + tan x + C
3
Consideriamo ora:
Z
dx
I (x) =
sin3 x
Conviene riscrivere I (x) nella forma:
Z
1 dx
I (x) =
8 sin3 x2 cos3 x
2
Poniamo:
x
ξ=
2
Ciò implica:
Z
1 dξ
I (ξ) = 3
4 sin ξ cos3 ξ
3.16 Integrali di funzioni trigonometriche 202

Qui è:

|n1 | = |n2 | = 3,
per cui:

Z 2
1 (y 2 + 1)
I (y) = dy
4 y3
Z Z 
1 2y 2 + 1
= ydy + dy
4 y3
 
1 y2 1
= + 2 ln |y| − 2 + C
4 2 2y

Ripristinando la variabile x:
1h 2x x  x i
+ 4 ln tan − cot2

I (x) = tan +C
8 2 2 2
Z Z
dx
3.16.1.1 (sin x)n
, (cosdxx)n

Questi integrali possono essere calcolati attraverso un procedimento ricorsivo (qui


è n ≥ 2). Iniziamo con il primo integrale:

Z
dx
Fn (x) =
(sin x)n
Z
sin2 x + cos2 x
= dx
(sin x)n
= Fn−2 (x) + Hn (x) ,

essendo:
Z
cos2 x
Hn (x) = dx
(sin x)n
Osserviamo che
 
d 1 cos x
n−1 = − (n − 1)
dx (sin x) (sin x)n
Quindi:
3.16 Integrali di funzioni trigonometriche 203

Z
cos x
Hn (x) = cos x dx
(sin x)n
Z  
1 1
=− cos xd
n−1 (sin x)n−1

Eseguendo un’integrazione per parti nell’ultimo integrale:


1 cos x 1
Hn (x) = − n−1 − Fn−2 (x)
n − 1 (sin x) n−1
Finalmente:
 
1 cos x
Fn (x) = (n − 2) Fn−2 (x) − (3.108)
n−1 (sin x)n−1
Attraverso la formula ricorrente (3.108) è possibile determinare Fn (x) per asseg-
nati valori di n (ved. Appendice). Passiamo all’integrale contenente il coseno.
Poniamo:

Z
dx
Gn (x) =
(cos x)n
Z
sin2 x + cos2 x
= dx
(cos x)n
= Gn−2 (x) + Kn (x) ,

essendo:
Z
sin2 x
Kn (x) = dx
(cos x)n
Osserviamo che
 
d 1 sin x
n−1 = (n − 1)
dx (cos x) (cos x)n
Quindi:

Z
sin x
Kn (x) = sin x dx
(cos x)n
Z  
1 1
= sin xd
n−1 (cos x)n−1

Eseguendo un’integrazione per parti nell’ultimo integrale:


3.16 Integrali di funzioni trigonometriche 204

1 sin x 1
Kn (x) = n−1 − Gn−2 (x)
n − 1 (cos x) n−1
Finalmente
 
1 sin x
Gn (x) = (n − 2) Gn−2 (x) + (3.109)
n−1 (cos x)n−1
Attraverso la formula ricorrente (3.109) è possibile determinare Gn (x) per asseg-
nati valori di n (ved. Appendice).

Osservazione. Le (3.108)-(3.109) sono inapplicabili per n = 1. A tale valore


corrispondono due integrali notevoli:
 x 

F1 (x) = ln tan +C
 x π2 

G1 (x) = ln tan + +C
2 4

3.16.2 Integrali del “tipo 2”


Per definizione:
Z
In (x) = (tan x)n dx, (3.110)

essendo n ∈ N. Per n = 0, 1 il calcolo è immediato:

I0 (x) = x + C
I1 (x) = − ln |cos x| + C
1 
= ln 1 + tan2 x + C
2
Per n ≥ 2:
Z
In (x) = (tan x)n−2 tan2 xdx (3.111)

Sostituendo la seconda delle (3.106) nella (3.111):


Z  
n−2 1
In (x) = (tan x) − 1 dx
cos2 x
Cioè:
1
In (x) = (tan x)n−1 − In−2 (x) (3.112)
n−1
3.16 Integrali di funzioni trigonometriche 205

Procedendo in maniera analoga per l’integrale:


Z
Jn (x) = (cot x)n dx,

si giunge:
1
(cot x)n−1 − Jn−2 (x)
Jn (x) = (3.113)
1−n
In Appendice 5 sono esplicitati gli integrali In (x), Jn (x) per alcuni valori di n.

3.16.3 Esercizi
Z Z Z Z
1) 3
cos xdx 2) 5
sin xdx 3) 2
sin x cos xdx 3
4) sin3 x2 cos5 x2 dx
Z Z Z Z
cos5 x
5) sin3 x
dx 6) 4
sin xdx 7) 4
cos xdx 8) sin2 x cos2 xdx
Z Z Z Z
9) 2
sin x cos xdx 4
10) 6
cos 3xdx 11) dx
sin4 x
12) dx
cos6 x
Z Z Z Z
cos2 x
13) sin6 x
dx 14) dx
sin2 x cos4 x
15) dx
sin5 x cos3 x
16) sin x
dx
cos3 x
2 2
Z Z Z Z
sin( )
x+ π4
17) sin x cos x
dx 18) dx
sin5 x
19) tan2 5xdx 20) dx
cos5 4x
Z Z Z Z
cos3 x √
21) sin4 x
dx 22) x sin x dx2 2
23) 5
sin x cos xdx
3
24) √ dx
sin x cos3 x

3.16.4 Soluzioni
R R R 
1. I (x) = cos3 xdx = cos2 x cos xdx = 1 − sin2 x d (sin x)
= sin x − 31 sin3 x + C
R
2. I (x) = sin5 xdx. Abbiamo:
Z
I (x) = sin4 x sin xdx
Z
2
=− 1 − cos2 x d (cos x)

Il cambio di variabile y = cos x implica:


Z
2
I (y) = − 1 − y 2 dy
 
2 3 1 5
=− y− y + y +C
3 5
3.16 Integrali di funzioni trigonometriche 206

Ripristinando la variabile x:
2 1
I (x) = cos3 x − cos x − cos5 x + C
3 5
R
3. I (x) = sin2 x cos3 dx. Abbiamo:
Z
I (x) = sin2 x cos2 x cos xdx
Z

= sin2 x 1 − sin2 x d (sin x)

Il cambio di variabile y = sin x implica:


Z

I (y) = y 2 1 − y 2 dy
1 1
= y3 − y5 + C
3 5

Ripristinando la variabile x:
1 3 1
I (x) = sin x − sin5 x + C
3 5
R
4. I (x) = sin3 x2 cos5 x2 dx. Poniamo:
x
y=
2

Quindi:
Z
I (ξ) = 2 sin3 y cos5 ydy
Z

= 2 sin3 y 1 − sin2 y d (sin y)

Il cambio di variabile ξ = sin y implica:


Z
2
I (ξ) = ξ 3 1 − ξ 2 dξ
 
1 8 1 6 1 4
=2 ξ − ξ + ξ +C
8 3 4

Ripristinando la variabile x:
1 h x x  x i
I (x) = 3 sin8 − 8 sin6 + 6 sin4 +C
12 2 2 2
3.16 Integrali di funzioni trigonometriche 207

R cos5 x
5. I (x) = sin3 x
dx. Abbiamo:
Z
cos4 x
I (x) = d (sin x)
sin3 x
Z 2
1 − sin2 x
= d (sin x)
sin3 x

Ponendo y = sin x:
Z 2
(1 − y 2 )
I (y) = dy
y3
Z  
2 1
= y − + 3 dy
y y
1 1
= y 2 − 2 ln |y| − 2 + C
2 2y

Ripristinando la variabile x:
1 2 1
I (x) = sin x − 2 ln |sin x| − +C
2 2 sin2 x
R
6. I (x) = sin4 xdx. Abbiamo:

1
sin4 x = (1 − cos 2x)2
4

Sostituendo:
1
I (x) = [x − sin 2x + J (x)] ,
4
essendo: Z Z
2 1
J (x) = cos 2xdx = cos2 2xd (2x)
2
Dalla seconda delle (3.5):
1
J (x) = (4x + sin 4x) + const
8

Quindi:
1
I (x) = (12x − 8 sin 2x + sin 4x) + const
32
3.16 Integrali di funzioni trigonometriche 208

R
7. I (x) = cos4 xdx. Abbiamo:

1
cos4 x = (1 + cos 2x)2
4

Sostituendo:
1
I (x) = [x + sin 2x + J (x)] ,
4
essendo: Z
1
J (x) = cos2 2xdx = (4x + sin 4x) + const
8
Quindi:
1
I (x) = (12x + 8 sin 2x + sin 4x) + C
32
R
8. I (x) = sin2 x cos2 xdx. Abbiamo:

1 2
sin2 x cos2 x = sin 2x,
4

donde: Z
1
I (x) = sin2 2xd (2x)
8
Dalla prima delle (3.5):
1
I (x) = (4x − sin 4x) + C
32
R
9. I (x) = sin2 x cos4 xdx. Risulta:
Z Z
2 4 sin2 2x 1
sin x cos xdx = · (1 + cos 2x) dx
4 2
Z Z 
1 2 2
= sin 2xdx + sin 2x cos 2xdx
8

Poniamo:
Z Z
2
I1 (x) = sin 2xdx, I2 (x) = sin2 2x cos 2xdx

Calcoliamo I1 (x):
Z
1
I1 (ξ) = sin2 ξdξ, con ξ = 2x
2
3.16 Integrali di funzioni trigonometriche 209

Quindi:
Z  
1 1 1
I1 (ξ) = (1 − cos 2ξ) = ξ − sin 2ξ + C1
4 4 2
1 1
I1 (x) = x − sin 4x + C1
2 8

Calcoliamo I2 (x):
Z
1 1 3
I2 (x) = sin2 2xd (sin 2x) = sin 2x + C2
2 6
da cui:
1 
I (x) = 12x − 3 sin 4x + 4 sin3 2x
192
R
10. I (x) = cos6 3xdx. Poniamo y = 3x:
Z
1
I (y) = cos6 ydy
3

Sviluppiamo l’integrando:
1
cos6 y = (1 + cos 2y)3
8
1 
= cos3 2y + 3 cos2 2y + 3 cos 2y + 1
8

Quindi:  
1 3 ′
I (y) = y + sin 2y + 3F1 (y) + F2 (y) + C ,
24 2
essendo:
Z
F1 (y) = cos2 (2y) dy
y 1
= + sin 4y + C1
Z2 8
F2 (y) = cos3 (2y) dy
 
1 1 3
= sin 2y − sin 2y + C2
2 3

Semplificando e ripristinando la variabile x:


1  
I (x) = 48 sin 6x + 180x + 9 sin 12x − 4 sin3 6x + C
576
3.16 Integrali di funzioni trigonometriche 210

R
11. I (x) = sindx4 x . Si calcola attraverso una formula di ricorrenza (ved. Ap-
pendice). Alternativamente, procediamo nel seguente modo:
Z
1
I (x) = − d (cot x)
sin2 x

Dalla prima delle (3.106):


1
2 = 1 + cot2 x,
sin x

donde:
Z

I (y) = − 1 + y 2 dy
1
= −y − y 3 + C,
3

essendo y = cot x.
1
I (x) = − cot x − cot3 x + C
3
R
12. I (x) = dx
cos6 x
. Abbiamo:
Z
1
I (x) = d (tan x)
cos4 x

Dalla seconda delle (3.106):


1 2
4
= 1 + tan2 x ,
cos x

donde:
Z
2
I (y) = 1 + y2 dy
1 2
= y 5 + y 3 + y + C,
5 3

essendo y = tan x.
1 2
I (x) = tan5 x + tan3 x + tan x + C
5 3
3.16 Integrali di funzioni trigonometriche 211

R cos2 x
13. I (x) = sin6 x
dx. Abbiamo:
Z
cos2 x
I (x) = − d (cot x)
Z  sin4 x
cos x 2 1
=− d (cot x)
sin x sin2 x

Tenendo conto della prima delle (3.106):


Z

I (y) = − y 2 + y 4 dy
1 1
= − y 3 − y 5 + C,
3 5
essendo y = cot x.
1 1
I (x) = − cot3 x − cot5 x + C
3 5
R
14. I (x) = dx
sin2 x cos4 x
. Applicando la (3.107):
Z 2
(y 2 + 1)
I (y) = dy
y2
Z  
2 1
= y + 2 + 2 dy
y
1 1
= y 3 − + 2y + C
3 y
1 1
I (x) = tan3 x − + 2 tan x + C
3 tan x
R
15. I (x) = dx
sin5 x cos3 x
. Applicando la (3.107):
Z 3
(y 2 + 1)
I (y) = dy
y5
Z  
3 3 1
= y + + 3 + 5 dy
y y y
1 3 1
= y + 3 ln |y| − 2 − 4 + C
2 2y 4y
1 3 1
I (x) = tan2 x + 3 ln |tan x| − − +C
2 2 tan2 x 4 tan4 x
R
16. I (x) = sin
dx
x
cos3 x . Prima di applicare la (3.107) eseguiamo il cambio di
2 2
variabile:
x
ξ= ,
2
3.16 Integrali di funzioni trigonometriche 212

per cui: Z

I (ξ) = 2
sin ξ cos3 ξ
La (3.107) si scrive:

Z
(y 2 + 1)
I (y) = dy
y
 x  x
+ tan2

= 2 ln tan +C
2 2

R sin(x+ π )
17. I (x) = sin x cos4x dx. Sviluppando il numeratore con le formule di addizione
degli archi:
√ Z
2 sin x + cos x
I (x) = dx
2 sin x cos x
√ Z Z 
2 dx dx
= +
2 cos x sin x

Gli integrali a secondo membro dell’equazione precedente, compongono una


coppia di integrali notevoli dati dalla (2.4) che qui riscriviamo:
Z
dx 1
= ln
− cot x + C1
sin x sin x
Z
dx 1
= ln + tan x + C1
cos x cos x

Quindi:
√  
2 1 1
I (x) = ln + tan x + ln − cot x + C
2 cos x sin x
R
18. I (x) = dx
sin5 x
. Abbiamo:
Z
3/2
I (x) = − 1 + cot2 x d (cot x)

Con l’usuale cambio di variabile y = cot x:


Z q
I (y) = − (1 + y 2 )3 dy
3.16 Integrali di funzioni trigonometriche 213

Tale integrale può essere risolto ponendo y = cosh t:


Z
I (t) = − sin4 tdt

Utilizzando la nota relazione:


1
sin2 t = (cosh 2t − 1)
2
si ha:
Z
1
I (t) = − (cosh 2t − 1)2
4
Z 
1 2
=− cosh (2t) dt − sinh (2t) + t + C
4

Calcoliamo a parte l’integrale a secondo membro:


Z Z 
2 1
cosh (2t) dt = (cosh (4t) dt + t)1
2
1 t
= sinh (4t) + + C1
8 2
Quindi:
1 1 3
I (t) = − sinh (4t) + sinh (2t) − t + C,
32 4 8
con t = arccosh(cot x).
R
19. I (x) = tan2 5xdx. Poniamo:
ξ = 5x,

per cui:
Z
1
I (ξ) = tan2 ξdξ
5
Z  
1 1
= − 1 dξ
5 cos2 ξ
1
= tan 5x − x + C
5
R
20. I (x) = cosdx5 4x . Dopo aver eseguito il cambio di variabile ξ = 4x conviene

applicare la formula ricorsiva esplicitata in Appendice 4, ottenendo:


    
1 3 π  sin 4x sin 4x
I (x) = ln tan 2x + + + +C
16 2 4 cos2 4x cos4 4x
3.16 Integrali di funzioni trigonometriche 214

R cos3 x
R R 
21. I (x) = sin4 x
dx = cos x
cos2 x sin 4 x dx = cos2 xd − 13 sin13 x ; integrando per
parti:
cos2 x 2 1
I (x) = − 3 + +C
3 sin x 3 sin x
R
22. I (x) = x sin2 x2 dx. Abbiamo:
Z
1 
I (x) = sin2 x2 d x2
2

Eseguendo il cambio di variabile y = x2 :


Z
1
I (y) = sin2 ydy,
2

che è un integrale notevole (eq. 3.5):


1
I (y) = (2y − sin 2y) + C
8
Ripristinando la variabile x:
1 
2x2 − sin 2x2 + C
I (x) =
8
R √ R 2√
23. I (x) = sin5 x 3 cos xdx = − (1 − cos2 x) 3 cos xd (cos x). Eseguendo il
cambio di variabile y = cos x:
Z
2
I (y) = − y 1/3 1 − y 2 dy
3 3 3
= y 10/3 − y 16/3 − y 4/3 + C
5 16 4
Quindi:
3√3 3√3 3√ 3
I (x) = − cos4 x + cos10 x − cos16 x + C
4 5 16
24. Qui è:
Z
1
I (x) = d (tan x)
(sin x) (cos x)−1/2
1/2

Z  1/4
1 1
= 1+ 2 1/4
d (tan x)
tan x (1 + tan2 x)
Z
= (tan x)−1/2 d (tan x)

= 2 tan x + C
3.16 Integrali di funzioni trigonometriche 215

3.16.5 Integrali del “tipo 3”


Sono integrali del tipo:
Z
fm (x) fn (x) dx, (3.114)

essendo fm (x) una funzione sin, cos

fm (x) = sin mx, cos mx


Si trasforma l’integrando della (3.114) in una somma procedendo poi per decom-
posizione. A tale scopo si utilizzano le note formule trigonometriche:

1
sin mx cos nx = [sin (m + n) x + sin (m − n) x] (3.115)
2
1
sin mx sin nx = [cos (m − n) x − cos (m + n) x]
2
1
cos mx cos nx = [cos (m − n) x + cos (m + n) x]
2
Ad esempio:

Z
I (x) = sin 4x cos 12xdx
Z Z 
1
= sin 16xdx − sin 8xdx
2
1
= (2 cos 8x − cos 16x) + C
32

3.16.6 Esercizi
Z Z Z
 
1) sin 3x cos 5xdx 2) sin 10x sin 15xdx 3) cos x
2
cos x
3
dx
Z Z Z
 
4) sin 3 cos 3 x dx 5) cos (ax + b) cos (ax − b) dx
x 2
6) sin (ωt) sin (ωt + φ) dt
Z Z
7) cos x cos 3xdx
2
8) sin x sin 2x sin 3xdx

3.16.7 Soluzioni
R R R 
1. I (x) = sin 3x cos 5xdx = 1
2
sin 8xdx − sin 2xdx
1
= 16
(4 cos 2x − cos 8x) + C
3.16 Integrali di funzioni trigonometriche 216

R R R 
2. I (x) = sin 10x sin 15xdx = 1
2
cos 5xdx − cos 25xdx
1
= (5 sin 5x − sin 25x) + C
50
R  
3. I (x) = cos x2 cos x3 dx = 3 sin 16 x + 35 sin 65 x + C
R   
4. I (x) = sin x3 cos 32 x dx = 12 3 cos x3 − cos x + C
R
5. I (x) = cos (ax + b) cos (ax − b) dx. Poniamo: α = ax + b, β = ax − b,
donde l’integrando:
1
cos α cos β = [cos (α − β) + cos (α + β)]
2
1
= [cos (2b) + cos (2ax)]
2

Quindi:
1
I (x) = [2ax cos (2b) + sin (2ax)] + C
4a
R
6. I (t) = sin (ωt) sin (ωt + φ) dt. Poniamo: α = ωt, β = ωt + φ, donde
l’integrando:
1
sin α sin β = [cos (α − β) − cos (α + β)]
2
1
= [cos φ − cos (2ωt + φ)]
2

Quindi:
 
1 1
I (t) = t cos φ − sin (2ωt + φ) + C
2 2ω
1
= [2ωt cos φ − sin (2ωt + φ)] + C

R
7. I (x) = cos x cos2 3xdx
Z
I (x) = cos x cos 3x cos 3xdx
Z
1
= (cos 2x + cos 4x) cos 3xdx
2
Z Z 
1
= cos 2x cos 3xdx + cos 4x cos 3xdx
2
   
1 1 1 1
= sin x + sin 5x + C1 + sin x + sin 7x + C2
2 5 2 7
1
= (70 sin x + 7 sin 5x + 5 sin 7x) + C
140
3.16 Integrali di funzioni trigonometriche 217

R
8. I (x) = sin x sin 2x sin 3xdx
Z
1
I (x) = (cos x − cos 3x) sin 3xdx
2
Z Z 
1
= cos x sin 3xdx − cos 3x sin 3xdx
2
 Z Z 
1 1 1
= (sin 4x + sin 2x) dx − sin 6xdx
2 2 2
1
= (2 cos 6x − 3 cos 4x − 6 cos 2x) + C
48

3.16.8 Integrali del “tipo 4”


Sono integrali del tipo:
Z
R (sin x, cos x) dx, (3.116)

essendo R una funzione razionale. Il cambio di variabile è:


x
x → t = tan , (3.117)
2
da cui:
 
2t 1 − t2
R (sin x, cos x) → R , ,
1 + t2 1 + t2
e il differenziale:
2dt
dx =
1 + t2
Ad esempio:
Z
dx
I (x) = ,
sin x + cos x + 1
diventa:
Z
dt
I (t) = = ln |1 + t| + C
1+t
Ripristinando x:
x

I (x) = ln 1 + tan + C
2
***
3.16 Integrali di funzioni trigonometriche 218

Se R è una funzione pari:

R (− sin x, − cos x) ≡ R (sin x, cos x) ,


il cambio di variabile è

x → t = tan x, (3.118)
donde:
 
t 1
R (sin x, cos x) → R √ ,√ ,
1+t2 1 + t2
e il differenziale:
dt
dx =
1 + t2
Ad esempio:
Z
dx
I (x) = ,
1 + cos2 x
diventa:
Z √  
dt 2 t
I (t) = = arctan √ +C
2 + t2 2 2
cioè:
√  
2 tan x
I (x) = arctan √ +C
2 2

3.16.9 Esercizi
Z Z Z
1) dx
3+5 cos x
2) dx
sin x+cos x
3) cos x
1+cos x
dx
Z Z Z
4) sin x
1−sin x
dx 5) dx
8−4 sin x+7 cos x
6) dx
cos x−2 sin x+3
Z Z Z
7) 3 sin x+2 cos x
2 sin x+3 cos x
dx 8) 1+tan x
1−tan x
dx 9) dx
1+3 cos2 x
Z Z Z
10) dx
3 sin2 x+5 cos2 x
11) dx
sin2 x+3 sin x cos x−cos2 x
12) dx
sin2 x−5 sin x cos x
Z Z Z
13) sin x
(1−cos x)3
dx 14) sin 2x
1+sin2 x
dx 15) cos 2x
cos4 x+sin4 x
dx
Z Z Z
16) cos x
sin2 x−6 sin x+5
dx 17) dx
(2−sin x)(3−sin x)
18) 1−sin x+cos x
1+sin x−cos x
dx
3.16 Integrali di funzioni trigonometriche 219

3.16.10 Soluzioni
R
1. I (x) = dx
3+5 cos x
. Qui è:

1
R (cos x) = ,
3 + 5 cos x

donde il cambio di variabile è (3.117). L’integrale diventa:


Z
dt 1 2 + t
I (t) = = ln +C
4 − t2 4 2 − t

Cioè: 
x
1 2 + tan 2 
I (x) = ln x
+C
4 2 − tan 2
R
2. dx
sin x+cos x
.
Z
x dt
x → t = tan =⇒ I (t) = − 2
2 t − 2t − 1
Applicando direttamente la (3.45) troviamo:
t + 1 + √2

1
I (t) = √ ln √ +C
2 2 t − 1 − 2

Cioè: √
x
1
tan 2
+ 1 + 2
I (x) = √ ln √ +C
2 2 tan x2 − 1 − 2
R
3. I (x) = cos x
1+cos x
dx.
Z  
1
I (x) = 1− dx
1 + cos x
= x − J (x) + const,

essendo: Z
dx
J (x) =
1 + cos x
Eseguiamo il cambio di variabile:
x
x → t = tan ,
2
3.16 Integrali di funzioni trigonometriche 220

donde:
Z
1 dt
J (t) = 2
1−t2 1 + t2
1 + 1+t2
Z
= dt = t + const,

quindi:
x
J (x) = tan + const
2
Cosicché l’integrale vale:
x
I (x) = x − tan + const
2
R R
4. I (x) = sin x
1−sin x
dx = sin x−1+1
1−sin x
dx = −x + J (x), essendo:
Z
dx
J (x) =
1 − sin x

Eseguiamo il cambio di variabile:


x
x → t = tan ,
2

donde:
Z
1 2dt
J (t) = 2t
1 − 1+t2 1 + t2
Z
dt 2
=2 2 = − + const,
(t − 1) t−1

quindi:
2
J (x) = − x + const
tan − 1
2

Cosicché l’integrale vale:


2
I (x) = −x − x + const
tan − 1 2

R
5. I (x) = dx
8−4 sin x+7 cos x
. Eseguiamo il cambio di variabile:
x
x → tan
2
3.16 Integrali di funzioni trigonometriche 221

Quindi: Z
dt
I (t) = 2
t2 − 8t + 15
L’integrale suddetto si risolve con la (3.45):

t − 5
I (t) = ln + C,
t − 3

da cui:
tan x2 − 5
I (x) = ln +C
tan x2 − 3
R
6. I (x) = dx
cos x−2 sin x+3
. Eseguiamo il cambio di variabile:
x
x → tan
2

Quindi: Z
dt
I (t) =
t2 − 2t + 2
L’integrale suddetto si risolve con la (3.45):

I (t) = arctan (t − 1) + C,

da cui: 
x
I (x) = arctan tan − 1 + C
2
R
7. I (x) = 32 sin x+2 cos x
sin x+3 cos x
dx. Osserviamo che il numeratore si può esprimere
come combinazione lineare del denominatore e della sua derivata prima:
d
3 sin x + 2 cos x = A (2 sin x + 3 cos x) + B (2 sin x + 3 cos x) (3.119)
dx

Quindi:
I (x) = Ax + B ln |2 sin x + 3 cos x| + C,

essendo C l’usuale costante di integrazione.


Dalla (3.119) si ottiene il sistema:

2A − 3B = 3
3A + 2B = 2,
3.16 Integrali di funzioni trigonometriche 222

quindi i coefficienti indeterminati A e B:


12 5
A= , B=−
13 13

e l’integrale:
12 5
I (x) = x− ln |2 sin x + 3 cos x| + C
13 13
R R R d(cos x−sin x)
8. I (x) = 1+tan x
1−tan x
dx = sin x+cos x
cos x−sin x
dx =− cos x−sin x
= − ln |cos x − sin x| + C
R
9. I (x) = 1+3dx cos2 x
. Osserviamo che:
Z
dx
I (x) =
1 + 3 cos2 x
Z
1 dx
= 1
cos2 x
+ 3 cos2 x

Ma:
1
= 1 + tan2 x
cos2 x
dx
= d (tan x) ,
cos2 x

donde eseguendo il cambio di variabile x → y = tan x:


Z
dy
I (x) =
4 + y2
Z 
1 d y2
= 
2 1+ y 2
2
 
1 tan x
= arctan +C
2 2
R
10. I (x) = dx
3 sin2 x+5 cos2 x
. Osserviamo che:
Z
dx
I (x) = 2 2
Z 3 sin x + 5 cos x
dx 1
= 2
3 tan x + 5 cos2 x
Z
1
= d (tan x) ,
5 + 3 tan2 x
3.16 Integrali di funzioni trigonometriche 223

donde eseguendo il cambio di variabile x → y = tan x:


Z
dy
I (x) =
5 + 3y 2
q 
Z d 3
1 5
y
=√ q 2
15 3
1+ 5
y
r !
1 3
= √ arctan tan x + C
15 5

R
11. I (x) = dx
sin2 x+3 sin x cos x−cos2 x
. Risulta:
Z
dx
I (x) = 2
cos2 x (tan x + 3 tan x − 1)

Eseguendo il cambio di variabile x → y = tan x:


Z
dy
I (y) =
y 2 + 3y − 1
2y + 3 − √13

1
= √ ln √ + C,
13 2y + 3 + 13

cioè:
2 tan x + 3 − √13

1
I (x) = √ ln √ +C
13 2 tan x + 3 + 13
R
12. I (x) = dx
sin2 x−5 sin x cos x
. Risulta:
Z
dx
I (x) = 2
sin x (1 − 5 cot x)

Eseguendo il cambio di variabile x → y = cot x:


Z
dy
I (y) =
5y − 1
1
= ln |5y − 1| + C,
5

cioè:
1 5 − tan x
I (x) = ln +C
5 tan x
3.16 Integrali di funzioni trigonometriche 224

R
13. I (x) = sin x
(1−cos x)3
dx. Risulta:
Z
d (cos x)
I (x) = −
(1 − cos x)3

Eseguendo il cambio di variabile x → y = cos x:


Z
d (1 − y)
I (y) =
(1 − y)3
1
=− +C
2 (1 − y)2

Cioè:
1
I (x) = − +C
2 (1 − cos x)2
R
14. I (x) = sin 2x
1+sin2 x
dx. Risulta:
Z
sin x cos x
I (x) = 2 2 dx
Z 1 + sin x
sin x
=2 d (sin x)
1 + sin2 x
Eseguendo il cambio di variabile x → y = sin x:
Z
ydy
I (y) = 2
1 + y2
Z
d (1 + y 2 )
=
1 + y2

= ln 1 + y 2 + C

Cioè: 
I (x) = ln 1 + sin2 x + C
R
15. cos 2x
cos4 x+sin4 x
dx. Risulta:
Z
cos2 x − sin2 x
I (x) = dx
cos4 x + sin4 x
Z
1 − tan2 x
= d (tan x)
1 + tan2 x
Eseguendo il cambio di variabile x → y = tan x:
Z
1 − y2
I (y) = dy (3.120)
1 + y4
= I1 (y) − I2 (y) ,
3.16 Integrali di funzioni trigonometriche 225

essendo:
Z
dy
I1 (y) =
1 + y4
Z 2
y dy
I2 (y) =
1 + y4

L’integrale I1 (y) è stato già calcolato precedentemente (eq. 3.65):


y 2 + √2y + 1
" #
1  √   √ 
I1 (y) = √ −2 arctan 1 − 2y + 2 arctan 1 + 2y + ln √
4 2 2
y − 2y + 1
+const

Per il secondo integrale procediamo per decomposizione in frazioni semplici:


y2 Ay + B Cy + D
4
= √ + √
1+y y 2 + 2y + 1 y 2 − 2y + 1

Risolvendo il conseguente sistema di Cramer:


 
1 1
(A, B, C, D) = − √ , 0, √ , 0 ,
2 2 2 2

donde:
1 1
I2 (y) = − √ J1 (y) + √ J2 (y) ,
2 2 2 2
essendo:
Z
ydy
J1 (y) = √
y 2 + 2y + 1
Z
ydy
J2 (y) = √ ,
2
y − 2y + 1

che si calcolano con la (3.45):


1 2 √ √ 
2y + 1 + const

J1 (y) = ln y + 2y + 1 − arctan
2
1 2 √
√ 
2y − 1 + const

J1 (y) = ln y − 2y + 1 + arctan
2
Quindi:
y 2 + √2y + 1

1 1 h  √   √ i
I2 (y) = − √ ln √ + √ arctan 1 + 2y − arctan 1 − 2y
4 2 y 2 − 2y + 1 2 2
+const
3.16 Integrali di funzioni trigonometriche 226

Sostituendo nella (3.120):


y 2 + √2y + 1

1
+ const

I (y) = √ ln √
2 2 2
y − 2y + 1

Cioè:
tan2 x + √2 tan x + 1

1
+ const

I (x) = √ ln 2

2 2 tan x − 2 tan x + 1

1 2 + sin 2x
+ const

= √ ln √
2 2 2 − sin 2x

R R d(sin x)
16. cos x
sin2 x−6 sin x+5
dx = sin2 x−6 sin x+5
. Eseguendo il cambio di variabile x →
y = sin x:
Z
dy
I (y) =
y 2 − 6y + 5

1 y − 5
= ln +C
4 y − 1

Da cui:
1 sin x − 5
I (x) = ln +C
4 sin x − 1
R
17. I (x) = (2−sin x)(3−sin
dx
x)
. Riduciamo l’integrando in frazioni semplici (rispet-
to a sin x):
1 A B
= +
(2 − sin x) (3 − sin x) 2 − sin x 3 − sin x
(3A + 2B) + (−A − B) sin x
=
(2 − sin x) (3 − sin x)

Deve essere:

A+B =0
3A + 2B = 1

Risolvendo:
(A, B) = (1, −1)

Quindi:
1 1 1
= −
(2 − sin x) (3 − sin x) 2 − sin x 3 − sin x
3.16 Integrali di funzioni trigonometriche 227

L’integrale:
I (x) = I1 (x) − I2 (x) ,

essendo:
Z
dx
I1 (x) =
2 − sin x
Z
dx
I2 (x) =
3 − sin x

Risolviamo I1 (x):
Z  
x dt 2 2t − 1
x → t = tan =⇒ I1 (t) = 2
= √ arctan √ + const
2 t −t+1 3 3

Perciò:  
2 2 tan x2 − 1
I1 (x) = √ arctan √ + const
3 3
Risolviamo I2 (x):
Z  
x dt 1 3t − 1
x → t = tan =⇒ I2 (t) = 2 = √ arctan √ + const
2 3t2 − 2t + 3 2 2 2

Perciò:  
1 3 tan x2 − 1
I1 (x) = √ arctan √ + const
2 2 2
L’integrale I (x):
   
2 2 tan x2 − 1 1 3 tan x2 − 1
I (x) = √ arctan √ + √ arctan √ +C
3 3 2 2 2
R
18. I (x) = 1−sin x+cos x
1+sin x−cos x
dx. Prima di eseguire il cambio di variabile x → t =
tan 2 , semplifichiamo l’integrando:
x

Z  
2
I (x) = −1 + dx
1 + sin x − cos x
= −x + 2J (x) ,

essendo: Z
dx
J (x) =
1 + sin x − cos x
3.16 Integrali di funzioni trigonometriche 228

Cambiando la variabile:
Z
dt
J (t) =
t (t + 1)
Z  
1 1
= − dt
t t+1

t
= ln
+C
t + 1

da cui I (x):
tan x2
I (x) = −x + 2 ln +C
tan x + 1
2

3.16.11 Esercizi riepilogativi sugli integrali trigonometrici


Z Z Z
1) 4
cos 2x sin 2xdx3
2) 3 5
sin 3x cos 3xdx 3) sin2 3x cos5 3xdx
Z Z Z
4) cosn nx dx (n > 1) 5) 4 2
sin 3x cos 3xdx 6) sin 3x sin 2xdx
Z Z Z

7) 1 − cos xdx 8) (1 + cos 3x)3/2 dx 9) √ dx
1−sin 2x
Z Z Z
10) tan3 3x sec4 3xdx 11) tan2 x sec3 xdx 12) tan3 2x sec3 2xdx
Z Z Z
13) 4
cot 3x csc 3xdx 14) 3 5
cot x csc xdx 15) sin4 2xdx
Z
16) sin7 xdx

R
1. I (x) = cos4 2x sin3 2x. Abbiamo:
Z
1 
I (x) = − cos4 2x 1 − cos2 2x d (cos 2x)
2

Poniamo y = cos 2x:


Z
1 
I (y) = y 4 y 2 − 1 dy
2
 
1 1 7 1 5
= y − y +C
2 7 5

Cioè:
1 1
I (x) = cos7 2x − cos5 2x + C
14 10
3.16 Integrali di funzioni trigonometriche 229

R
2. I (x) = sin3 3x cos5 3xdx. Abbiamo:
Z

I (x) = 1 − sin2 3x cos5 3x sin 3xdx

Poniamo y = cos 3x:


Z
1 
I (y) = − y 5 1 − y 2 dy
3
 
1 1 6 1 8
=− y − y +C
3 6 8

Cioè:
1 1
I (x) = cos8 3x − cos6 3x + C
24 18
R
3. I (x) = sin2 3x cos5 3xdx. Abbiamo:
Z
1 2
I (x) = sin2 3x 1 − sin2 3x d (sin 3x)
3

Poniamo y = sin 3x:


Z
1 2
I (y) = y 2 1 − y 2 dy
3
 
1 1 3 2 5 1 7
= y − y + y +C
3 3 5 7

Cioè:  
1 1 3 2 5 1 7
I (x) = sin 3x − sin 3x + sin 3x + C
3 3 5 7
R
4. In (x) = cosn nx dx.
Z 
x n−1 x
In (x) = n cos cos dx
n n
Z   n−1  x
2 x 2
=n 1 − sin d sin
n n

Eseguendo il cambio di variabile x → y = sin nx :


Z
 n−1
In (y) = n 1 − y 2 2 dy
3.16 Integrali di funzioni trigonometriche 230

Ad esempio, per n = 3:
Z

I3 (y) = 3 1 − y 2 dy
 
1 3
= 3 y − y + C,
3

per cui:
x x
I3 (x) = 3 sin − sin3 + C
3 3
R
5. I (x) = sin4 3x cos2 3xdx. Abbiamo:
Z
I (x) = (sin 3x cos 3x)2 sin2 3xdx

Osservando che:
1
sin 3x cos 3x = sin 6x
2
1
sin2 3x = (1 − cos 6x) ,
2
si ha:
Z
1
I (x) = sin2 6x (1 − cos 6x) dx
8
1
= [I1 (x) − I2 (x)] ,
8
essendo:
Z Z  
2 1 1 1
I1 (x) = (1 − cos 12x) dx =
sin 6xdx = x− sin 12x + C1
2 2 12
Z Z
2 1 1
I2 (x) = sin 6x cos 6xdx = sin2 6xd (sin 6x) = sin3 6x + C2 ,
6 18

per cui:
x 1 1
I (x) = − sin 12x − sin3 6x + C
16 144 192
R
6. I (x) = sin 3x sin 2xdx . Per le (3.115):
1
sin 3x sin 2x = (cos x − cos 5x) ,
2
donde:
1 1
I (x) = sin x − sin 5x + C
2 10
3.16 Integrali di funzioni trigonometriche 231

R√
7. I (x) = 1 − cos xdx. Osservando che:
x
cos x = 1 − 2 sin2 , (3.121)
2
segue:
x
1 − cos x = 2 sin2 ,
2
donde:
√ Z x √ x
I (x) = 2 sin dx = −2 2 cos + C
2 2
R
8. I (x) = (1 + cos 3x)3/2 dx. Dalla (3.121):
y
cos y = 2 cos2 − 1
2
Se poniamo y = 3x:
 
2 3
cos 3x = 2 cos x − 1,
2

donde:  
3/2
√ 3
(1 + cos 3x) = 2 2 cos3 x
2
L’integrale diventa:
   
√ Z 2 3 3
I (x) = 2 2 cos x cos x dx
2 2
Z      
4√ 2 3 3
= 2 1 − sin x d sin x
3 2 2
   
4√ 3 2 3
= 2 sin x 3 − sin x +C
9 2 2
R R
9. I (x) = √1−sin
dx
2x
= q dx
. Poniamo:
1−cos( π2 −2x)

π
y = − 2x
2
donde:
Z
1 dy
I (y) = − √
2 1 − cos y
Z
1 dy
=− √ y
2 2 sin 2

1 1
= − √ ln y − cot y + C

2 sin 2
3.16 Integrali di funzioni trigonometriche 232

Ripristinando la variabile x:

2 1 π 

I (x) = − ln  − cot −x +C
2 sin π4 − x 4
R
10. I (x) = tan3 3x sec4 3xdx. Poniamo 3x = ξ:
Z
1 dξ
I (ξ) = tan3 ξ 4 (3.122)
3 cos ξ
Z
1 1
= tan3 ξ 2 d (tan ξ)
3 cos ξ
Ma
1 sin2 ξ + cos2 ξ
=
cos2 ξ cos2 ξ
2
= tan ξ + 1

Quindi la (3.122) diventa:


Z
1 
I (ξ) = tan3 ξ tan2 ξ + 1 d (tan ξ)
3
Esegiamo il cambio di variabile ξ → y = tan ξ:
Z
1 
I (y) = y 3 y 2 + 1 dy
3
 
1 1 6 1 4
= y + y + const
3 6 4
Ritornando alla variabile x:
 
1 1 6 1 4
I (x) = tan 3x + tan 3x + C
6 3 2
R R R
11. I (x) = tan2 x sec3 xdx = cosdx5 x − cosdx3 x = G5 (x)−G2 (x), essendo Gn (x)
espresso dalla (3.109). Abbiamo:
 
1 sin x
G3 (x) = G1 (x) + + const
2 cos2 x
  x π  
1 sin x
+ const

= ln tan + +
2 2 4 cos2 x
 
1 sin x
G5 (x) = 3G3 (x) + + const,
4 cos4 x
donde:
1 sin x
I (x) = − G3 (x) + +C
4 4 cos4 x
3.16 Integrali di funzioni trigonometriche 233

R
12. I (x) = tan3 2x sec3 2xdx. Poniamo ξ = 2x:
Z
1 dξ
I (ξ) = tan3 ξ 3
2 cos ξ
Z
1 1 − cos2 ξ
=− d (cos ξ)
2 cos6 ξ

Eseguiamo il cambio di variabile y = cos ξ:


Z Z 
1 −4 −6
I (y) = y dy − y dy
2
 
1 1 −3 1 −5
= − y + y +C
2 3 5

Ritornando alla variabile x:


 
1 1 1
I (x) = − +C
2 5 cos 2x 3 cos3 2x
5

R R
13. I (x) = cot 3x csc4 3xdx = − 13 cot 3x
sin2 3x
d (cot 3x). Osservando che:

1
= 1 + cot2 3x,
sin2 3x

si ha:
Z
1 
I (x) = − cot 3x 1 + cot2 3x d (cot 3x)
3
1 
= − cot2 3x 2 + cot2 3x + C
12
R
14. I (x) = cot3 x csc5 xdx. Sviluppiamo l’integrando:

cos2 x
cot3 x csc5 xdx = dx
sin8 x
d (sin x) d (sin x)
= − ,
sin8 x sin6 x

donde:
Z Z
d (sin x) d (sin x)
I (x) = 8 −
sin x sin6 x
1 1
=− + +C
7 sin x 5 sin5 x
7
3.17 Integrazione delle funzioni iperboliche 234

R
15. I (x) = sin4 2xdx. Abbiamo (ξ = 2x):
Z
1
I (ξ) = sin4 ξdξ
2
Z  
1 2 1
= sin ξd (2ξ − sin 2ξ)
2 4
 
1 2 2 1 1 1 1
= 2ξ sin ξ − sin ξ sin 2ξ + ξ cos 2ξ − cos 2ξ − sin 2ξ + ξ − sin 4ξ + C
8 2 2 2 8

Ripristinando la variabile x:
 
1 2 2 1 1
I (x) = 4x sin 2x − sin 2x sin 4x + 2x cos 4x − sin 4x + x − sin 8x + C
8 2 8
1 3 3
= − sin3 2x cos 2x − cos 2x sin 2x + x + C
8 16 8
R R 3
16. I (x) = sin7 xdx = − (1 − cos2 x) d (cos x). Eseguiamo il cambio di
variabile x → y = cos x:
1 3
I (y) = y 7 − y 5 + y 3 − y + C
7 5

Ripristinando la variabile x:
1 3
I (x) = cos7 x − cos5 x + cos3 x − cos x + C
7 5

3.17 Integrazione delle funzioni iperboliche


Il punto di partenza è la relazione fondamentale:

cosh2 x − sinh2 x = 1 (3.123)


La prima relazione utile si ottiene dalla formula di duplicazione:

cosh 2x = 2 sinh2 x + 1, (3.124)


risolvendo rispetto a sinh2 x:
1
sinh2 x = (cosh 2x − 1) (3.125)
2
La (3.125) è utilizzabile quando l’integrando contiene sinh2 x.
Tenendo conto della (3.123), la (3.125) diventa:
1
cosh2 x = (cosh 2x + 1) (3.126)
2
3.17 Integrazione delle funzioni iperboliche 235

Infine dalla:

sinh 2x = 2 sinh x cosh x,


si ottiene:
1
sinh 2x,
sinh x cosh x = (3.127)
2
utilizzabile quando l’integrando contiene sinh x cosh x.
Esempio 1
Calcoliamo:

Z
I (x) = cosh2 xdx
Z
1
= (cosh 2x + 1) dx
2
 Z Z 
1 1
= cos 2xd (2x) + dx
2 2
 
1 1
= sinh 2x + x + C
2 2
1 1
= x + sinh 2x + C
2 4
Esempio 2
Calcoliamo:

Z
I (x) = cosh3 xdx
Z
= cosh2 xd (sinh x)
Z

= sinh2 x + 1 d (sinh x)
1
= sinh3 x + sinh x + C
3

3.17.1 Esercizi
Z Z Z
3 4
1) sinh xdx 2) cosh xdx 3) sinh3 x cosh xdx
Z Z Z
2 2
4) sinh x cosh xdx 5) dx
sinh x cosh2 x
6) dx
sinh2 x cosh2 x
Z
7) tanh3 xdx
3.17 Integrazione delle funzioni iperboliche 236

R R R 
1. I (x) = sinh3 xdx = sinh2 xd (cosh x) = cosh2 x − 1 d (cosh x)
= 31 cosh3 x − cosh x + C
R R R R 
2. I (x) = cosh4 xdx = 14 (cosh 2x + 1)2 dx = 1
4
cosh2 2xdx + 2 cosh 2x + x
 
= 41 18 sinh (4x) + x2 + 21 sinh (2x) + c + C
1
= sinh (4x) + 14 sinh (2x) + 43 x + C.
32
R R
3. I (x) = sinh3 x cosh xdx = sinh3 xd (sinh x) = 14 sinh4 x + C
R R R
4. I (x) = sinh2 x cosh2 xdx = (sinh x cosh x)2 dx = 14 sinh2 2xdx
R
Calcoliamo a parte sinh2 2xdx:
Z Z
2 1
sinh 2xdx = sinh2 tdt
t=2x 2
Z
1
= (cosh 2t − 1) dt
4
1 1
= sinh 2t − t + C1
8 4
1 1
=⇒ I (x) = sinh (4x) − x + C
32 8
R R
5. I (x) = sinh xdx
cosh2 x
= sinh 1
x
d (tanh x)
Abbiamo:
s r
1 cosh2 x − sinh2 x 1
= = − 1,
sinh x sinh2 x tanh2 x
quindi:
Z r
1
I (x) = − 1d (tanh x)
tanh2 x
Poniamo y = tanh x, per cui:
Z p
1 − y2
I (y) = dy
y
Z
= y m (1 − y n )p dy

con m = −1, n = 2, p = 1/2. Risulta: m+1n


∈ N, quindi la sostituzione è
2 2
1−y =t
Z
t2
I (t) = dt
t2 − 1
Z  
1 t − 1
= 1+ 2 dt = t + ln +C
t −1 t + 1
3.17 Integrazione delle funzioni iperboliche 237

Ripristinando la variabile y:
p p

I (y) = 1− y2 2
+ ln |y| − ln 1 + 1 − y

Ma y = tanh x  x 
1
I (x) = + ln tanh +C
cosh x 2
R
6. I (x) = dx
sinh2 x cosh2 x
Abbiamo:
1 1
2 = − 1, (3.128)
sinh x tanh2 x
quindi: Z  
1
I (x) = − 1 d (tanh x)
tanh2 x
Poniamo y = tanh x, per cui:
Z
1 − y2
I (y) = dy
y2
Z Z
dy
= − dy
y2
1
=− −y+C
y

Ripristinando la variabile x:

I (x) = − coth x − tanh x + C


R
7. I (x) = tanh3 xdx. È del tipo
Z
In (x) = tanhn xdx

e va calcolato tramite una relazione di ricorrenza.

I0 (x) = x + C
I1 (x) = ln |cosh x| + C

Per ogni n
3.18 Sostituzioni trigonometriche ed iperboliche 238

3.18 Sostituzioni trigonometriche ed iperboliche


Sia
Z  √ 
I (x) = R x, ax2 + bx + c dx,

essendo R una funzione razionale. Per calcolare I (x) è conveniente ridurre il


trinomio ax2 + bx + c alla forma a (x + k)2 + l, e con un opportuno cambio di
variabile si ha:

Z
 p 
sost. trig.
I (y) = R y, α2 − y 2 dy =⇒ y = α sin t (3.129)
Z  p 
sost. iper.
I (y) = R y, α2 + y 2 dy =⇒ y = α sinh t
Z  p 
sost. iper.
I (y) = R y, y 2 − α2 dy =⇒ y = α cosh t

Nel secondo e terzo integrale è possibile eseguire sostituzioni trigonometriche


anzichè iperboliche:

Z
 p 
sost. trig.
2 2
I (y) = R y, α + y dy =⇒ y = α tan t (3.130)
Z  p 
sost. trig.
I (y) = R y, y 2 − α2 dy =⇒ y = α sec t
Capitolo 4

Integrali definiti

4.1 Somme integrali


Si chiede di calcolare i seguenti integrali considerandoli come il limite delle cor-
rispondenti somme integrali.
Zb Zb ZT
1) x2 dx 2) dx 3) (v0 + gt) dt, con v0 , g =const
0 a 0
Z1 Z10 Zx
4) x2 dx 5) 2x dx 6) f (x) = sin tdt
−2 0 0

4.1.1 Soluzioni
1. Eseguiamo una equipartizione Dn di [0, b]:
b
xk = k , k ∈ N = {0, 1, ..., n − 1}
n
L’ampiezza è
b
δn =
n
La somma integrale:
n−1
X
σDn = f (ξk ) (xk+1 − xk )
k=0

Prendiamo ξk = xk :
n−1
 3 X
b
σDn = k2
n k=0
4.1 Somme integrali 240

Ma:
n−1
X n (2n − 1) (n − 1)
k2 = ,
k=0
6

donde:
b3 (2n − 1) (n − 1)
σDn = ,
6 n2
da cui l’integrale:
Zb
b3
x2 dx = lim σDn = lim σDn =
δn →0 n→+∞ 3
0

2. Eseguiamo unequipartizione Dn di [a, b]:

k (b − a)
xk = a + , k ∈ N = {0, 1, 2, ..., n − 1}
n

L’ampiezza di Dn è:
b−a
δn =
n
Assumendo ξk = xk+1
n−1
X
σDn = f (xk+1 ) (xk+1 − xk )
k=0
n
b − aX
= f (xk )
n k=1
= b − a,

quindi:
Zb
dx = lim σδn = lim σn = b − a
δn →0 n→+∞
a

3. Quest’integrale ha una semplice interpretazione fisica, se poniamo:


ZT
y (T ) = (v0 + gt) dt
0

Qui y (T ) è al tempo T la quota di un punto materiale in caduta libera in


un campo gravitazionale (g è l’accelerazione di gravità) con velocità iniziale
4.1 Somme integrali 241

v0 . L’asse y è orientato verso il basso e si trascura la resistenza dell’aria.


L’integrando è la velocità istantanea:

v (t) = v0 + gt

Eseguiamo un’equipartizione Dn di [0, T ]:

T
tk = k , con k ∈ N = {0, 1, 2, ..., n − 1}
n

L’ampiezza è
T
δn = max (tk+1 − tk ) =
k∈N n
La somma integrale è
n−1
X
σDn = v (τk ) (tk+1 − tk )
k=0

Assumendo τk = tk+1 :
n  
TX T
σDn = v0 + g k
n k=1 n
n + 1 gT
= T v0 +
n 2

Quindi l’integrale:

y (T ) = lim σDn
δn →0

= lim σDn
n→+∞
1
= v0 T + gT 2
2

4. Eseguiamo una equipartizione Dn di [−2, 1]:


3
xk = −2 + k, con k = 0, 1, ..., n − 1
n

di ampiezza:
3
δn =
n
4.1 Somme integrali 242

Assumendo ξk = xk+1 :
n
3X 2
σDn = x
n k=1 k
n  2
3X 3
= k−2
n k=1 n
3 2n2 − 3n + 3
=
2 n

da cui:
Z1
x2 dx = lim σDn = 3
n→+∞
−2

5. Eseguiamo una equipartizione Dn di [0, 10]:


10
xk = k, con k = 0, 1, ..., n − 1
n

di ampiezza:
10
δn =
n
Assumendo ξk = xk+1 :
n
10 X k 10
σDn = 2 n
n k=1
10+n
 n
52 n −1 + (1024)1/n
= n
(−1 + 10241/n )

da cui:
Z10
10230
2x dx = lim σDn =
n→+∞ ln (1024)
0

6. Eseguiamo l’equipartizione:
x
tk = k , con k ∈ N = {0, 1, ..., n − 1}
n

L’ampiezza dell’intervallo [tk , tk+1 ] è:


x
δk,n = tk+1 − tk = ,
n
4.2 Teorema fondamentale del calcolo integrale. 243

donde l’ampiezza della partizione:


x
δn = max (δk ) =
k∈N n

Assumendo τk = tk+1 e ponendo g (t) = sin t:


n−1
X x
σDn = g (tk+1 )
k=0
n
n  x
x X
= sin k
n k=1 n

Per il calcolo della sommatoria utilizziamo una nota relazione trigonomet-


rica: n    
X cos y2 − cos n + 21 y
sin (ky) = y
 ,
k=1
2 sin 2

donde:  y
   
x cos 2
− cos n + 21 y
σDn = 
2n sin y2

Passiamo al limite per n → +∞:


 x
  1
 
x cos 2n
− cos 1 + 2n
x
lim σDn = lim x

n→∞ n→∞ 2n sin 2n
x (1 − cos x)
= x

2 lim n sin 2n
n→+∞

Calcoliamo a parte il limite a denominatore:



x sin mx
2 x
lim n sin =−1 lim+ = ,
n→+∞ 2n m=n m→0 m 2

Quindi
f (x) = lim σDn = 1 − cos x
n→∞

4.2 Teorema fondamentale del calcolo integrale.


1. Calcolare , ,
dI dI
da db
essendo:

Zb
dx
I=
ln (sin x)
a
4.2 Teorema fondamentale del calcolo integrale. 244

sia F (x) una qualunque primitiva di [ln (sin x)]−1 :


Z
dx
F (x) = ,
ln (sin x)

per cui:
I = F (b) − F (a)

Quindi le derivate:
dI 1
= −F ′ (a) = −
da ln (sin a)
dI 1
= F ′ (b) =
db ln (sin b)

2. Calcolare la derivata della funzione:


Zx
F (x) = ln tdt
1

Poniamo: Z
G (t) = ln tdt

Quindi:
F (x) = G (x) − G (1) =⇒ F ′ (x) = G′ (x) = ln x

3. Calcolare la derivata della funzione:


Zx2
2
F (x) = e−t dt
x

Poniamo: Z
2
G (t) = e−t dt

Quindi: 
F (x) = G x2 − G (x)

La derivata è:
d  d
F ′ (x) = G x2 − G (x)
dx  dx
= 2xG′ x2 − G′ (x)
4 2
= 2xe−x − e−x
4.2 Teorema fondamentale del calcolo integrale. 245

4. Calcolare la derivata della funzione:


Z0 √
F (x) = 1 + t4 dt
x

Poniamo: Z √
G (t) = 1 + t4 dt

Quindi: √
F (x) = G (0) − G (x) =⇒ F ′ (x) = − 1 + x4

5. Calcolare la derivata della funzione:



Zx

F (x) = cos t2 dt
1/x

Poniamo: Z

G (t) = cos t2 dt

Quindi:  
√  1
F (x) = G x − G
x
La derivata è:
 
′ d √  d 1
F (x) = G x − G
dx dx x
 
1 1 1
= √ cos x + 2 cos
2 x x x2

6. Determinare i punti estremali della funzione seno integrale:


Zx
sin t
F (x) = dt
t
0

Poniamo: Z
sin t
G (t) = dt,
t
donde:
F (x) = G (x) − G (0)
4.2 Teorema fondamentale del calcolo integrale. 246

Segue:
sin x
F ′ (x) = G′ (x) =
x
I punti estremali di F (x) sono le radici dell’equazione:
sin x
F ′ (x) = 0 ⇐⇒ =0
x

Cioè:
xk = kπ, ∀k ∈ Z − {0}

In figura 4.1 è riportato il grafico di F (x).

***

Calcolare:
Z1 Z−1
1) 1+xdx
2) dx
x3
0 −2
Zx Zx
3) e dt 4)
t
cos tdt
−x 0
Soluzioni
R1
1. I = 0 dx
1+x
. Poniamo:
Z
dx
F (x) = = ln |1 + x| + C,
1+x

donde:
I = F (1) − F (0) = ln 2
R −1 −1
2. I = dx
−2 x3
= − 2x12 −2 = − 38 .
Rx x
3. I = et dt = et |−x = ex − e−x
−x
Rx
4. I (x) = 0 cos tdt = sin t|x0 = sin x

***

Calcolare i seguenti limiti attraverso gli integrali definiti.



1. lim n12 + n22 + ... + n−1n2
n→+∞
4.2 Teorema fondamentale del calcolo integrale. 247

3
€€€€
2

1
€€€€
2

x
-10 Π -8 Π -6 Π -4 Π -2 Π 2Π 4Π 6Π 8Π 10 Π

1
- €€€€
2

-1

3
- €€€€
2

Figura 4.1: Grafico della funzione seno integrale e degli asintoti orizzontali: y =
±π/2.
4.2 Teorema fondamentale del calcolo integrale. 248


2. lim 1
n+1
+ 1
n+2
+ ... + 1
n+n
n→+∞

1p +2p +...+np
3. λp = lim np+1
(p > 0)
n→+∞

Soluzioni

1. È il limite di una somma integrale relativa a f (x) = x in [0, 1]. Infatti,


eseguiamo una equipartizione Dn di [0, 1]:
k
xk = , con k ∈ N = {0, 1, 2, ..., n − 1}
n
La norma è:
1
δn = max (xk − xk−1 ) =
k∈N n
Assumendo ξk = xk :
n−1
X
σDn = f (ξk ) (xk − xk−1 )
k=0
n−1
1 X
= k
n2 k=0
1 2 n−1
= 2
+ 2 + ... + ,
n n n2
donde:
  Z1
1 2 n−1 1
lim + + ... + = lim σDn = xdx =
n→+∞ n2 n2 n2 n→+∞ 2
0

2. Poniamo:
1 1 1
σDn = + + ... +
n+1 n+2 n+n
n
X 1
=
k=1
n+k

essendo Dn una equipartizione di [0, 1]. Deve essere:


n
X
σDn = f (xk ) (xk − xk−1 )
k=1
n  
1 X k
= f
n k=1 n
4.2 Teorema fondamentale del calcolo integrale. 249

Confrontando con la precedente:


 
k n
f =
n n+k
1
= ,
1 + nk

cioè:
1
f (x) = ,
1+x
per cui σDn è una somma integrale relativa a f (x) = x. Da ciò segue:
  Z1
1 1 1 dx
lim + + ... + = = ln 2
n→+∞ n+1 n+2 n+n 1+x
0

3. Poniamo:
1p + 2p + ... + np
σDn =
np+1
n
1 X p
= p+1 k
n k=1

essendo Dn una equipartizione di [0, 1]. Deve essere:


n
X
σDn = f (xk ) (xk − xk−1 )
k=1
n  
1 X k
= f
n k=1 n

Confrontando con la precedente:


   p
k k
f = ,
n n

cioè:
f (x) = xp

Quindi:
Z1
1
λp = xp dx =
p+1
0
4.2 Teorema fondamentale del calcolo integrale. 250

***

Calcolare:

Z2 Z8 Z4 Z6
√ √  √
1+ y √
1) (x2 − 2x + 3) dx 2) 2x + 3 x dx 3) y2
dy 4) x − 2dx
1 0 1 2
Z−3 Z−3 Z1 Z1
y 5 dy
5) √ dx
25+3x
6) dx
x2 −1
7) xdx
x2 +3x+2
8) y+2
0 −2 0 −1√
Z1 Z4 Z1 Z2/2
s3 ds
9) dx
x2 +4x+5
10) dx
x2 −3x+2
11) s8 +1
12) √ dx
1−x2
0 5 0 0
Z7/2 Z1 Zπ/2 Ze2
2 dy
13) √ dx
14) √y 15) sin3 xdx 16) dx
x ln x
−x2 +4x+5 y 6 +4
2 0 0 e

Soluzioni
R2  x=2
1. I = 1 (x2 − 2x + 3) dx = 31 x3 − x2 + 3x x=1 = 73
R8 √ √  √ R8 R8
2. I = 0 2x + 3 x dx = 2 0 x1/2 dx + 0 x1/3 dx

= 2 · 32 · 83/2 + 43 · 84/3 = 100
3

R 4 1+√y R 4 −2 R 4 −3/2 h i4  4
3. I = 1 y2 dy = 1 y dy + 1 y dy = − y1 + −2y −1/2 1
1
1 1
 7
= − 4 + 1 + −2 · 2 + 2 · 1 = 4
R6√
4. I = 2 x − 2 = F (6) − F (2), essendo
Z

F (x) = x − 2dx
Z
= (x − 2)1/2 d (x − 2)
2
= (x − 2)3/2 + C,
3

donde:
16
I=
3
4.2 Teorema fondamentale del calcolo integrale. 251

R −3
5. I = 0
√ dx
25+3x
= F (−3) − F (0), essendo:
Z
dx
F (x) = √
25 + 3x
Z
1
= (25 + 3x)−1/2 d (25 + 3x)
3
2√
= 25 + 3x + C,
3
donde:
2√ 2√ 2
I= 16 − 25 = −
3 3 3
R −3
6. I = dx
−2 x2 −1
= F (−3) − F (−2), essendo:
Z
dx 1 x − 1
F (x) = = ln + C,
x2 − 1 2 x + 1

donde:
1
F (−3) = ln 2 + C
2
1
F (−2) = ln 3 + C
2
Quindi:
1 2
I= ln
2 3
R1
7. I = xdx
0 x2 +3x+2
= F (1) − F (0), essendo:

Z1
xdx
F (x) =
x2 + 3x + 2
0

1 2 3 x + 1
= ln x + 3x + 2 − ln +C
2 2 x + 2
= − ln |x + 1| + 2 ln |x + 2| + C,

donde:
F (1) = − ln 2 + 2 ln 3 + C
F (0) = 2 ln 2 + C

Quindi:
9
I = ln
8
4.2 Teorema fondamentale del calcolo integrale. 252

R1 y 5 dy
8. I = −1 y+2
= F (1) − F (−1), essendo:
Z 5
y dy
F (y) =
y+2
Z  
4 3 2 32
= y − 2y + 4y − 8y + 16 − dy
y+2
1 1 1
= y 5 − y 4 + y 3 − 4y 2 + 16y − 32 ln |y + 2| + C,
5 2 5
donde:
526
I= − 32 ln 3
15
R1
9. I = dx
0 x2 +4x+5
= F (1) − F (0), essendo:
Z
dx
F (x) = 2
x + 4x + 5

Per la (3.45):
F (x) = arctan (x + 2) + C,

donde:
I = arctan 3 − arctan 2,

che può eesere semplificata:


tan I = tan (α − β) , con α = arctan 3, β = arctan 2

Come è noto:
tan α − tan β
tan (α − β) = ,
1 + tan α tan β
da cui:
1 1
tan I = =⇒ I = arctan
7 7
R1
10. I = dx
0 x2 −3x+2
= F (4) − F (5), essendo:
Z
dx
F (x) =
x2 − 3x + 2

Per la (3.45):
x − 2
F (x) = ln + C,
x − 1
donde:
2 3 8
I = ln − ln = ln
3 4 9
4.2 Teorema fondamentale del calcolo integrale. 253

R1 s3 ds
11. I = 0 s8 +1
= F (1) − F (0), essendo:
Z
s3 ds
F (s) =
s8 + 1

Eseguendo il cambio di variabile s → y = s4 :


Z
1 dy 1
F (y) = 2
= arctan y
4 y +1 4

Ripristinando la variabile s:
1 
F (s) = arctan s4 ,
4

donde:
π
I=
16

R √2 2
12. I = 0
2 √ dx
1−x2
= [arcsin x]0 =
2 π
4
R 7/2 
13. I = 2
√ dx
−x2 +4x+5
=F 7
2
− F (2), essendo:
Z
dx
F (x) = √
−x2 + 4x + 5

Dalla (3.46):  
x−2
F (x) = arcsin + C,
3
donde:  
1 π
I = arcsin =
2 6
R1 2 dy
14. I = 0
√y = F (1) − F (0), essendo:
y 6 +4

Z
y 2 dy
F (y) = p
y6 + 4

Eseguendo il cambio di variabile y → x = y 3 :


Z √
1 dx 1

F (x) = √ 2
= ln x + x + 4 + C
3 2
x +4 3
4.2 Teorema fondamentale del calcolo integrale. 254

Cioè:
1 3 p 6

F (y) = ln y + y + 4 + C
3
donde: √ !
1 1+ 5
I = ln
3 2
R π/3 R π/2
15. I (x) = 0
sin3 xdx = 0
sin2 xd (− cos x). Integriamo per parti:
Z π/2 Z π/2
2
π/2 2
sin xd (− cos x) = − cos x sin x 0 + 2 sin x cos2 xdx
0 0
h π  i
=2 F − F (0) ,
2

essendo:
Z
F (x) = sin x cos2 xdx
Z
= cos2 xd (− cos x)
1
= − cos3 x + C,
3

donde:
2
I=
3
R e2
16. I = e
dx
x ln x
= F (e2 ) − F (e), essendo:
Z
dx
F (x) =
x ln x
Z
d (ln x)
=
ln x
= ln (ln x) + C,

donde:

F e2 = ln 2 + C
F (e) = C

Quindi:
I = ln 2

Calcolare:
4.2 Teorema fondamentale del calcolo integrale. 255

Zπ/4 Zπ/3 Z1 Z6
ex

1) tan xdx 2) cot4 xdx 3) 1+e2x
dx 4) x − 2dx
−π/4 π/6 0 2
Z−3 Z−3 Z1 Z1
y 5 dy
5) √ dx
25+3x
6) dx
x2 −1
7) xdx
x2 +3x+2
8) y+2
0 −2 0 −1√
Z1 Z4 Z1 Z2/2
s3 ds
9) dx
x2 +4x+5
10) dx
x2 −3x+2
11) s8 +1
12) √ dx
1−x2
0 5 0 0
Z7/2 Z1 Zπ/2 Ze2
2 dy
13) √ dx
14) √y 15) sin3 xdx 16) dx
x ln x
−x2 +4x+5 y 6 +4
2 0 0 e

Soluzioni

1. I = −4π tan x. Risulta I = 0. Infatti se f (x) è una funzione dispari e per
4
ogni a ∈ (0, +∞):
Za
Ia = f (x) dx
−a
Z0 Za
= f (x) dx + f (x) dx
−a 0

Nel secondo integrale eseguiamo il cambio di variabile x → x′ = −x, per


cui:
0 ≤ x = −x′ ≤ a =⇒ 0 ≥ x′ ≥ −a

cioè:
Z0 Z−a
Ia = f (x) dx − f (x′ ) dx′
−a 0
Z0 Z0
= f (x) dx + f (x′ ) dx′ = 0
−a −a

R π3  π
2. I = π cot4 xdx = − 13 cot3 x + cot x + x π3 = 8

9 3
+ π6 .
6 6

R1 ex
3. I = 0 1+e2x
dx = F (1) − F (0), essendo:
Z
ex
F (x) = dx
1 + e2x
4.2 Teorema fondamentale del calcolo integrale. 256

Eseguendo il cambio di variabile x → y = ex :


Z
dy
F (y) = = arctan y + C
1 + y2

Cioè:
F (x) = arctan (ex ) + C,

donde:
π
I = arctan e −
4
Capitolo 5

Estensione del concetto di integrale

5.1 Introduzione
Nel capitolo 1 abbiamo introdotto la nozione di integrale definito di una funzione
continua in un intervallo chiuso e limitato [a, b]. Ci si può chiedere se tale nozione
possa essere estesa al caso di una funzione che abbia punti di discontinuità e/o
che sia definita in un intervallo illimitato. Sotto opportune ipotesi, la risposta
è affermativa. Per rendere operativa tale estensione della nozione di integrale,
ricordiamo la seguente

Definizione. Una funzione reale di variabile reale f (x) definita in un inter-


vallo A ⊆ R, si dice generalmente continua, se l’insieme dei suoi punti di
discontinuità è al più numerabile.

In questa sezione ci occuperemo del caso in cui f (x) è generalmente continua


e non negativa nell’intervallo A ⊆ R. È facile rendersi conto che è comunque
possibile costruire una successione finita di intervalli:

[a1 , b1 ] , [a2 , b2 ] , ..., [an , bn ]


tali che:

[ak , bk ] ⊂ A, con k = 1, 2, ..., n


[ak , bk ] ∩ [ak′ , bk′ ] = ∅, ∀k 6= k ′
∀k, f (x) è continua in [ak , bk ]

Si osservi che tale proprietà continua a valere anche se A è illimitato. Ad esempio


se A = [a, +∞), fissiamo A′ = [a, b] ⊂ A e ripetiamo il procedimento per
l’intervallo A′ .
Poniamo per definizione:
5.1 Introduzione 258

n
[
def
D = [ak , bk ] (5.1)
k=1

Chiamiamo l’insieme (5.1) dominio limitato e misurabile di continuità per


f (x).
Osservazione. Assegnato l’intervallo A ⊆ R, esistono infiniti domini (5.1).
Esempio 20. Consideriamo la seguente funzione non negativa in A = [0, 3]:
1
f (x) = + (x − 2)2 se x ∈ [0, 1) (5.2)
x
f (x) = −x2 + 4x se x ∈ [1, 3]

L’insieme delle discontinuità di f (x) è:

S = {ξ0 = 0, ξ1 = 1} ,

donde f (x) è generalmente continua. Costruiamo gli intervalli:

[a1 , b1 ] , [a2 , b2 ]

essendo a1 = 0.2, b1 = 0.8, a2 = 1.2, b2 = 3 (figura 5.1).


Si conclude che un dominio limitato di continuità per f (x) è D = [a1 , b1 ]∪[a2 , b2 ].

***

Costruiamo quindi la famiglia:

F = {D | D è un dominio limitato e mis di continuità per f (x)}


Ciò posto, indichiamo con il simbolo:
Z
f (x) dx
X

l’integrale di f (x) esteso ad un intervallo chiuso e limitato X in cui f (x) è


continua. Evidentemente:
bk
Z n Z
X
∀D ∈ F, f (x) dx = f (x) dx ≥ 0,
D k=1a
k

poiché è f (x) ≥ 0 per ipotesi. Inoltre:


Z
∀D ∈ F, f (x) dx = λ ∈ Λ ⊆ [0, +∞) ,
D
5.1 Introduzione 259

x
a1 b1 Ξ1 a2 b2

Figura 5.1: Grafico della funzione (5.2)


5.2 Rettangoloide generalizzato 260

Definizione. Si chiama integrale della funzione generalmente continua


f (x) ≥ 0 esteso all’intervallo A, l’estremo superiore dell’insieme Λ:
Z
f (x) dx = sup Λ ≤ +∞ (5.3)
A

5.2 Rettangoloide generalizzato


La definizione (5.3) non si presta ad un calcolo diretto dell’integrale di una fun-
zione generalmente continua esteso ad un intervallo limitato o illimitato. Ciò può
essere realizzato attraverso un’operazione di passaggio al limite. Iniziamo con
l’osservare che:

D ∈ F =⇒ ∃D′ ∈ F | D′ ⊇ D,
per cui possiamo costruire una successione di domini:

{Dn }n∈N | D1 ⊆ D2 ⊆ ...Dn ⊆ ... (5.4)


Inoltre:

∀D ∈ F, D ⊆ A − S
essendo S l’insieme (numerabile) dei punti di discontinuità di f (x).

Definizione. Al crescere indefinito di n, la successione (5.4) tende all’insieme


A − S se risulta:
∀D ⊆ A − S, ∃ν ∈ N | Dν ⊇ D
Quindi:
∀n ∈ N, ∃ν ∈ N | Dν ⊇ Dn
Esprimiamo ciò scrivendo:
lim Dn = A − S
n→+∞

Teorema 21. Nelle ipotesi poste:


Z Z
lim f (x) dx = sup Λ = f (x) dx
n→+∞
Dn A

Dimostrazione. Omessa.
Questo teorema ci dice che nelle ipotesi poste:
Z Z
f (x) dx = lim f (x) dx (5.5)
n→+∞
A Dn
5.2 Rettangoloide generalizzato 261

Diamo ora un’interpretazione geometrica alla (5.5). A tale scopo consideriamo il


sottoinsieme di R2 :
def 
U = (x, y) ∈ R2 | x ∈ A − S, 0 ≤ y ≤ f (x)
Tale insieme di punti si chiama rettangoloide generalizzato di base A − S,
relativo a f (x).
Si osservi che U è illimitato in uno dei seguenti casi: 1) A è limitato e f (x) non
limitata (cioè dotata di singolarità); 2) A illimitato e f (x) limitata (dotata al più
di punti di discontinuità eliminabili o di prima specie); 3) A e f (x) non limitati.
Ciò premesso, sussiste il
Teorema 22. Nelle ipotesi poste:
Z
misU = f (x) dx
A

Dimostrazione. Omessa.
Osserviamo che la misura di U può essere finita anche se U è illimitato, come nel
caso della funzione:
1
f (x) = √ con x ∈ A = [−1, 1]
1 − x2
Tale funzione è generalmente continua in A:

S = {−1, 1} =⇒ A − S = (−1, 1)
Più precisamente:

lim f (x) = +∞
x→1−
lim f (x) = +∞
x→−1+

Il rettangoloide generalizzato è:
 
2 1
U = (x, y) ∈ R | −1 < x < 1, 0 ≤ x ≤ √
1 − x2
Costruiamo la successione {Dn }n∈N :
 
1 1
Dn = −1 + , 1 −
n n

come mostrato in figura 5.2.


5.2 Rettangoloide generalizzato 262

x
-1+1/n 1-1/n

Figura 5.2: Grafico della funzione f (x) = √ 1


1−x2
.

Quindi:

1
1− n
Z1 Z
dx dx
√ = lim √
1 − x2 n→+∞ 1 − x2
−1 1
−1+ n
    
1 1
= lim arcsin 1 − − arcsin −1 +
n→+∞ n n
= arcsin (1) − arcsin (−1) = π,

donde:
misU = π
Un esempio di insieme illimitato di misura infinita è dato dal rettangoloide
generalizzato relativo alla seguente funzione:
1
f (x) = con x ∈ A = [0, 1]
ex −1
f (x) è non negativa, inoltre:

lim f (x) = +∞,


x→0+
5.2 Rettangoloide generalizzato 263

donde:
A − S = (0, 1]
Costruiamo la successione di domini:
 
1
Dn = ,1 ,
n
come mostrato in figura 5.3.

14

12

10

8
y

0
0 1/n 0.2 0.4 0.6 0.8 1
x

Figura 5.3: Grafico della funzione f (x) = 1


ex −1
.

Evidentemente:

lim Dn = (0, 1]
n→+∞

Abbiamo:

Z1 Z1
dx dx
= lim
ex − 1 n→+∞ ex − 1
0 1/n
 1
= lim ln 1 − e−x 1/n
n→+∞

= ln 1 − e−1 − ln 0+
= +∞,
5.2 Rettangoloide generalizzato 264

donde:

misU = +∞
Riportiamo di seguito un esempio di integrale di una funzione continua esteso ad
un intervallo illimitato.
1
f (x) = per x ∈ A = (−∞, +∞) (5.6)
1 + x2
La funzione (5.6) è manifestamente continua ed infinitesima all’infinito:
1
lim 2
= 0+
|x|→+∞ 1 + x

Costruiamo la successione (5.4) assumendo:

Dn = [−n, n]
come mostrato in fig. 5.2.

0.8

0.6
y

0.4

0.2

0
-3 -2 -1 0 1 2 3
x

Figura 5.4: Grafico della funzione f (x) = 1


1+x2
.

donde:
5.2 Rettangoloide generalizzato 265

Z+∞ Zn
dx dx
2
= lim
1+x n→+∞ 1 + x2
−∞ −n

= lim [arctan (n) − arctan (−n)]


n→+∞
π  π
= − − =π
2 2
Quindi anche in questo caso abbiamo un insieme illimitato di misura finita:


U = (x, y) ∈ R2 : −∞ < x < +∞, 0 ≤ y ≤ f (x)
misU = π

***

Calcoliamo l’integrale:
xZ0 +h

I= f (x) dx,
x0 −h

essendo:
1
f (x) = (5.7)
|x − x0 |α
Qui è A = [x0 − h, x0 + h], essendo per ipotesi x0 ∈ R, e h, α > 0. L’integrando
ha una singolarità in x0 :
1
lim α = +∞,
x→x0 |x − x0 |

donde A − S = A − {x0 }.
Costruiamo la successione (5.4) assumendo:
   
1 1
Dn = x0 − h, x0 − ∪ x0 + , x0 + h
n n
come mostrato in fig. 5.5.
Quindi:
 1

x0 − n xZ0 +h
Z
dx dx 
(5.8)

I = lim  +
|x − x0 |α |x − x0 |α

n→+∞
x0 −h 1
x0 + n
5.2 Rettangoloide generalizzato 266

140

120

100

80

60

40

20

0 x -h x0 x0+h
0

Figura 5.5: Grafico della funzione f (x) = 1


|x−x0 |α
.

Poniamo:

1
x0 − n
Z
dx
I1 =
|x − x0 |α
x0 −h
xZ0 +h
dx
I2 =
|x − x0 |α
1
x0 + n

Eseguendo il cambio di variabile x → ξ = x − x0 :

1
Z− n

I1 = −
ξα
−h
Zh

I2 =
ξα
1
n

In virtù della simmetria di Γ)y = f (ξ) rispetto all’asse delle ordinate: I1 = I2 ,


per cui:
5.2 Rettangoloide generalizzato 267

Zh

I = 2 lim
n→+∞ ξα
1
n

Se α 6= 1:

 
2 1 2h1−α
I= − 0 = , se α ∈ (0, 1)
1 − α hα−1 1−α
 
2 1
I=− − (+∞) = +∞, se α ∈ (1, +∞)
α − 1 hα−1

Se α = 1

I = 2 lim ln (nh) = +∞
n→+∞

Cioè:
 2h1−α
,
se α ∈ (0, 1)
I= 1−α
+∞, se α ∈ [1, +∞)
In fig. 5.6 sono riportati i grafici della funzione (5.7) per x0 = 5, h = 2, nei due
casi: a = 0.4, 2.

2.5

1.5
y

0.5

x0
0
3 3.5 4 4.5 5 5.5 6 6.5 7
x

Figura 5.6: Grafico della funzione (5.7).


5.2 Rettangoloide generalizzato 268

Per quanto visto l’integrale (5.8) converge solo per α ∈ (0, 1):

lim I (α) = +∞
α→1−
In fig. 5.7 è riportato l’andamento di I in funzione del parametro α.

200

180

160

140

120
I(α)

100

80

60

40

20

0
0 0.2 0.4 0.6 0.8 1
α

Figura 5.7: Andamento dell’integrale I in funzione di α.

***
Calcoliamo gli integrali:

Z+∞
I= f (x) dx (5.9)
x0 +h
xZ0 −h

J= g (x) dx,
−∞

essendo

1
f (x) = (5.10)
(x − x0 )α
1
g (x) =
(x0 − x)α
5.2 Rettangoloide generalizzato 269

Nella (5.10) x0 è un punto arbitrario dell’asse x, mentre α, h parametri positivi.


Il grafico è riportato in fig. (5.8).

0.25

0.2

0.15
y

0.1

0.05

0
x0+h x x0+n

Figura 5.8: Grafico della funzione (5.10).

Costruiamo i domini Dn :

Dn = [x0 + h, x0 + n]
Quindi:

I = lim In ,
n→+∞

essendo:
xZ0 +n
dx
In =
(x − x0 )α
x0 +h

Risulta:

n1−α − h1−α
In = , se α 6= 1
1−α
n
In = ln , se α = 1
h
Quindi:
5.2 Rettangoloide generalizzato 270


n1−α − h1−α +∞, se 0 < α < 1
I (α 6= 1) = lim = h1−α
n→+∞ 1−α α−1
, se α > 1
n
I (α = 1) = lim ln = +∞
n→+∞ h
Si conclude che l’integrale converge solo per α > 1.

100

80

60
y

40

20

0 x0+h x +h
x0

L’integrale J (secondo degli integrali (5.9)) assume lo stesso valore di I, in virtù


della simmetria dell’integrando. Infatti possiamo scrivere:

Z+∞
I= f (x) dx
x0 +h
xZ0 −h

J= f (x) dx,
−∞

avendo ridefinito la funzione integranda:


1
f (x) =
|x − x0 |α
Tale funzione è manifestamente simmetrica rispetto alla retta x − x0 = 0, per cui:
5.2 Rettangoloide generalizzato 271

Z+∞

I=
ξα
h
Z−h

J =−
ξα
−∞

***

Calcoliamo l’integrale:
Z+∞
I= e−|x| dx
−∞

Procediamo per decomposizione:

Z0 Z+∞
−|x|
I= e dx + e−|x| dx
−∞ 0
def
= I1 + I2 ,

essendo:

Z0 Z0
−|x|
I1 = e dx = ex dx
−∞ −∞
Z+∞ Z+∞
−|x|
I2 = e dx = e−x dx
0 0

Eseguiamo in I1 il cambio di variabile: x → x′ = −x. Osserviamo che:

−∞ ≤ x = −x′ ≤ 0,
per cui:

+∞ ≥ x′ ≥ 0
Quindi:
5.2 Rettangoloide generalizzato 272

Z0

I1 = − e−x dx′
+∞
Z+∞

= ex dx′ = I2 ,
0

donde:
Z+∞
I = 2 e−x dx
0

Assumendo:

Dn = [0, n] ,
si ha:

Zn
I = 2 lim e−x dx
n→+∞
0

= 2 lim 1 − e−n
n→+∞

=2

che è la misura del rettangoloide generalizzato:



U = (x, y) ∈ R2 | −∞ < x < +∞, 0 ≤ y ≤ e−|x| ,
come riportato in figura (5.9).

***

Passiamo ora al caso in cui la funzione f (x) è generalmente continua e di segno


qualunque in A ⊆ R.
Definiamo:

|f (x)| + f (x) |f (x)| − f (x)


f1 (x) = , f2 (x) = (5.11)
2 2
Quindi:

f (x) = f1 (x) − f2 (x) (5.12)


5.2 Rettangoloide generalizzato 273

0.8

0.6
y

0.4

0.2

0
-4 -3 -2 -1 0 1 2 3 4
x

Figura 5.9: Grafico di f (x) = e−|x| .

Evidentemente:

f1 (x) = f (x) , per x ∈ A | f (x) > 0


f1 (x) = 0, altrimenti

Mentre:

f2 (x) = −f (x) , per x ∈ A | f (x) < 0


f2 (x) = 0, altrimenti

Sempre dalla (5.11) segue

∀x ∈ A, f1 (x) ≥ 0, f2 (x) ≥ 0
Cioè le (5.11) sono non negative in A, e al pari di f (x) sono generalmente con-
tinue in A. Più precisamente, se S è l’insieme dei punti di discontinuità di
f (x), indicando con S1 , S2 l’insieme dei punti di discontinuità di f1 (x) , f2 (x)
rispettivamente, segue:

Sk ⊆ S con k = 1, 2 (5.13)
Verifichiamo la (5.13) attraverso degli esempi. Sia ξ ∈ S tale che:
5.2 Rettangoloide generalizzato 274

x
Ξ

lim f (x) = l1 ∈ (−∞, 0)


x→ξ −

lim f (x) = 0
x→ξ +

Un possibile andamento è illustrato in fig. (5.2).


Abbiamo:

lim f (x) = l1 ∈ (−∞, 0) =⇒ (∃δ > 0 | x ∈ (ξ − δ, ξ) ∩ A =⇒ f (x) < 0 =⇒ f1 (x) = 0)


x→ξ −

Cioè:

lim f1 (x) = 0
x→ξ −

lim f1 (x) = 0,
x→ξ +

da cui la continuità di f1 (x) in ξ. Quindi ξ ∈ S, ξ ∈


/ S1 .
Per la f2 (x) consideriamo quest’altro esempio:

lim f (x) = l2 ∈ (0, +∞)


x→ξ +

lim f (x) = 0
x→ξ −

Abbiamo:
5.2 Rettangoloide generalizzato 275

lim f (x) = l2 ∈ (0, +∞) =⇒ (∃δ > 0 | x ∈ (ξ, ξ + δ) ∩ A =⇒ f (x) > 0 =⇒ f2 (x) = 0)
x→ξ +

Cioè:

lim f2 (x) = 0
x→ξ −

lim f2 (x) = 0,
x→ξ +

da cui la continuità di f2 (x) in ξ. Quindi ξ ∈ S, ξ ∈


/ S2 . L’andamento è illustrato
in fig. (5.2).

Ricapitolando: assegnata la funzione f (x) generalmente continua in A e di segno


qualunque, restano definite le funzioni (5.11) non negative e generalmente contin-
ue in A. La funzione f (x) si esprime poi attraverso la (5.12). Per quanto detto,
le fk (x) sono non negative, quindi hanno senso gli integrali:
Z Z
f1 (x) dx ≤ +∞, f2 (x) dx ≤ +∞ (5.14)
A A

Si noti che il rettangoloide generalizzato relativo ad una f (x) che cambia segno
in A, sarà costituito da punti di ordinata y < 0; ad esempio consideriamo la
funzione il cui grafico è riportato in figura (5.10).
Il rettangoloide generalizzato è riportato in fig. (5.2).
Passando alle funzioni (5.11) possiamo definire i seguenti rettangoloidi generaliz-
zati:
5.2 Rettangoloide generalizzato 276

x
Ξ1 Ξ2

Figura 5.10: Grafico di una funzione generalmente continua e di segno qualunque.

x
Ξ1 Ξ2
5.2 Rettangoloide generalizzato 277

U1 = {(x, y) ∈ R | x ∈ A − S, 0 ≤ y ≤ f1 (x)}
U2 = {(x, y) ∈ R | x ∈ A − S, − f2 (x) ≤ y ≤ 0} ,

riportati in figg. (5.11)-(5.12).

x
Ξ1 Ξ2

Figura 5.11: Rettangoloide U1

y
x
Ξ1 Ξ2

Figura 5.12: Rettangoloide U2

Evidentemente:
Z
misU1 = f1 (x) dx,
A

mentre:
Z Z
misU2 = − [−f2 (x)] dx = f2 (x) dx
A A

In forza delle (5.14):


5.2 Rettangoloide generalizzato 278

misU1 ≤ +∞, misU2 ≤ +∞


D’altro canto la (5.12) suggerisce di porre:
Z Z Z
f (x) dx = f1 (x) dx − f2 (x) dx (5.15)
A A A

La (5.15) non è accettabile se i due integrali a secondo membro sono entrambi


infiniti; da un punto di vista geometrico significa che i due rettangoloidi U1 e
U2 hanno entrambi misura infinita. In tal caso la (5.15) si presenta nella forma
indeterminata ∞ − ∞. Ciò implica la seguente

Definizione. La funzione f (x) è integrabile in A se almeno uno dei due inte-


grali (5.14) è < +∞.

In particolare:


Z Z Z
f1 (x) dx = +∞, f2 (x) dx < +∞ =⇒ f (x) dx = +∞
A A A

Z Z Z
f1 (x) dx < +∞, f2 (x) dx = +∞ =⇒ f (x) dx = −∞
A A A

Z Z Z
f1 (x) dx < +∞, f2 (x) dx < +∞ =⇒ f (x) dx < +∞
A A A

Da un punto di vista geometrico:


Z
f (x) dx = misU1 − misU2 ,
A
R
cioè RA f (x) dx ha senso solo se uno dei due rettangoloidi ha misura finita. In tal
caso A f (x) dx è la differenza tra le due aree, e può essere finita o pari a +∞ o
−∞.
Per quanto detto:
Z
misU1 = +∞, misU2 = +∞) =⇒ f (x) dx non ha senso
A

Si osservi che potremmo comunque scegliere una particolare successione {Dn }


tale da rimuovere la forma indeterminata ∞ − ∞, ma in tal caso il valore assunto
dall’integrale viene a dipendere dalla scelta della successione, per cui questa non
5.2 Rettangoloide generalizzato 279

è una definizione operativa. Inoltre molte proprietà classiche non sono più ver-
ificate. Tuttavia la rimozione della forma indeterminata è spesso utilizzata nelle
applicazioni. Si parla in tal caso di “integrali impropri”.
Assumiamo per ora la definizione precedente:

Z Z Z
f (x) dx = f1 (x) dx − f2 (x) dx ⇐⇒ (5.16)
A A A
Z
⇐⇒ ∃k ∈ {1, 2} | fk (x) dx < +∞
A

Si osservi che la (5.16) contiene come caso particolare la definizione di integrale


per una funzione generalmente continua e non negativa. Infatti:

Z Z
∀x ∈ A, f (x) ≥ 0 =⇒ f1 (x) ≡ f (x) , f2 (x) ≡ 0 =⇒ f (x) dx = f1 (x) dx
A A
(5.17)
Viceversa:

Z Z
∀x ∈ A, f (x) ≤ 0 =⇒ f1 (x) ≡ 0, f2 (x) ≡ −f (x) =⇒ f (x) dx = − f2 (x) dx
A A
(5.18)

Teorema 23. Sia f (x) una funzione generalmente continua in A ed ivi di segno
qualunque. Se f (x) è integrabile in A, indicato con S l’insieme dei punti di
discontinuità, risulta:
Z Z
∀ {Dn }n∈N | lim Dn = A − S, lim f (x) dx = f (x) dx
n→+∞ n→+∞
Dn A

Dimostrazione. Omessa
Si osservi che tale teorema è applicabile solo se la funzione è integrabile. In altri
termini, l’integrabilità della funzione va stabilita a priori.

Esempio 24. Studiamo l’integrabilità di f (x) = log x in A = [0, 1].


x0 = 0 è una singolarità poichè limx→0+ log x = −∞, quindi A − S = (0, 1]. D’al-
tro canto è f (x) ≤ 0 in A, per cui la funzione è ivi integrabile. Più precisamente
(per la (5.18)) Z Z
f (x) dx = − f2 (x) dx
A A
5.2 Rettangoloide generalizzato 280

Risulta: Z Z
f (x) dx = lim f (x) dx
n→+∞
A Dn

Assumiamo come successione di domini limitati e misurabili di continuità per


f (x):  
1
Dn = ,1 (5.19)
n
Ciò è riportato in fig. (24).

y
x
1
1
n
-1

-2

-3

-4

Al crescere di n, il dominio (5.19) si “avvicina” all’intervallo A, come mostrato in


fig. (24).

y
x
1
1
n
-1

-2

-3

-4
5.2 Rettangoloide generalizzato 281

Quindi:
Z1 Z1
log xdx = lim log xdx
n→+∞
0 1/n

Risulta:
Z1
log xdx =
1/n
Z1
= [x log x]x=1
x= 1 − dx
n
1/n
 
1 1 1
= − ln −1+
n n n

Quindi:
Z1
1 + ln n
log xdx = −1 + lim
n→+∞ n
0

Ma:
1 + ln n 1 ln n
lim = lim + lim = 0,
n→+∞ n n→+∞ n
| {z } | {z n }
n→+∞

=0 =0

donde:
Z1
log xdx = −1
0

Esempio 25. Studiamo l’integrabilità f (x) = e−x sin x nell’intervallo A = [0, +∞).
Qui abbiamo una funzione continua in un intervallo illimitato. Osserviamo che:

|f (x)| = e−x |sin x| ≤ e−x =⇒ −e−x ≤ f (x) ≤ ex

Cioè f (x) è un’oscillazione sinuisoidale il cui inviluppo di modulazione è y =


±e−x . Inoltre:
lim f (x) = 0
x→+∞

Il grafico è riportato in fig. (5.13).


f (x) cambia segno infinite volte in A, per cui determiniamo le funzioni:

|f (x)| + f (x) |f (x)| − f (x)


f1 (x) = , f2 (x) =
2 2
5.2 Rettangoloide generalizzato 282

Π
fH 2 L

Π
x
2
Π 2Π

Figura 5.13: Grafico di f (x) = e−x sin x in [0, 2π].

Risulta:  [
−x


 e sin x, x ∈ [2kπ, (2k + 1) π]
k∈N
f1 (x) = [


 0, x ∈ [(2k + 1) π, (2k + 2) π]
k∈N
 [
−x


 −e sin x, x ∈ [(2k + 1) π, (2k + 2) π]
k∈N
f2 (x) = [


 0, x ∈ [2kπ, (2k + 1) π]
k∈N

Restano così definiti i rettangoloidi generalizzati:



U1 = (x, y) ∈ R2 | x ∈ A, 0 ≤ y ≤ f1 (x)

U2 = (x, y) ∈ R2 | x ∈ A, − f2 (x) ≤ y ≤ 0

Poniamo: 
W = (x, y) ∈ R2 | x ∈ A, 0 ≤ y ≤ e−x ,
donde:
Z+∞
misW = e−x dx = 1
0

Le curve y = e sin x, y = e sono tangenti nei punti di intersezione xk =


−x −x
π
2
+ 2kπ come possiamo vedere in fig. (25).
Poniamo: 
U2′ = (x, y) ∈ R2 | x ∈ A, 0 ≤ y ≤ f2 (x)
5.2 Rettangoloide generalizzato 283

Π
fH 2 L

Π
x
2
Π

Evidentemente:
misU2 = misU2′
Inoltre dal grafico di fig. (5.14).
y

0.4

0.3

0.2

0.1

Π
x
2
Π 2Π

Figura 5.14:

segue:
misU1 + misU2′ < misW = 1 =⇒ misU1 + misU2 < 1
Cioè:
misU1 < +∞, misU2 < +∞
Si conclude che f (x) è integrabile in A. Per il calcolo dell’integrale, assumiamo:
Dn = [0, n]
5.2 Rettangoloide generalizzato 284

Quindi:
Z+∞ Zn
−x
e sin xdx = lim e−x sin xdx
n→+∞
0 0
Rn
L’integrale 0
−x
e sin xdx si calcola per parti:
Zn
1 1
e−x sin xdx = − e−n [sin (n) + cos (n)] +
2 2
0

Osservando che:
lim e−n [sin (n) + cos (n)] = 0,
n→+∞

si conclude:
Z+∞
1
e−x sin xdx =
2
0

Esempio 26. Studiamo l’integrabilità della funzione:


 x
e sin x, x ∈ [0, π] ∪ [2π, 3π] ∪ [4π, 5π] ∪ ...
f (x) = −x
e sin x, x ∈ [π, 2π] ∪ [3π, 4π] ∪ [5π, 6π] ∪ ...

nell’intervallo A = [0, +∞].


Osserviamo che il grafico di f (x) è un’oscillazione sinusoidale il cui inviluppo di
modulazione è y = ex per le semionde positive e y = e−x per le semionde negative.
 x
e sin x, x ∈ [0, π] ∪ [2π, 3π] ∪ [4π, 5π] ∪ ...
f1 (x) =
0, x ∈ [π, 2π] ∪ [3π, 4π] ∪ [5π, 6π] ∪ ...

−e−x sin x, x ∈ [π, 2π] ∪ [3π, 4π] ∪ [5π, 6π] ∪ ...
f2 (x) =
0, x ∈ [0, π] ∪ [2π, 3π] ∪ [4π, 5π] ∪ ...
Quindi i rettangoloidi:

U1 = (x, y) ∈ R2 | x ∈ A, 0 ≤ y ≤ f1 (x)

U2 = (x, y) ∈ R2 | x ∈ A, − f2 (x) ≤ y ≤ 0
Appendice A

Integrali notevoli
Z Z
A.1 sin2 xdx, cos2 xdx

Z
1 1
sin2 xdx =
(x − sin x cos x) + C = (2x − sin 2x) + C (A.1)
2 4
Z
1 1
cos2 xdx = (x + sin x cos x) + C = (2x + sin 2x) + C
2 4
Z
dx
A.2 n
( −1)
x 2

Z
1. dx
x2 −1
= − arctanh x+const
Z

2. dx
(x2 −1)2
= x
2(1−x2 )
− 14 ln x−1
x+1
+const

Z
x(3x2 −5)
3. dx
(x2 −1)3
= 8(x2 −1) 2 + 3
16
ln x−1
x+1
+const

Z
4. dx
(x2 −1)4
1
= − 48(−1+x) 3 +
1
16(−1+x)2
− 5
32(−1+x)
− 5
32
ln (−1 + x) − 1
48(1+x)3

1 5 5
16(1+x)2
− 32(1+x) + 32 ln (1 + x)
Z
5. dx
(x2 −1)5
1
= − 128(−1+x) 4+
5
192(−1+x)3
15
− 256(−1+x) 35 35
2 + 256(−1+x) + 256 ln (−1 + x)+
1 5 15 35 35
128(1+x)4
+ 192(1+x) 3 +
256(1+x)2
+ 256(1+x) − 256 ln (1 + x)
Z
dx
A.3 (x2 +1)n
286

Z
6. dx
(x2 −1)6
1
= − 320(−1+x) 5 +
3
256(−1+x)4
7
− 256(−1+x) 3 +
7
128(−1+x)2
63
− 512(−1+x) −
63 1 3 7 7 63 63
ln (−1 + x)− 320(1+x) 5− − 256(1+x) 3− − 512(1+x) + 512 ln (1 + x)
512 256(1+x)4 128(1+x)2
Z
7. dx
(x2 −1)7
1
= − 768(−1+x) 6 +
7
1280(−1+x)5
7
− 512(−1+x) 4 +
7
256(−1+x)3
105
− 2048(−1+x) 2 +
231 231 1 7 7 7
2048(−1+x)
+ 2048 ln (−1 + x) + 768(1+x) 6 +
1280(1+x)5
+ 512(1+x) 4 +
256(1+x)3
+
105 231 231
2048(1+x)2
+ 2048(1+x) − 2048 ln (1 + x)
Z
8. dx
(x2 −1)8
1
= − 1792(−1+x) 7 +
1
384(−1+x)6
9
− 1280(−1+x) 5 +
15
1024(−1+x)4
55
− 2048(−1+x) 3 +
99 429 429 1 1 9
2048(−1+x)2
− 4096(−1+x) − 4096 ln (−1 + x) − 1792(1+x) 7 −
384(1+x)6
− 1280(1+x) 5 −
15 55 99 429 429
1024(1+x)4
− 2048(1+x)3 − 2048(1+x)2 − 4096(1+x) + 4096 ln (1 + x)

Z
dx
A.3 n
( x2 +1 )
Z
1. dx
x2 +1
= arctan x+const
Z
2. dx
(x2 +1)2
= 1 x
2 x2 +1
+ 12 arctan x+const
Z
3. dx
(x2 +1)3
= 1 x
4 (x2 +1)2
+ 3 x
8 x2 +1
+ 38 arctan x
Z
4. dx
(x2 +1)4
= 1 x
6 (x2 +1)3
+ 5 x
24 (x2 +1)2
+ 5 x
16 x2 +1
+ 5
16
arctan x+const
Z
5. dx
(x2 +1)5
= 1 x
8 (x2 +1)4
+ 7 x
48 (x2 +1)3
+ 35 x
192 (x2 +1)2
+ 35 x
128 x2 +1
+ 35
128
arctan x+const
Z
6. dx
(x2 +1)6
= 1 x
10 (x2 +1)5
9
+ 80 x
(x2 +1)4
21
+ 160 x
(x2 +1)3
21
+ 128 x
(x2 +1)2
63
+ 256 x
+ 63
x2 +1 256
arctan x+const
Z
7. dx
(x2 +1)7
= 121 x
(x2 +1)6
11
+ 120 x
(x2 +1)5
+ 33 x
320 (x2 +1)4
+ 77 x
640 (x2 +1)3
+ 77 x
512 (x2 +1)2
+
231 x 231
1024 x2 +1
+ 1024 arctan x
Z
8. dx
(x2 +1)8
1
= 14 x
(x2 +1)7
13
+ 168 x
(x2 +1)6
143
+ 1680 x
(x2 +1)5
+ 429 x
4480 (x2 +1)4
+ 143 x
1280 (x2 +1)3
+
143 x 429 x 429
1024 (x2 +1)2
+ 2048 x2 +1
+ 2048 arctan x
Z
9. dx
(x2 +1)9
1
= 16 x
(x2 +1)8
15
+ 224 x
(x2 +1)7
+ 89665 x
(x2 +1)6
143
+ 1792 x
(x2 +1)5
+ 1287 x
14 336 (x2 +1)4
+
429 x
4096 (x2 +1)3
+ 162145 x
384 (x2 +1)2
+ 326435 x
768 x2 +1
+ 326435
768
arctan x
Z Z
dx dx
A.4 (sin x)n
; (cos x)n
287

Z
10. dx
(x2 +1)10
1
= 18 x
(x2 +1)9
17
+ 288 x
(x2 +1)8
85
+ 1344 x
(x2 +1)7
+ 161105 x
128 (x2 +1)6
+ 322431 x
256 (x2 +1)5
+
2431 x
28 672 (x2 +1)4
+ 242431 x
576 (x2 +1)3
+ 12 155 x
98 304 (x2 +1)2
+ 12 155 x
65 536 x2 +1
+ 12 155
65 536
arctan x
Z
11. dx
(x2 +1)11
= 201 x
(x2 +1)10
19
+ 360 x
(x2 +1)9
323
+ 5760 x
(x2 +1)8
323
+ 5376 x
(x2 +1)7
+ 644199 x
512 (x2 +1)6
+
46 189 x 46 189 x 46 189 x 46 189 x 46 189 x 46 189
645 120 (x2 +1)5
+ 573 440 (x2 +1)4
+ 491 520 (x2 +1)3
+ 393 216 (x2 +1)2
+ 262 144 x2 +1
+ 262 144
arctan x
Z Z
dx dx
A.4 (sin x)n
; (cos x)n

Applicando le (3.108)-(3.109) per valori crescenti di n = 3, 4, 5:

Z
dx 1 h  x  cos x i

= ln tan − +C
sin3 x 2 2 sin2 x
Z   x π  
dx 1 sin x
= ln tan + + +C
cos3 x 2 2 4 cos2 x
Z
dx 1h cos x i
= − 2 cot x + +C
sin4 x 3 sin3 x
Z  
dx 1 sin x
= tan x + +C
cos4 x 3 cos3 x
Z   x  cos x  cos x 
dx 1 3 
= ln tan − − +C
sin5 x 4 2 2 sin2 x sin4 x
Z    x π   
dx 1 3 sin x sin x
= ln tan + + + +C
cos5 x 4 2 2 4 cos2 x cos4 x
Z Z
A.5 (tan x)n dx; (cot x)n dx

Applicando le (3.112)-(3.113) per n = 2, 3, ..., 8:


Z Z
n
A.5 (tan x) dx; (cot x)n dx 288

Z
tan2 xdx = tan x − x + C
Z
cot2 xdx = − cot x − x + C
Z
1 1 
tan3 xdx = tan2 x − ln 1 + tan2 x + C
2 2
Z
1 1 
cot3 xdx = − cot2 x + ln 1 + cot2 x
2 2
Z
1
tan4 xdx = tan3 x − tan x + x + C
3
Z
1
cot4 xdx = − cot3 x + cot x + x + C
3
Z
1 1 1 
tan5 xdx = tan4 x − tan2 x + ln 1 + tan2 x + C
4 2 2
Z
1 1 1 
cot5 xdx = − cot4 x + cot2 x − ln 1 + cot2 x + C
4 2 2
Z
1 1
tan6 xdx = tan5 x − tan3 x + tan x − x + C
5 3
Z
1 1
cot6 xdx = − cot5 x + cot3 x − cot x − x + C
5 3
Z
1 1 1 1 
tan7 xdx = tan6 x − tan4 x + tan2 x − ln 1 + tan2 x + C
6 4 2 2
Z
1 1 1 1 
cot7 xdx = − cot6 x + cot4 x − cot2 x + ln 1 + cot2 x + C
6 4 2 2
Z
1 1 1
tan8 xdx = tan7 x − tan5 x + tan3 x − tan x + x + C
7 5 3
Z
1 1 1
cot8 xdx = − cot7 x + cot5 x − cot3 x + cot x + x + C
7 5 3

You might also like